NUR318 Exam III Study Guide Maternal Health Nursing

Ace your homework & exams now with Quizwiz!

• Pregestational diabetes mellitus is diabetes that existed before pregnancy. Gestational diabetes mellitus is glucose intolerance that begins or is first recognized during pregnancy.

...

• Pregnant women with diabetes are much more likely to develop hypoglycemia than hyperglycemia.

...

• Premature rupture of membranes (PROM) is the spontaneous rupture of the amniotic sac and leakage of amniotic fluid before the onset of labor at any gestational age. Preterm premature rupture of membranes (PPROM) is a rupture before 37 weeks of gestation.

...

• Preterm infants (infants born before 37 weeks of gestation) are at risk because their organ systems are immature and because they do not have adequate physiologic reserves to function in an extrauterine environment.

...

• Preterm infants are vulnerable to fluid depletion and may receive supplemental parenteral fluids to supply additional calories, electrolytes, or water.

...

• Preterm labor consists of cervical changes and uterine contractions occurring between 20 and 37 weeks of pregnancy. Preterm birth is a birth occurring before the completion of 37 weeks of gestation.

...

• Rh incompatibility occurs when an RhD-negative mother has an RhD-positive fetus who inherits the dominant Rh-positive gene from the father.

...

• Sepsis is one of the most significant causes of neonatal morbidity and mortality.

...

• Severe preeclampsia, HELLP syndrome, and gram-negative sepsis can trigger disseminated intravascular coagulation.

...

• Skin-to-skin contact, also called kangaroo care, and short periods of gentle massage can help reduce stress in preterm infants.

...

• Suicidal thoughts require immediate assessment and intervention.

...

• Synthetic oxytocin (Pitocin) may be used to induce or augment labor.

...

• The acronym TORCH stands for Toxoplasmosis, Other (gonorrhea, hepatitis B, syphilis, varicella-zoster virus, parvovirus B19, and human immunodeficiency virus), Rubella, Cytomegalovirus, and Herpes simplex virus.

...

• The cause of preterm labor is unknown.

...

• The classic signs of hemorrhagic shock may not appear until the postpartum woman has lost as much as 40% of her blood volume.

...

• The daily fetal movement count, also called the kick count, is frequently used to monitor the fetus in pregnancies complicated by conditions that can affect fetal oxygenation.

...

• The effect of pregnancy on asthma is unpredictable.

...

• The four most common types of hypertensive disorders during pregnancy are gestational hypertension, preeclampsia, chronic hypertension, and preeclampsia superimposed on chronic hypertension.

...

• The goal of care for the preterm infant is to promote normal growth and development by providing an extrauterine environment that approximates a healthy intrauterine environment.

...

• The goals of oxygen therapy are to provide adequate oxygen to the tissues, prevent lactic acid accumulation, and avoid the effects of hyperoxia and free radicals.

...

• The incidence of congenital malformations is related to the severity and duration of the diabetes.

...

• The incidence of urinary incontinence is higher among women who have given birth than among nulliparous women.

...

• The main cause of diabetes-associated birth defects is hyperglycemia during the first trimester, when organ systems are forming.

...

• The major cardiovascular changes of pregnancy that affect women who have cardiac disease include increased intravascular volume, decreased systemic vascular resistance, cardiac output changes during labor and birth, and intravascular volume changes just after birth.

...

• The major causes of bleeding in late pregnancy are placenta previa and premature separation of the placenta.

...

• The major risk factors for spontaneous preterm birth include a previous preterm birth, multiple gestations, bleeding after the first trimester of pregnancy, and a low maternal body mass index.

...

• The most common maternal complication of preterm PROM is infection of the amniotic cavity, called chorioamnionitis.

...

• The most common medical complication of pregnancy is urinary tract infection.

...

• The most common postpartum infection is endometritis. Other infections include wound infections, urinary tract infections, and mastitis.

...

• The most common serious autoimmune disease affecting women of reproductive age is systemic lupus erythematosus.

...

• The most commonly used tocolytic is magnesium sulfate.

...

• The nonstress test is the most widely used technique for antepartum evaluation of the fetus.

...

• The pregnant woman with hyperemesis gravidarum may experience weight loss, dehydration, electrolyte imbalances, nutritional deficiencies, and ketonuria.

...

• The primary factor in maintaining euglycemia during pregnancy is adequate insulin therapy.

...

• The signs and symptoms of miscarriage depend on the duration of pregnancy.

...

• The successful discharge of a high risk infant requires a multidisciplinary approach.

...

• The three classic symptoms of ectopic (or tubal) pregnancy are abdominal pain, delayed menses, and abnormal vaginal bleeding (spotting) 6 to 8 weeks after the last normal menstrual period.

...

• The view provided by ultrasound scanning helps make invasive tests, such as amniocentesis, safer.

...

• Tocolytics are used to delay birth long enough to institute interventions that reduce neonatal morbidity and mortality.

...

• Typically, infants born before 32 weeks of gestation do not have adequate amounts of pulmonary surfactant. Surfactant replacement therapy may be administered as an adjunct to oxygen and ventilation therapy.

...

• Using magnetic resonance imaging, an examiner can evaluate fetal structure and growth; the placenta; the quantity of amniotic fluid; maternal structures; the biochemical status of tissues and organs; and soft-tissue, metabolic, and functional anomalies.

...

• Uterine displacement and prolapse may result from congenital or acquired weakness of the pelvic support structures.

...

• When postpartum bleeding continues and the source cannot be identified, the woman's coagulation status should be assessed.

...

• When the woman is undergoing examinations such as ultrasonography, magnetic resonance imaging, chorionic villus sampling, percutaneous umbilical blood sampling, and amniocentesis, the nurse acts as educator and support person.

...

• With frequent maternal and fetal evaluation, a woman who has mild gestational hypertension or mild preeclampsia may be managed at home.

...

• Women diagnosed with severe gestational hypertension or severe preeclampsia should be hospitalized for a 24-hour observation period.

...

• Women of reproductive age who have diabetes should have preconception counseling.

...

• Women who have postpartum depression are usually treated with antidepressant drug therapy.

...

• Women with diabetes mellitus must maintain strict glucose control before conception and during pregnancy.

...

respiratory distress syndrome (RDS)

Condition resulting from decreased pulmonary gas exchange, leading to retention of carbon dioxide (increase in arterial partial pressure of carbon dioxide [pCO2]); most common neonatal causes are prematurity and perinatal asphyxia, and maternal diabetes mellitus; also called hyaline membrane disease

• Asthma may be the most common serious medical condition to complicate pregnancy.

...

• Because of advances in noninvasive screening techniques, the use of amniocentesis and chorionic villus sampling is declining.

...

• Before effective relationships and parenting patterns can evolve, the parents of a preterm infant must accomplish several psychologic tasks.

...

• Bladder training and pelvic muscle exercises, called Kegel exercises, may reduce or relieve mild to moderate urinary incontinence.

...

• Causes of postpartum hemorrhage include uterine atony, genital tract lacerations, retained placenta, inversion of the uterus, and subinvolution of the uterus.

...

• Cervical ripening using chemical measures, such as administering prostaglandins, or mechanical measures, such as inserting a balloon catheter, can increase the success of labor induction.

...

• Cesarean birth is the birth of a fetus through a transabdominal incision in the uterus.

...

• Chronic hypertension is hypertension that begins before pregnancy or is diagnosed before 20 weeks of gestation.

...

• Common bleeding disorders of early pregnancy include miscarriage (spontaneous abortion), premature dilation of the cervix, ectopic pregnancy, and hydatidiform mole.

...

• Developmental care is tailored to the infant's developmental level and tolerance based on a comprehensive behavioral assessment.

...

• Developmental intervention activities are based on the infant's cues, temperament, state, behavioral organization, and needs.

...

• Diagnostic ultrasonography provides information on fetal activity, growth, anatomy, well-being, and gestational age.

...

• During pregnancy, the most common nongynecologic abdominal conditions requiring surgery are appendicitis and symptomatic cholelithiasis.

...

• During the postpartum period, new mothers may experience mood disorders. Nurses should offer anticipatory guidance, assess the mental health of the mother, offer therapeutic interventions, and provide referral if necessary.

...

• Dystocia—a long, difficult, or abnormal labor—occurs in 8% to 11% of all births.

...

• Early signs of respiratory distress include tachypnea, nasal flaring, and expiratory grunting.

...

• Evidence supports the use of parenteral nutrition within hours of birth and minimal enteral feedings within the first 5 days of life.

...

• Exchange transfusion is the repetitive withdrawal of small amounts of the infant's blood and replacement with donor blood.

...

• Gavage feeding provides breast milk or formula through a nasogastric or orogastric tube.

...

• Gestational hypertension is hypertension without proteinuria that develops after week 20 of gestation.

...

• Grieving parents and other family members look to the medical and nursing staff for guidance regarding decisions they must make.

...

• HELLP syndrome, a variant of severe preeclampsia, involves hepatic dysfunction characterized by hemolysis (H), elevated liver enzymes (EL), and low platelets (LP).

...

• Health problems that occur in infants of mothers with diabetes include congenital anomalies, macrosomia, birth injury, respiratory distress syndrome, hypoglycemia, cardiomyopathy, hyperbilirubinemia, and polycythemia.

...

• Heart disease complicates 0.5% to 2% of pregnancies.

...

• Hemorrhage may lead to hemorrhagic shock, also called hypovolemic shock.

...

• High risk infants are classified by birth weight, gestational age, and predominant pathophysiologic problems.

...

• Hydatidiform mole, a benign proliferative growth of the placental trophoblast, occurs in 1 of every 1,000 pregnancies in the United States.

...

• Hyperthyroidism in pregnancy is rare, and hypothyroidism during pregnancy is less common than hyperthyroidism.

...

• If a disorder jeopardizes the life or well-being of the mother or infant, the pregnancy is considered high risk.

...

• In the United States, major congenital defects are the leading cause of death in infants under age 1.

...

• In the last 20 years, the incidence of postpartum thromboembolic disease has decreased because early ambulation after childbirth has become standard practice.

...

• Indications for amniocentesis include prenatal diagnosis of genetic disorders or congenital anomalies (especially neural tube defects), assessment of pulmonary maturity, and diagnosis of fetal hemolytic disease.

...

• Induction of labor is indicated when continuing the pregnancy could be dangerous for the woman or fetus and when there are no contraindications to artificial rupture of membranes or augmentation of uterine contractions.

...

• Interventions for dystocia include external cephalic version, cervical ripening, induction or augmentation of labor, forceps- or vacuum-assisted birth, and cesarean birth.

...

• Late preterm infants who make an effective transition to extrauterine life remain at risk for problems with thermoregulation, hypoglycemia, hyperbilirubinemia, sepsis, and respiratory function.

...

• Magnesium sulfate is the drug of choice for preventing and treating convulsions caused by preeclampsia.

...

• Major trauma complicates about 8% of pregnancies, and trauma is the leading cause of nonobstetric maternal death.

...

• Many health care providers recommend modified bed rest for women at high risk of preterm birth.

...

• Maternal behaviors such as drug abuse, tobacco smoking, and alcohol consumption are hazardous to the fetus and neonate.

...

• Maternal causes of dystocia include ineffective uterine contractions or bearing-down efforts, changes in the pelvic structure, positioning during labor and birth, and psychologic responses to labor. Fetal causes include anomalies, excessive fetal size, malpresentation, malposition, and multifetal pregnancy.

...

• Maternal complications for women with diabetes include miscarriage, infection, ketoacidosis, and difficult labor.

...

• Maternal complications of cesarean birth include aspiration, hemorrhage, atelectasis, endometritis, abdominal wound dehiscence or infection, urinary tract infection, injuries to the bladder or bowel, and complications related to anesthesia.

...

• Medical management of recurrent premature dilation of the cervix includes bed rest, pessaries, antibiotics, antiinflammatory drugs, and progesterone supplementation. Surgical management consists of placing a cervical cerclage.

...

• More than half of preterm births occur in women without obvious risk factors.

...

• Most infant deaths and the most serious morbidity occur in the 16% of preterm infants who are born before 32 weeks of gestation.

...

• Motor-vehicle accidents account for most cases of blunt-force abdominal trauma during pregnancy.

...

• Nurses help bereaved parents acknowledge and express their feelings.

...

• Nurses should teach pregnant women to detect the early symptoms of preterm labor.

...

• Nursing care of parents experiencing a perinatal loss begins when they are first faced with the potential loss of the pregnancy or death of their infant.

...

• Obstetric emergencies include meconium-stained amniotic fluid, shoulder dystocia, prolapsed cord, rupture of the uterus, and anaphylactoid syndrome of pregnancy.

...

• One of the most effective interventions is antenatal glucocorticoid administration, which accelerates fetal lung maturity.

...

• Postpartum depression often goes undetected because women are hesitant to report its symptoms.

...

• Postpartum hemorrhage, a life-threatening event, often goes unrecognized until the mother has profound symptoms.

...

• Preeclampsia is a condition in which hypertension and proteinuria develop after 20 weeks of gestation in a previously normotensive woman. Preeclampsia complicates 3% to 7% of all pregnancies.

...

• Preeclampsia superimposed on chronic hypertension increases morbidity for the mother and fetus.

...

Chapter 19: Assessment of High Risk Pregnancy

...

Chapter 20: Pregnancy at Risk: Preexisting Conditions

...

Chapter 21: Pregnancy at Risk: Gestational Conditions

...

Chapter 22: Labor and Birth at Risk

...

Chapter 23: Postpartum Complications

...

Chapter 24: The Newborn at Risk

...

Critical Thinking/Cinical Decision Making: Trial of Labor for Vaginal Birth after Cesarean (TOL/VBAC) Heather, a 28-year-old G2 P1 gave birth by cesarean during her last pregnancy. During her routine prenatal visit at 32 weeks of gestation, Heather tells the nurse that she really wants to have a vaginal birth this time. Heather asks, "What do you think? Can I try for a VBAC?"

...

Critical Thinking/Clinical Decision Making: Ectopic Pregnancy Kendra is a 19-year-old G1 P0 who enters the emergency department with lower abdominal pain and vaginal spotting. She reports that her last menstrual period was 6 weeks ago. A pregnancy test is positive.

...

Critical Thinking/Clinical Decision Making: Fetal Assessment Using the Nonstress Test LaTonya is a 30-year-old G5 T3 P0 A1 L3 who is now at 41 weeks of gestation. Her physician has scheduled her for twice-weekly nonstress testing, and this appointment is her first. You are the nurse assigned to perform LaTonya's nonstress test (NST) today. As you help her get comfortable and attach the fetal heart rate and contraction monitors, LaTonya grumbles, "I don't see why I had to come get this test done. It was really hard to find a babysitter for my other kids, and I live on the other side of town!"

...

Critical Thinking/Clinical Decision Making: Late Preterm Infant A 2013-g (4-lb 7-oz) male infant is born at an estimated gestational age of 35 weeks. The parents are very excited about this birth because they have been trying to become pregnant for 6 years. The baby is placed on the mother's (Lucia's) abdomen after birth for skin-to-skin contact but does not breastfeed. The nurse assessing the baby notes that he has some mild grunting, nasal flaring, and intercostal retractions; he is taken to the transitional nursery for further evaluation and treatment. Jorge, the father, speaks little English but asks when they will be able to hold their son again. Lucia is crying and asks to have her baby brought back to her as soon as his condition is stable because she really wants to breastfeed him.

...

Critical Thinking/Clinical Decision Making: Postpartum Hemorrhage You are a member of the interdisciplinary team for the labor and birth unit at your hospital. The team has been asked to develop a protocol for managing postpartum hemorrhage (PPH). Your assignment is to identify the nursing role during the early postpartum period. What will you include in your report?

...

Critical Thinking/Clinical Decision Making: Sickle Cell Hemoglobinopathy Latasha is a 23-year-old G1 P0 with sickle cell anemia who is hospitalized with a crisis at 16 weeks of gestation. Latasha says, "I've been in and out of the hospital all my life because of my sickle cell disease. I sure hope my baby won't have this disease!"

...

• A diagnosis of a high risk pregnancy may cause the mother to exhibit psychologic responses including anxiety, low self-esteem, guilt, frustration, and an inability to function.

...

• A kick count of fewer than three in 1 hour warrants further evaluation.

...

• A mother may experience postpartum depression with or without psychotic features.

...

• A postterm pregnancy poses risks to the mother and the fetus.

...

• A pregnant woman diagnosed with pyelonephritis should be admitted to the hospital immediately and treated with IV antibiotics.

...

• A pregnant woman may require cardiopulmonary resuscitation if she sustains trauma or has an airway obstruction caused by choking.

...

• A pregnant woman with diabetes is monitored more frequently and thoroughly than other pregnant women.

...

• A risk to pregnancy may be biophysical, psychosocial, sociodemographic, or environmental.

...

• About 25% of women with mitral stenosis first become symptomatic during pregnancy. Maternal mortality is linked to functional capacity.

...

• About 70% of patients with an autoimmune disease are women of childbearing age.

...

• About 90% of pregnant women with diabetes have gestational diabetes mellitus.

...

• After a cesarean birth, the nurse should stress that the woman is a new mother first and a surgical patient second.

...

• After a pregnant woman sustains trauma, the first priorities are to resuscitate her and stabilize her condition. Fetal survival depends on maternal survival.

...

• After a woman experiences preeclampsia the first time, the chances that she will experience it in a future pregnancy increase sevenfold.

...

• An infant whose gestation is prolonged beyond 42 weeks is considered a postmature infant.

...

• Anemia affects as many as 60% of pregnant women.

...

• Any use of alcohol or illicit drugs during pregnancy is considered abuse.

...

• As pregnancy progresses, insulin requirements increase for a woman with diabetes.

...

1. Evidence—Is evidence sufficient to draw conclusions about what to tell Lucia and Jorge about their infant son?

1. Yes, there is now sufficient evidence. Lucia and Jorge can be informed that the infant will undergo a comprehensive evaluation in the transitional nursery. Supplemental oxygen may be administered and a diagnostic radiograph obtained. Because the infant is near term, a comprehensive evaluation is important. It would help the parents for the primary nurse caring for the infant to remain in close contact with Lucia and Jorge and keep them informed of the results of the various tests and the infant's condition.

1. Evidence—Is evidence sufficient to draw conclusions about information to include in the report about the nurse's role in caring for the woman with early PPH?

1. Yes.

1. What additional information is necessary to counsel Latasha regarding her baby's chance of having sickle cell disease?

1. Latasha's partner (the baby's father) must be tested to determine his sickle cell status before she can be counseled regarding her baby's chances of inheriting sickle cell trait or sickle cell disease.

1. Evidence—Does the nurse have sufficient evidence at this time to advise Heather about the safety and feasibility of a trial of labor (TOL) for vaginal birth after cesarean (VABC)? If not, what data should be collected?

1. No. At a minimum, the nurse must know the type of scar that is present on Heather's uterus and the reason for Heather's previous cesarean. If the cesarean incision involved the contractile portion of the uterus (a classic uterine scar), VBAC is not an option because the risk of uterine rupture is too great. Many indications for a primary cesarean birth, for example, breech presentation, may not recur in subsequent pregnancies, making a VBAC feasible.

1. Does the nurse have sufficient evidence at this time to diagnose ectopic pregnancy?

1. No. Kendra's symptoms could be associated with many different problems, including a variety of obstetric/gynecologic and medical/surgical conditions.

1. What is the purpose of performing fetal assessment tests, such as the NST, late in pregnancy?

1. The purpose of third trimester fetal assessment is to determine whether the intrauterine environment continues to be supportive to the fetus. The testing is often used to determine the timing of childbirth for women at risk for uteroplacental insufficiency (UPI).

2. What assumptions can be made about the following issues? a. The chance that Latasha's baby will inherit either sickle cell trait or sickle cell disease b. Pregnancy risks related to sickle cell disease

2. Assumptions. a. Latasha has sickle cell disease, so every child born to her will be affected in some way by the disease because she can only contribute an abnormal (sickle) gene to the child. If her partner has normal hemoglobin, this baby will have sickle cell trait. If Latasha's partner has sickle cell trait, the baby will either have sickle cell trait or sickle cell disease, depending on whether the normal or abnormal gene is inherited from the father. Finally, if the father has sickle cell disease, the baby will also have sickle cell disease, because both parents can only contribute an abnormal (sickle) gene to their offspring. b. Women with sickle cell disease are at risk for poor pregnancy outcomes, including spontaneous abortion, intrauterine growth restriction (IUGR), and stillbirth. Although maternal mortality is rare, maternal morbidity is significant, and includes an increased risk for preeclampsia and infection, particularly in the urinary tract and in the lungs. The frequency of painful crises also appears to be greater during pregnancy.

2. What assumptions can be made about the following issues? a. Possible diagnoses for Kendra b. Laboratory and diagnostic tests necessary to diagnose ectopic pregnancy c. If her examination reveals that Kendra has an ectopic pregnancy, criteria that must be met to treat this condition medically with methotrexate

2. Assumptions. a. Possible diagnoses for Kendra include miscarriage, ruptured corpus luteum cyst, appendicitis, salpingitis, ovarian cysts, torsion of the ovary, and urinary tract infection. b. Laboratory screening includes determination of serum progesterone and β-hCG levels. If either of these values is lower than would be expected for a normal pregnancy, the woman is asked to return within 48 hours for serial measurements. Transvaginal ultrasonography is done to confirm intrauterine or tubal pregnancy. By this time in Kendra's pregnancy, an intrauterine sac should be visible on ultrasound. c. Hemodynamically stable women with ectopic pregnancies are eligible for methotrexate therapy if the mass is unruptured and measures less than 3.5 cm in diameter by ultrasound, if there is no fetal cardiac activity noted on ultrasound, if the serum β-hCG level is less than 5000 international units/L, and if the woman is willing to comply with posttreatment monitoring.

2. What assumptions can be made about the following issues? a. The physiologic principle on which the NST is based b. Advantages of the NST c. Disadvantages of the NST d. The desired result of the NST

2. Assumptions. a. The basis for the nonstress test (NST) is the assumption that the normal fetus will produce characteristic heart rate patterns in response to fetal movement. In the term fetus, accelerations are associated with movement more than 85% of the time. b. Advantages of the NST are (1) it can be performed easily and quickly in an outpatient setting because it is noninvasive, (2) it is relatively inexpensive, and (3) it has no known contraindications. c. Disadvantages of the NST are (1) it requires repeat testing at least once (usually twice) a week; (2) it has a high false positive rate; and (3) the NST is slightly less sensitive in detecting fetal compromise than are the contraction stress test (CST) or biophysical profile (BPP). d. The desired result for the NST is a reactive test. A reactive test is one that produces at least two accelerations in a 20-minute period. Each acceleration should last at least 15 seconds and peak at least 15 beats per minute above the baseline FHR.

2. Assumptions—What assumptions can be made about the following issues related to TOL and VBAC? a. Risks that Heather faces if she chooses a TOL for VBAC b. Criteria that must be met for Heather to attempt a TOL for VBAC c. Labor management practices that facilitate a successful VBAC d. Labor management techniques that should be avoided during a TOL for VBAC

2. Assumptions. a. The biggest risk that Heather faces with a TOL/VBAC is the possibility of uterine rupture. Other risks include possible operative injury and hysterectomy. b. The American College of Obstetricians and Gynecologists (ACOG, 2004) recommends that TOL/VBAC be attempted only if all of the following criteria are met: • One previous low-transverse cesarean birth • Clinically adequate pelvis • No other uterine scars or history of previous rupture • Physician immediately available throughout active labor who is capable of monitoring labor and performing an emergency cesarean birth if necessary • Availability of anesthesia and personnel for emergency cesarean birth c. A TOL for VBAC is more likely to be successful if labor is spontaneous, rather than induced or augmented. Interventions that facilitate a successful VBAC include one-to-one support, ambulation, maternal position changes, and nonpharmacologic pain relief measures. d. Certain medications, such as prostaglandins, should not be used for cervical ripening or labor induction/augmentation in women undergoing a TOL for VBAC.

2. Assumptions—What assumptions can be made about the following? a. The mother's and the father's reaction to their son's birth b. The infant's expected progress c. The possibility of Lucia breastfeeding the baby

2. Assumptions. a. The parents' reaction is very normal. This birth was a much anticipated event; parents' expectations were quite high; for the infant to be sick, even if mildly ill, and for the parents to not be able to interact with the infant after the birth is extremely disappointing. The parents' reactions may closely parallel those of parents who lose a baby—the initial feelings of shock and denial, followed by bargaining, with anger aimed at the nursing and medical staff for removing their newborn, would not be unusual. The staff can help the parents by allowing them to visit the transitional nursery and see and touch the baby; depending on the infant's need for oxygen, allowing the mother and father to hold the baby would be optimal. Adequate thermoregulation can be maintained with skin-to-skin contact. Effective communication is an essential component in interactions with the parents of a preterm infant. b. Late preterm infants may experience some of the same transitional problems as do less mature infants. Respiratory distress, hypoglycemia, and temperature instability are possible complications of near-term infants. However, until more data regarding the infant's status is available, no further conclusions may be drawn. Most near-term infants without associated anomalies are able to make an effective transition to extrauterine life within the first 24 to 48 hours of life. c. Depending on the infant's respiratory status, Lucia should be able to breastfeed the baby within a few hours. Once the baby is stable, there is no reason to withhold breastfeeding attempts. Colostrum has a high glucose content and its consumption may prevent hypoglycemia. Lucia should be encouraged to pump and save her colostrum if the infant's condition prevents breastfeeding within a few hours of birth.

2. Assumptions—Describe underlying assumptions about each of following issues: a. Women at high risk for early PPH b. Frequent assessments in the early postpartum period c. Use of uterotonic medications for PPH

2. Assumptions. a. The woman's history (antepartal, intrapartal) should be reviewed for identification of risk factors. These include prolonged labor, augmented labor, rapid labor, history of postpartum hemorrhage (PPH), preeclampsia, episiotomy, overdistended uterus, operative birth, Asian or Hispanic ethnicity, and chorioamnionitis. b. The usual nursing assessments in the fourth stage of labor are vital signs and uterine (fundus and lochia) checks every 15 minutes for the first 2 hours; then every hour for 2 to 3 hours; then every 4 hours for the first 12 hours. c. Oxytocin usually is the first-line medication. See Medication Guide in Chapter 23.

3. What implications and priorities for nursing care can be drawn at this time?

3. After an ectopic pregnancy is diagnosed, priorities for nursing care include explaining management choices and supporting Kendra in her decision. If Kendra opts for medical management, the nurse should explain possible medication side effects and the behavioral/lifestyle changes that are necessary (e.g., no sexual intercourse until treatment has been completed). Kendra must also be informed that follow-up appointments for lab work will be necessary for weeks and perhaps for months. It is crucial that Kendra be willing and able to keep all follow-up appointments.

3. What implications and priorities for nursing care can be drawn at this time?

3. Priority nursing care at this time is explaining to LaTonya the purpose of performing fetal assessment during the remainder of her pregnancy. The NST helps determine whether it is safe for the pregnancy to continue or whether labor should be induced. The nurse can stress that the test is not painful and can generally be completed quickly (usually in less than an hour). Another high priority for nursing care is encouraging LaTonya to keep all her NST appointments. The nurse may also need to arrange for supportive services, for example, transportation to appointments or child care, to assist LaTonya with keeping appointments.

3. What implications and priorities for nursing care can be drawn at this time?

3. Priority nursing care for Heather is to provide her with the information she needs, such as type of prior uterine scar and reason for previous cesarean birth, to help her decide whether TOL/VBAC is a viable option for her. Assuming that Heather is deemed to be a candidate for TOL/VBAC, the nurse should also encourage her to discuss her plans with her primary health care provider as soon as possible. Some obstetric care providers, such as family medicine physicians or certified nurse midwives, may not be able to offer TOL/VBAC as an option for their patients. Many obstetricians choose not to offer this option. Also, some birthing facilities do not offer the option of TOL/VBAC because they are not prepared to provide emergency cesarean birth at all times. Heather may need to change care providers or plan to give birth at a different facility in order to attempt a TOL for VBAC. As she gets closer to her due date, it may be difficult for Heather to locate another obstetric care provider willing to accept her as a patient.

3. What implications and priorities for nursing care can be drawn at this time?

3. The nurse should explain to Latasha that the baby's sickle cell status (trait or disease) was determined at conception and cannot be altered. Latasha should be encouraged to consider sickle cell inheritance risks as she contemplates future pregnancies. The nurse should also explain that Latasha will be monitored carefully during this pregnancy for the development of urinary tract infection (UTI) or preeclampsia. In addition, she will have serial ultrasounds to monitor fetal growth and will likely have antepartum fetal testing performed regularly during the third trimester. Infections will be treated aggressively with antibiotics. If additional crises occur, they will be managed with analgesia, oxygen, and hydration. Latasha may also receive prophylactic transfusions as a way to improve oxygen-carrying capacity and suppress the synthesis of sickle hemoglobin. Another high priority for nursing care is encouraging Latasha to keep all her prenatal care appointments.

3. What implications and priorities for nursing care can be drawn at this time?

3. The priorities for nursing care of the infant include assessment of the infant's respiratory, cardiac, and metabolic status by monitoring vital signs and observing for complications. Just as important is the need for the parents to be informed of the infant's status and for them to have some type of contact with the infant. The nurse may allow the parents to visit the transitional nursery if the infant requires oxygen and cannot be taken to the mother's room. All efforts, however, should be made to reunite the couple with their baby. The assistance of a lactation consultant may be necessary if pumping and saving mother's milk is required.

3. What implications and priorities need to be included in this report?

3. This report needs to identify the risk factors for PPH and the nursing assessments needed during the early postpartum period. Patients at higher risk may need more frequent checks. Uterine atony is the most common cause of PPH; consequently, fundal checks and fundal massage, if boggy, and making sure the bladder is empty are priorities. Protocols for medication administration should be developed as well as protocols for initiating primary or secondary intravenous infusion lines for medications, fluids to replace blood loss, and volume expanders or blood products. Primary health care providers need to be notified (e.g., use SBAR [Situation-Background-Assessment-Recommendation] process). Nurses should be ready to assist with medical and surgical interventions and to offer or expand on explanations of what is happening. Continuous support of the patient and family is important.

4. Does the evidence objectively support your conclusion?

4. The available evidence supports the conclusion that the infant's sudden removal from contact with the parents represents a situational crisis that can have long-lasting consequences. There is insufficient evidence to make any further conclusions about the infant's recovery.

4. Does the evidence objectively support your conclusion?

4. The evidence indicates that TOL/VBAC is a safe procedure for women who meet the criteria listed in answer 2a above.

4. Does the evidence objectively support your conclusion?

4. Yes, see references. REFERENCES American College of Obstetricians and Gynecologists (ACOG). (2006). Postpartum hemorrhage: ACOG practice bulletin no.76, Washington, DC: ACOG. Francois K, Foley M. (2007). Antepartum and postpartum hemorrhage. In Gabbe S, Niebyl J, Simpson J, editors: Obstetrics: normal and problem pregnancies, ed 5, Philadelphia: Churchill Livingstone. Gilbert E. (2007). Manual of high risk pregnancy & delivery. 4th ed. St. Louis: Mosby.

4. Does the evidence objectively support your conclusion?

4. Yes. Methotrexate therapy avoids surgery and is a safe, effective, and cost-effective way of managing many cases of tubal pregnancy.

4. Does the evidence objectively support your conclusion?

4. Yes. Research has shown that women with sickle cell disease have worse pregnancy outcomes than women who do not have this disorder. Women with sickle cell trait are also at increased risk for pregnancy complications, including preeclampsia, intrauterine fetal death (IUFD), preterm birth and low birthweight infants, and postpartum endometritis. They also have an increased risk for UTIs, and may be deficient in iron.

4. Does the evidence objectively support your conclusion?

4. Yes. The NST has been shown to be good at predicting fetal well-being. A reactive test associated with normal amniotic fluid volume indicates fetal well-being in 99% of pregnancies.

5. Do alternative perspectives to your conclusion exist?

5. No, unless new evidence-based practice is identified. However, postpartum assessment time intervals may be different from site to site.

5. Do alternative perspectives to your conclusion exist?

5. The rate of VBAC is decreasing in the United States. Currently, only about 9% of women with a previous cesarean birth choose to have a TOL with the goal of a VBAC.

5. Do alternative perspectives to your conclusion exist?

5. There are no alternative conclusions to those expressed above.

5. Do alternative perspectives to your conclusion exist?

5. Yes. Although the purpose of the NST is always to identify FHR accelerations, all practitioners do not use the standard definition of a reactive test, which is at least two accelerations in 20 minutes. Other criteria sometimes used for a reactive NST are three contractions in 20 minutes or two contractions in 30 minutes.

5. Do alternative perspectives to your conclusion exist?

5. Yes. If Kendra does not meet the criteria necessary for medical management or does not desire medical management, her ectopic pregnancy can effectively be treated surgically. Surgical intervention may involve removal of the entire tube (salpingectomy). If the tube has not ruptured and Kendra desires future fertility, salpingostomy may be done instead. In this procedure, an incision is made over the pregnancy site in the tube and the products of conception are gently and very carefully removed.

5. Do alternative perspectives to your conclusion exist?

5. Yes. There is no universal agreement among care providers regarding the management of pregnant patients with sickle cell disease. For example, some authorities recommend prophylactic transfusions as a way to improve oxygen-carrying capacity and suppress the synthesis of sickle hemoglobin. Others, however, believe that prophylactic transfusions do not improve fetal or neonatal outcome.

alcohol-related birth defect (ARBD)

Congenital abnormality or anomaly resulting from excessive maternal alcohol intake during pregnancy characterized by typical craniofacial and limb defects, cardiovascular defects, intrauterine growth restriction, and developmental delay; newer terminology for fetal alcohol syndrome (FAS)

-Oxytocin stress test- negative is good. Positive is not good

Contraction Stress Test The CST (or OCT) was the first widely used electronic fetal assessment test. It was devised as a graded stress test of the fetus, and its purpose was to identify the jeopardized fetus that was stable at rest but showed evidence of compromise after stress. Uterine contractions decrease uterine blood flow and placental perfusion. If this decrease is sufficient to produce hypoxia in the fetus, a deceleration in FHR will result. NURSING ALERT In a healthy fetoplacental unit, uterine contractions do not usually produce late decelerations, whereas, if underlying UPI exists, contractions will produce late decelerations. The CST provides an earlier warning of fetal compromise than the NST and with fewer false-positive results. However, in addition to the contraindications described earlier the CST is more time consuming and expensive than the NST. It is also an invasive procedure if oxytocin stimulation is required. Because of these disadvantages, the CST is infrequently used. Procedure The woman is placed in semi-Fowler position or sits in a reclining chair with a slight left tilt to optimize uterine perfusion and avoid supine hypotension. She is monitored electronically with the fetal ultrasound transducer and uterine tocodynamometer. The tracing is observed for 10 to 20 minutes for baseline rate and variability and the possible occurrence of spontaneous contractions. The two methods of CST are the nipple-stimulated contraction test and the oxytocin-stimulated contraction test. Nipple-stimulated contraction test Several methods of nipple stimulation have been described. In one approach the woman applies warm, moist washcloths to both breasts for several minutes. The woman is then asked to massage one nipple for 10 minutes. Massaging the nipple causes a release of oxytocin from the posterior pituitary. An alternative approach is for her to massage one nipple through her clothes for 2 minutes, rest for 5 minutes, and repeat the cycles of massage and rest as necessary to achieve adequate uterine activity. When adequate contractions or hyperstimulation (defined as uterine contractions lasting more than 90 seconds or five or more contractions in 10 minutes) occurs, stimulation should be stopped (Druzin et al., 2007). Oxytocin-stimulated contraction test Exogenous oxytocin also can be used to stimulate uterine contractions. An intravenous (IV) infusion is begun, and a dilute solution of oxytocin (e.g., 10 units in 1000 ml of fluid) is connected to the main line tubing through a piggyback port and delivered by an infusion pump to ensure an accurate dose. One method of oxytocin infusion is to begin at 0.5 milliunits/min and double the dose every 20 minutes until three uterine contractions of good quality, each lasting 40 to 60 seconds, are observed within a 10-minute period. A rate of 10 milliunits/min is usually adequate to elicit uterine contractions (Druzin et al., 2007). Interpretation CST results are either negative, positive, equivocal, suspicious, or unsatisfactory. If no late decelerations are observed with the contractions, the findings are considered negative (Fig. 19-13, A). Repetitive late decelerations render the test results positive (Fig. 19-13, B). Box 19-8 lists criteria for each possible test result. The desired CST result is negative because it has consistently been associated with good fetal outcomes. With a negative result the test is repeated in 1 week. Positive CST results have been associated with intrauterine fetal death, late FHR decelerations in labor, IUGR, and meconium-stained amniotic fluid. A positive CST result usually leads to hospitalization for further close observation or BOX 19-8 Interpretation of the Contraction Stress Test Negative test: At least three uterine contractions occur in a 10-minute period, with no late or significant variable decelerations. Positive test: Late decelerations occur with 50% or more of contractions (even if fewer than three contractions occur in 10 minutes). Equivocal-suspicious test: Prolonged decelerations, variable decelerations, or late decelerations occur with less than 50% of contractions. Equivocal-hyperstimulatory test: Decelerations occur in the presence of contractions more frequent than every 2 minutes or lasting longer than 90 seconds. Unsatisfactory test: Fewer than three uterine contractions in a 10-minute period or inability to obtain a continuous tracing of the fetal heart rate.

postpartum depression (PPD)

Depression occurring within 4 weeks of childbirth, lasting longer than postpartum blues and characterized by a variety of symptoms that interfere with activities of daily living and care of the baby

pregestational diabetes mellitus

Diabetes mellitus type 1 or type 2 that exists before pregnancy

preeclampsia

Disease encountered after 20 weeks of gestation or early in the puerperium; a vasospastic disease process characterized by hypertension and proteinuria

alcohol-related neurodevelopmental disorder (ARND)

Disorder in infants affected by prenatal exposure to alcohol but who do not meet the criteria for FAS; previously termed fetal alcohol effects (FAE)

7. A mother in late middle age who is certain she is not pregnant tells the nurse during an office visit that she has urinary problems and sensations of bearing down and of something in her vagina. The nurse would realize that the woman most likely is suffering from: A. Pelvic relaxation. B. Cystoceles and/or rectoceles. C. Uterine displacement. D. Genital fistulas.

B. Cystoceles and/or rectoceles. Cystoceles are protrusions of the bladder downward into the vagina; rectoceles are herniations of the anterior rectal wall through a relaxed or ruptured vaginal fascia. Both can present as a bearing down sensation with urinary dysfunction. They occur more often in older women who have borne children.

HELLP syndrome

A laboratory diagnosis for a variant of severe preeclampsia that involves hepatic dysfunction, characterized by hemolysis, elevated liver enzymes, and low platelet count

9. Possible alternative and complementary therapies for postpartum depression (PPD) for breastfeeding mothers include (choose all that apply): A. Acupressure. B. Aromatherapy. C. St. John's wort. D. Wine consumption. E. Yoga.

A. Acupressure. B. Aromatherapy. E. Yoga. Possible alternative/complementary therapies for postpartum depression include acupuncture, acupressure, aromatherapy, therapeutic touch, massage, relaxation techniques, reflexology, and yoga. St. John's wort has not been proven to be safe for women who are breastfeeding. Women who are breastfeeding and/or have a history of PPD should not consume alcohol.

2. The abuse of which of the following substances during pregnancy is the leading cause of cognitive impairment in the United States? A. Alcohol B. Tobacco C. Marijuana D. Heroin

A. Alcohol Alcohol abuse during pregnancy is recognized as one of the leading causes of cognitive impairment in the United States.

1. A laboring woman's amniotic membranes have just ruptured. The immediate action of the nurse would be to: A. Assess the fetal heart rate (FHR) pattern. B. Perform a vaginal examination. C. Inspect the characteristics of the fluid. D. Assess maternal temperature.

A. Assess the fetal heart rate (FHR) pattern. The first nursing action after the membranes are ruptured is to check the FHR. Compression of the cord could occur after rupture leading to fetal hypoxia as reflected in an alteration in FHR pattern, characteristically variable decelerations. The same initial action should follow artificial rupture of the membranes (amniotomy). Performing a vaginal examination, inspecting the characteristics of the fluid, and assessing maternal temperature are all important and should be done after the FHR and pattern are assessed. Performing a vaginal examination, inspecting the characteristics of the fluid, and assessing maternal temperature are all important and should be done after the FHR and pattern are assessed. Performing a vaginal examination, inspecting the characteristics of the fluid, and assessing maternal temperature are all important and should be done after the FHR and pattern are assessed.

9. The nurse is caring for a woman who is at 24 weeks of gestation with suspected severe preeclampsia. Which signs and symptoms would the nurse expect to observe? Select all that apply. A. Decreased urinary output and irritability B. Transient headache and +1 proteinuria C. Ankle clonus and epigastric pain D. Platelet count of less than 100,000/mm3 and visual problems E. Seizure activity and hypotension

A. Decreased urinary output and irritability C. Ankle clonus and epigastric pain D. Platelet count of less than 100,000/mm3 and visual problems Decreased urinary output and irritability are signs of severe eclampsia. A transient headache and +1 proteinuria are signs of preeclampsia and should be monitored. Ankle clonus and epigastric pain are signs of severe eclampsia. Platelet count of less than 100,000/mm3 and visual problems are signs of severe preeclampsia. Seizure activity and hyperreflexia are signs of eclampsia.

4. In caring for an immediate postpartum woman, you note petechiae and oozing from her IV site. You would monitor her closely for the clotting disorder: A. Disseminated intravascular coagulation (DIC) B. Amniotic fluid embolism (AFE) C. Hemorrhage D. HELLP syndrome

A. Disseminated intravascular coagulation (DIC) The diagnosis of DIC is made according to clinical findings and laboratory markers. Physical examination reveals unusual bleeding. Petechiae may appear around a blood pressure cuff on the woman's arm. Excessive bleeding may occur from the site of a slight trauma such as venipuncture sites. These symptoms are not associated with AFE, nor is AFE a bleeding disorder. Hemorrhage occurs for a variety of reasons in the postpartum woman. These symptoms are associated with DIC. Hemorrhage would be a finding associated with DIC and is not a clotting disorder in and of itself. HELLP is not a clotting disorder, but it may contribute to the clotting disorder DIC.

mood disorders

Disorders that have a disturbance in the prevailing emotional state as the dominant feature; cause is unknown

-Disseminated intravascular coagulation (DIC)

Disseminated intravascular coagulation (DIC), or consumptive coagulopathy, is a pathologic form of clotting that is diffuse and consumes large amounts of clotting factors, causing widespread external bleeding, internal bleeding, or both, and clotting (Cunningham et al., 2005). DIC is never a primary diagnosis. Instead, it results from some problem that triggered the clotting cascade, either extrinsically, by the release of large amounts of tissue thromboplastin, or intrinsically, by widespread damage to vascular integrity. In the obstetric population, DIC is most often triggered by the release of large amounts of tissue thromboplastin, which occurs in abruptio placentae and in retained dead fetus and anaphylactoid syndrome of pregnancy (amniotic fluid embolus) syndromes. Severe preeclampsia, HELLP syndrome, and gram-negative sepsis are examples of conditions that can trigger DIC because of widespread damage to vascular integrity (Cunningham et al., 2005; Gilbert, 2007). DIC is an overactivation of the clotting cascade and the fibrinolytic system, resulting in depletion of platelets and clotting factors, which results in the formation of multiple fibrin clots throughout the body's vasculature, even in the microcirculation. Blood cells are destroyed as they pass through these fibrin choked vessels. Thus DIC results in a clinical picture of clotting, bleeding, and ischemia (Cunningham et al.; Labelle & Kitchens, 2005). Clinical manifestations and laboratory test results are summarized in Box 21-6. Management Medical management in all cases of DIC involves correction of the underlying cause (e.g., removal of the dead fetus, treatment of existing infection or of preeclampsia or eclampsia, or removal of a placental abruption). Volume replacement, blood component therapy, optimization of oxygenation and perfusion status, and continued reassessment of laboratory parameters are the usual forms of treatment (Francois & Foley, 2007). Vitamin K administration and recombinant activated factor VIIa also may be considered as adjuvant therapies (Francois & Foley). Nursing interventions include assessment for signs of bleeding (see Box 21-6) and signs of complications from the administration of blood and blood products, administering fluid or blood replacement as ordered, cardiac and hemodynamic monitoring, and protecting the woman from injury. Because renal failure is one consequence of DIC, urinary output is closely monitored by using an indwelling Foley catheter. Urinary output must be maintained at more than 30 ml/hr (Gilbert, 2007). Vital signs are assessed frequently. If DIC develops before birth, the woman should be maintained in a side-lying tilt to maximize blood flow to the uterus. Oxygen may be administered through a nonrebreather facemask at 8 to 10 L/min or per hospital protocol or physician order. Fetal assessments are performed to monitor fetal well-being (Labelle & Kitchens, 2005). DIC usually is "cured" with the birth and as coagulation abnormalities resolve. BOX 21-6 Clinical Manifestations and Laboratory Screening Results for Women with Disseminated Intravascular Coagulation POSSIBLE PHYSICAL EXAMINATION FINDINGS • Spontaneous bleeding from gums, nose • Oozing, excessive bleeding from venipuncture site, intravenous access site, or site of insertion of urinary catheter • Petechiae, for example, on the arm where blood pressure cuff was placed • Other signs of bruising • Hematuria • Gastrointestinal bleeding • Tachycardia • Diaphoresis LABORATORY COAGULATION SCREENING TEST RESULTS • Platelets—decreased • Fibrinogen—decreased • Factor V (proaccelerin)—decreased • Factor VIII (antihemolytic factor)—decreased • Prothrombin time—prolonged • Partial prothrombin time—prolonged • Fibrin degradation products—increased • D-dimer test (specific fibrin degradation fragment)—increased • Red blood smear—fragmented red blood cells The woman and her family will be anxious and concerned about her condition and prognosis. The nurse offers explanations about care and provides emotional support to the woman and her family through this critical time.

3. The nurse sees a woman for the first time when she is 30 weeks pregnant. The woman has smoked throughout the pregnancy, and fundal height measurements now are suggestive of growth restriction in the fetus. In addition to ultrasound to measure fetal size, what would be another tool useful in confirming the diagnosis? A. Doppler blood flow analysis B. Contraction stress test (CST) C. Amniocentesis D. Daily fetal movement counts

A. Doppler blood flow analysis Doppler blood flow analysis allows the examiner to study the blood flow noninvasively in the fetus and the placenta. It is a helpful tool in the management of high risk pregnancies because of intrauterine growth restriction (IUGR), diabetes mellitus, multiple fetuses, or preterm labor. Because of the potential risk of inducing labor and causing fetal distress, a CST is not performed on a woman whose fetus is preterm. Indications for an amniocentesis include diagnosis of genetic disorders or congenital anomalies, assessment of pulmonary maturity, and the diagnosis of fetal hemolytic disease, not IUGR. Fetal kick count monitoring is performed to monitor the fetus in pregnancies complicated by conditions that may affect fetal oxygenation. Although this may be a useful tool at some point later in this woman's pregnancy, it is not used to diagnose IUGR.

5. A woman with severe preeclampsia has been receiving magnesium sulfate by intravenous infusion for 8 hours. The nurse assesses the woman and documents the following findings: temperature of 37.1° C, pulse rate of 96 beats/min, respiratory rate of 24 breaths/min, blood pressure (BP) of 155/112 mm Hg, 3+ deep tendon reflexes, and no ankle clonus. The nurse calls the physician, anticipating an order for: A. Hydralazine. B. Magnesium sulfate bolus. C. Diazepam. D. Calcium gluconate.

A. Hydralazine. Hydralazine is an antihypertensive commonly used to treat hypertension in severe preeclampsia. Typically it is administered for a systolic BP over 160 mm Hg or a diastolic BP over 110 mm Hg. An additional bolus of magnesium sulfate may be ordered for increasing signs of central nervous system irritability related to severe preeclampsia (e.g., clonus) or if eclampsia develops. Diazepam sometimes is used to stop or shorten eclamptic seizures. Calcium gluconate is used as the antidote for magnesium sulfate toxicity. The woman is not currently displaying any signs or symptoms of magnesium toxicity.

4. With regard to injuries to the infant's plexus during labor and birth, nurses should be aware that: A. If the nerves are stretched with no avulsion, they should recover completely in 3 to 6 months. B. Erb palsy is damage to the lower plexus. C. Parents of children with brachial palsy are taught to pick up the child from under the axillae. D. Breastfeeding is not recommended for infants with facial nerve paralysis until the condition resolves.

A. If the nerves are stretched with no avulsion, they should recover completely in 3 to 6 months. It is true that if the nerves are stretched with no avulsion, they should recover completely in 3 to 6 months. However, if the ganglia are disconnected completely from the spinal cord, the damage is permanent. Erb palsy is damage to the upper plexus and is less serious than brachial palsy. Parents of children with brachial palsy are taught to avoid picking up the child under the axillae or by pulling on the arms. Breastfeeding is not contraindicated, but both the mother and infant will need help from the nurse at the start.

10. Which infant would be more likely to have Rh incompatibility? A. Infant of an Rh-negative mother and a father who is Rh positive and homozygous for the Rh factor B. Infant who is Rh negative and whose mother is Rh negative C. Infant of an Rh-negative mother and a father who is Rh positive and heterozygous for the Rh factor D. Infant who is Rh positive and whose mother is Rh positive

A. Infant of an Rh-negative mother and a father who is Rh positive and homozygous for the Rh factor If the mother is Rh negative and the father is Rh positive and homozygous for the Rh factor, all the offspring will be Rh positive. Only Rh-positive offspring of an Rh-negative mother are at risk for Rh incompatibility. Only the Rh-positive offspring of an Rh-negative mother are at risk. If the mother is Rh negative and the father is Rh positive and heterozygous for the factor, there is a 50% chance that each infant born of the union will be Rh positive and a 50% chance that each will be born Rh negative. Only the Rh-positive offspring of an Rh-negative mother are at risk.

5. What PPH conditions are considered medical emergencies that require immediate treatment? A. Inversion of the uterus and hypovolemic shock B. Hypotonic uterus and coagulopathies C. Subinvolution of the uterus and idiopathic thrombocytopenic purpura (ITP) D. Uterine atony and disseminated intravascular coagulation (DIC)

A. Inversion of the uterus and hypovolemic shock Inversion of the uterus and hypovolemic shock are considered medical emergencies. Although a hypotonic uterus and coagulopathies are serious conditions, they do not necessarily constitute a medical emergency that requires immediate treatment. Although subinvolution and ITP are serious conditions, they do not necessarily constitute a medical emergency that requires immediate treatment. Although uterine atony and DIC are serious conditions, they do not necessarily constitute a medical emergency that requires immediate treatment.

9. With regard to anemia, nurses should be aware that: A. It is the most common medical disorder of pregnancy. B. It can trigger reflex brachycardia. C. The most common form of anemia is caused by folate deficiency. D. Thalassemia is a European version of sickle cell anemia.

A. It is the most common medical disorder of pregnancy. Anemia is the most common medical disorder of pregnancy. Combined with any other complication, anemia can result in congestive heart failure. Reflex bradycardia is a slowing of the heart in response to the blood flow that increases immediately after birth. The most common form of anemia is iron deficiency anemia. Both thalassemia and sickle cell hemoglobinopathy are hereditary but not directly related or confined to geographic areas.

-infants receiving phototherapy (for high bilirubin) need eye shields to protect eyes

During phototherapy the unclothed infant is placed under a bank of lights approximately 45 to 50 cm from the light source. The distance may vary based on unit protocol and type of light used. A Plexiglas panel or shield should always be placed between the lights and the infant when conventional lighting is used. The most effective therapy is achieved with lights at 400 to 500 manometers, and a blue-green light spectrum is the most efficient (Steffensrud, 2004). The lamp's energy output should be monitored routinely with a photometer during treatment to ensure efficacy of therapy. Phototherapy is carried out until the infant's serum bilirubin level decreases to within an acceptable range. The decision to discontinue therapy is based on the observation of a definite downward trend in the bilirubin values. Several precautions must be taken while the infant is undergoing phototherapy. The infant's eyes must be protected by an opaque mask to prevent overexposure to the light. The eye shield should cover the eyes completely but not occlude the nares. Before the mask is applied the infant's eyes should be closed gently to prevent excoriation of the corneas. The mask should be removed periodically and during infant feedings so that the eyes can be checked and cleansed with water and the parents can have visual contact with the infant. To promote optimal skin exposure during phototherapy a "string bikini" made from a disposable facemask is often used instead of a diaper, which allows optimal skin exposure and provides protection for the genitals and the bedding. Before use the metal strip must be removed from the mask to prevent burning the infant. Lotions and ointments should not be used during phototherapy because they absorb heat and can cause burns. Phototherapy can cause changes in the infant's temperature, depending partially on the bed used: bassinet, incubator, or radiant warmer. The infant's temperature should be closely monitored. Phototherapy lights can increase the rate of insensible water loss, which contributes to fluid loss and dehydration. Therefore the infant must be adequately hydrated. Hydration maintenance in the healthy newborn is accomplished with human milk or infant formula; administering glucose water or plain water has no advantage or benefit because these liquids do not promote excretion of bilirubin in the stools and may actually perpetuate enterohepatic circulation, thus delaying bilirubin excretion. It is important to closely monitor urinary output while the infant is receiving phototherapy. Urine output may be decreased or unaltered; the urine may have a dark-gold or brown appearance. The number and consistency of stools are monitored. Bilirubin breakdown increases gastric motility, which results in loose stools that can cause skin excoriation and breakdown. The infant's buttocks must be cleaned after each stool to help maintain skin integrity. A fine maculopapular rash may appear during phototherapy, but this condition is transient. Because visualization of the infant's skin color is difficult with blue light, appropriate cardiorespiratory monitoring should be implemented based on the infant's overall condition.

1. The nurse is caring for a woman with mitral stenosis who is in the active stage of labor. Which action should the nurse take to promote cardiac function? A. Maintain the woman in a side-lying position with the head and shoulders elevated to facilitate hemodynamics B. Prepare the woman for delivery by cesarean section because this is the recommended delivery method to sustain hemodynamics C. Encourage the woman to avoid the use of narcotics or epidural regional analgesia because this alters cardiac function D. Promote the use of the Valsalva maneuver during pushing in the second stage to improve diastolic ventricular filling

A. Maintain the woman in a side-lying position with the head and shoulders elevated to facilitate hemodynamics The side-lying position with the head and shoulders elevated helps to facilitate hemodynamics during labor. A vaginal delivery is the preferred method of delivery for a woman with cardiac disease because it sustains hemodynamics better than a cesarean section. The use of supportive care, medication, and narcotics or epidural regional analgesia is not contraindicated for a woman with heart disease. The use of the Valsalva maneuver during pushing in the second stage should be avoided because it reduces diastolic ventricular filling and obstructs left ventricular outflow.

7. A woman with asthma is experiencing a postpartum hemorrhage. Which drug would NOT be used to treat her bleeding because it may exacerbate her asthma? A. Pitocin B. Nonsteroidal antiinflammatory drugs (NSAIDs) C. Hemabate D. Fentanyl

A. Pitocin Pitocin would be the drug of choice to treat this woman's bleeding because it would not exacerbate her asthma. NSAIDs are not used to treat bleeding. Prostaglandin derivatives should not be used to treat women with asthma, because they may exacerbate symptoms. Oxytocin is the recommended medication for uterine bleeding. Fentanyl is used to treat pain, not bleeding.

3. Because pregnant women may need surgery during pregnancy, nurses should be aware that: A. The diagnosis of appendicitis may be difficult to make, because the normal signs and symptoms mimic some normal changes in pregnancy. B. Rupture of the appendix is less likely in pregnant women because of the close monitoring. C. Surgery for intestinal obstructions should be delayed as long as possible because it usually affects the pregnancy. D. When pregnancy takes over, a woman is less likely to have ovarian problems that require invasive responses.

A. The diagnosis of appendicitis may be difficult to make, because the normal signs and symptoms mimic some normal changes in pregnancy. Both appendicitis and pregnancy are linked with nausea, vomiting, and increased white blood cell count. Rupture of the appendix is two to three times more likely in pregnant women. Surgery to remove obstructions should be done right away. It usually does not affect the pregnancy. Pregnancy predisposes a woman to ovarian problems.

hyperemesis gravidarum

Abnormal condition of pregnancy characterized by protracted vomiting, weight loss, and fluid and electrolyte imbalance

developmental dysplasia of the hip (DDH)

Abnormal development of the hip joint, resulting in instability of the hip causing one or both of the femoral heads to be displaced from the acetabulum (hip socket)

dysfunctional labor

Abnormal uterine contractions that prevent normal progress of cervical dilation, effacement, or descent

ketoacidosis

Accumulation of ketone bodies in the blood as a consequence of hyperglycemia; leads to metabolic acidosis

therapeutic rest

Administration of analgesics and implementation of comfort or relaxation measures to decrease pain and induce rest for management of hypertonic uterine dysfunction

-problems assoc with diabetes in pregnancy- congenital anomalies, spont. Ab if uncontrolled glucose levels in early pregnancy, and macrosomia in later preg.

All infants born to mothers with diabetes (IDMs) are at some risk for complications. The degree of risk is influenced by the severity and duration of maternal disease. Pathophysiology The mechanisms responsible for the problems seen in IDMs are not fully understood. Congenital anomalies are believed to be caused by fluctuations in blood glucose levels and episodes of ketoacidosis in early pregnancy. Later in pregnancy, when the mother's pancreas cannot release sufficient insulin to meet increased demands, maternal hyperglycemia results. The high levels of glucose cross the placenta and stimulate the fetal pancreas to release additional insulin. The combination of the increased supply of maternal glucose and other nutrients, the inability of maternal insulin to cross the placenta, and increased fetal insulin results in excessive fetal growth called macrosomia (see the discussion that follows). Hyperinsulinemia accounts for many of the problems the fetus or infant develops. In addition to fluctuating glucose levels, maternal vascular involvement or superimposed maternal infection adversely affects the fetus. Normally, maternal blood has a more alkaline pH than carbon dioxide-rich fetal blood. This phenomenon encourages the exchange of oxygen and carbon dioxide across the placental membrane. When the maternal blood is more acidotic than the fetal blood, such as during ketoacidosis, little carbon dioxide or oxygen exchange occurs at the level of the placenta. The mortality for the unborn infant resulting from an episode of maternal ketoacidosis may be as high as 50% or more (Kalhan & Parimi, 2006). The single most important factor influencing fetal well-being is the euglycemic status of the mother. Indications are that some neonatal conditions (macrosomia, hypoglycemia, polyhydramnios, preterm birth, and perhaps fetal lung immaturity) may be eliminated, or the incidence decreased, by maintaining tight control of maternal glucose levels within narrow limits. Tight glucose control is defined as the maintenance of maternal blood glucose levels between 100 and 120 mg/dl. Problems seen in infants of diabetic mothers (IDMs) include the following: • Congenital anomalies occur in 7% to 10% of IDMs; the incidence is greatest in SGA infants. The most commonly occurring anomalies involve the cardiac, musculoskeletal, and central nervous systems. CNS anomalies include anencephaly, encephalocele, meningomyelocele, and hydrocephalus (see Table 24-10). The musculoskeletal system may be affected by caudal regression syndrome (i.e., sacral agenesis, with weakness or deformities of the lower extremities, malformation and fixation of the hip joints, shortening or deformity of the femurs). • Macrosomia: At birth the typical LGA infant has a round, cherubic ("tomato" or cushingoid) face, chubby body, and a plethoric or flushed complexion (Fig. 24-9). The infant has enlarged internal organs (i.e., hepatosplenomegaly, splanchnomegaly, cardiomegaly) and increased body fat, especially around the shoulders. The placenta and umbilical cord are larger than average. The brain is the only organ that is not enlarged. IDMs may be LGA but physiologically immature. The macrosomic infant is at risk for RDS, hypoglycemia, hypocalcemia and hypomagnesemia, cardiomyopathy, polycythemia, and hyperbilirubinemia. The excessive shoulder size in these infants often leads to dystocia. Macrosomic infants born vaginally or by cesarean birth after a trial of labor may incur birth trauma. • Birth injury (resulting from macrosomia or method of birth) and perinatal asphyxia occur in 20% of infants of gestational diabetic mothers and 35% of IDMs. Examples of birth trauma include cephalhematoma; paralysis of the facial nerve (seventh cranial nerve) (Table 24-10); fracture of the clavicle or humerus (see Table 24-10); brachial plexus paralysis, usually Erb-Duchenne (right upper arm) palsy (see Table 24-10); and phrenic nerve paralysis, invariably associated with diaphragmatic paralysis. • RDS: IDMs are four to six times more likely than normal infants to develop RDS. In the fetus exposed to high levels of maternal glucose, synthesis of surfactant may be delayed because of the high fetal serum level of insulin. In the IDM the presence of phosphatidylglycerol in the amniotic fluid is the best predictor of normal neonatal respiratory function. • Hypoglycemia affects many IDMs. After constant exposure to high circulating levels of glucose, hyperplasia of the fetal pancreas occurs, resulting in hyperinsulinemia. With clamping of the umbilical cord, the fetal glucose supply is cut off. The neonate's blood glucose level falls rapidly because of fetal hyperinsulinism. • Cardiomyopathy: Two types of cardiomyopathy can occur. Hypertrophic cardiomyopathy (HCM) is characterized by a hypercontractile and thickened myocardium. The ventricular walls are thickened, as is the septum, which in severe cases results in outflow tract obstructions. The mitral valve is poorly functioning. In nonhypertrophic cardiomyopathy (non-HCM) the myocardium is poorly contractile and overstretched. The ventricles are increased in size, but outflow is not obstructed. Most infants are asymptomatic, but severe outflow obstruction may cause left ventricular heart failure. HCM may be treated with a beta-adrenergic blocker (e.g., propranolol to decrease contractility and heart rate). A cardiotonic agent is used to treat non-HCM (e.g., digoxin to increase contractility and decrease heart rate). The abnormality usually resolves in 3 to 12 months. • Hyperbilirubinemia and polycythemia: IDMs are at increased risk of developing hyperbilirubinemia (see Chapter 17). Many IDMs are also polycythemic. Polycythemia increases blood viscosity, thereby impairing circulation. In addition, this increased number of RBCs to be hemolyzed increases the potential bilirubin load that the neonate must clear. Bruising associated with birth of a macrosomic infant will contribute further to high bilirubin levels. Nursing care Ideally, planning for the IDM begins during the antenatal period. Pediatric NICU staff members are present at the birth. Implementation of care depends on the neonate's particular problems. If the maternal blood glucose level was well controlled throughout the pregnancy, the infant may require only monitoring. Because euglycemia is not always possible, the nurse must promptly recognize and treat any consequences of maternal diabetes that arise.

thrush

Fungal infection of the mouth or throat characterized by the formation of white patches on a red, moist, inflamed mucous membrane, caused by Candida albicans

-amniocentesis- indications, risks, complications

Amniocentesis is performed to obtain amniotic fluid, which contains fetal cells. Under direct ultrasonographic visualization, a needle is inserted transabdominally into the uterus, amniotic fluid is withdrawn into a syringe, and the various assessments are performed. Amniocentesis is possible after week 14 of pregnancy, when the uterus becomes an abdominal organ, and sufficient amniotic fluid is available for testing. Indications for the procedure include prenatal diagnosis of genetic disorders or congenital anomalies (NTDs in particular), assessment of pulmonary maturity, and diagnosis of fetal hemolytic disease. Complications in the mother and fetus occur in less than 1% of the cases and include the following: • Maternal: hemorrhage, fetomaternal hemorrhage with possible maternal Rh isoimmunization, infection, labor, abruptio placentae, inadvertent damage to the intestines or bladder, and amniotic fluid embolism • Fetal: death, hemorrhage, infection (amnionitis), direct injury from the needle, miscarriage or preterm labor, and leakage of amniotic fluid NURSING ALERT Because of the possibility of fetomaternal hemorrhage, administering RhoD immune globulin to the woman who is Rh negative is standard practice after an amniocentesis. Many of the complications have been minimized or eliminated by using ultrasonography to direct the procedure. Indications for Use Genetic concerns Prenatal assessment of genetic disorders is indicated in women older than 35 years (Box 19-5), with a previous child with a chromosomal abnormality, or with a family history of chromosomal anomalies. Inherited errors of metabolism (such as Tay-Sachs disease, hemophilia, and thalassemia) and other disorders for which marker genes are known also may be detected. Fetal cells are cultured for karyotyping of chromosomes (see Chapter 5). Karyotyping also permits determination of fetal sex, which is important if an X-linked disorder (occurring almost always in a male fetus) is suspected. Biochemical analysis of enzymes in amniotic fluid can detect inborn errors of metabolism. For example, AFP levels in amniotic fluid are assessed as a follow-up for elevated levels in maternal serum. High AFP levels in amniotic fluid help confirm the diagnosis of an NTD such as spina bifida or anencephaly or an abdominal wall defect such as omphalocele. The elevation results from the increased leakage of cerebrospinal fluid into the amniotic fluid through the closure defect. AFP levels may also be elevated in a normal multifetal pregnancy and with intestinal atresia, presumably caused by lack of fetal swallowing. BOX 19-5 Elimination of Maternal Age as an Indication for Invasive Prenatal Diagnosis Maternal age of 35 years and older has been a standard indication for invasive prenatal testing since 1979. In January 2007, however, the American College of Obstetricians and Gynecologists (ACOG) published new guidelines stating that no specific age should be used as a threshold for invasive or noninvasive screening. Furthermore, all women, regardless of age, should have the option of invasive testing without first having screening (ACOG, 2007). A concurrent test that finds the presence of acetylcholinesterase almost always indicates a fetal defect (Wapner, Jenkins, & Khalek, 2009). In such instances, follow-up ultrasound examination is recommended. Fetal maturity Accurate assessment of fetal maturity is possible through examination of amniotic fluid or its exfoliated cellular contents. The laboratory tests described are determinants of term pregnancy and fetal maturity (see Table 19-4). A quick means of determining an approximate L/S ratio is the shake test (foam test), or bubble stability test. Serial dilutions of fresh amniotic fluid are mixed with ethanol and shaken. After 15 minutes the amount of bubbles present at different dilutions indicates the presence of surfactant. Currently, the fetal lung maturity (FLM) assay is often used to determine fetal lung maturity because it is simple to perform. FLM test results are similar to those of the L/S ratio in terms of predicting pulmonary maturity (Mercer, 2009). Fetal hemolytic disease Another indication for amniocentesis is the identification and follow-up of fetal hemolytic disease in cases of isoimmunization. The procedure is usually not performed until the mother's antibody titer reaches 1:8 and is increasing. Although PUBS is still the procedure of choice to treat fetal hemolytic disease, it is now used less frequently for evaluating this condition. Doppler velocimetry of the fetal middle cerebral artery is currently used to predict anemia associated with fetal hemolytic disease accurately and noninvasively (Tucker et al., 2009).

hydramnios (polyhydramnios)

Amniotic fluid in excess of 2000 ml

late preterm (near term) infant

An infant born between 34 0/7 and 36 6/7 weeks of gestation, regardless of birth weight

oxytocin

Hormone produced by the posterior pituitary gland that stimulates uterine contractions and the release of milk in the mammary glands (let-down reflex); synthetic oxytocin is a medication that mimics the uterine stimulating action of oxytocin

-betamethasone- uses, indications

Antenatal glucocorticoids given as intramuscular injections to the mother to accelerate fetal lung maturity are now considered one of the most effective and cost-efficient interventions for preventing morbidity and mortality associated with preterm labor. Antenatal glucocorticoids have been shown to reduce significantly the incidence of respiratory distress syndrome, intraventricular hemorrhage, necrotizing enterocolitis, and death in neonates without increasing the risk of infection in either mothers or newborns (Mercer, 2009a). The National Institutes of Health consensus panel recommended that all women at 24 to 34 weeks of gestation be given a single course of antenatal glucocorticoids when preterm birth is threatened, unless evidence indicates that corticosteroids will have an adverse effect on the mother or birth is imminent. In general, women who are candidates for tocolytic therapy are also candidates for antenatal glucocorticoids (Mercer, 2009a). The regimen for the administration of antenatal glucocorticoids is given in the Medication Guide: Antenatal Glucocorticoid Therapy with Betamethasone, Dexamethasone. Medication Guide: Antenatal Glucocorticoid Therapy with Betamethasone, Dexamethasone ACTION • Stimulates fetal lung maturation by promoting release of enzymes that induce production or release of lung surfactant. NOTE: The U.S. Food and Drug Administration has not approved these medications for this use (i.e., this is an unlabeled use for obstetrics). INDICATION • To accelerate lung maturity in fetuses between 24 and 34 weeks of gestation DOSAGE AND ROUTE • Betamethasone: 12 mg intramuscularly (IM) for two doses 24 hr apart • Dexamethasone: 6 mg IM for four doses 12 hr apart ADVERSE REACTIONS • Pulmonary edema (if given with beta-adrenergic medications) • May worsen maternal condition (diabetes, hypertension). NURSING CONSIDERATIONS • Give deep intramuscular injection in gluteal muscle. • Teach signs of pulmonary edema. • Assess blood glucose levels and lung sounds.

acoustic stimulation test

Antepartum test to elicit fetal heart rate response to sound; performed by applying sound source (laryngeal stimulator) to the maternal abdomen over the fetal head

5. In appraising the growth and development potential of a preterm infant, nurses should: A. Tell parents their child won't catch up until about age 10 (girls) to 12 (boys). B. Correct for milestones such as motor competencies and vocalizations until the child is approximately 3 years of age. C. Know that the greatest catch-up period is between 9 and 15 months postconceptual age. D. Know that the length and breadth of the trunk is the first part of the infant to experience catch-up growth.

B. Correct for milestones such as motor competencies and vocalizations until the child is approximately 3 years of age. The infant, girl or boy, experiences catch-up body growth during the first 2 to 3 years of life. Corrections are made with a formula that adds gestational age and postnatal age. Maximum catch-up growth occurs between 36 and 40 weeks postconceptual age. The head is the first part of the infant to experience catch-up growth.

4. The nurse is caring for a woman whose labor is being augmented with oxytocin. The nurse recognizes that the oxytocin should be discontinued immediately if there is evidence of: A. Uterine contractions occurring every 8 to 10 minutes. B. A fetal heart rate (FHR) of 180 with absence of variability. C. The woman needing to void. D. Rupture of the woman's amniotic membranes.

B. A fetal heart rate (FHR) of 180 with absence of variability. The oxytocin should be discontinued if uterine hyperstimulation occurs. Uterine contractions that are occurring every 8 to 10 minutes do not qualify as hyperstimulation. An FHR of 180 is nonreassuring. The oxytocin should be discontinued immediately, and the physician should be notified. The woman needing to void is not an indication to discontinue the oxytocin induction immediately or to call the physician. Unless a change occurs in the FHR pattern that is nonreassuring or the woman experiences uterine hyperstimulation, the oxytocin does not need to be discontinued. The physician should be notified that the woman's membranes have ruptured.

1. A woman with severe preeclampsia is receiving a magnesium sulfate infusion. The nurse becomes concerned after assessment when the woman exhibits: A. A sleepy, sedated affect. B. A respiratory rate of 10 breaths/min. C. Deep tendon reflexes of 2. D. Absent ankle clonus.

B. A respiratory rate of 10 breaths/min. Because magnesium sulfate is a central nervous system depressant, the woman will most likely become sedated when the infusion is initiated. A respiratory rate of 10 breaths/min indicates that the woman is experiencing respiratory depression from magnesium toxicity. Deep tendon reflexes of 2 is a normal finding. Absent ankle clonus is a normal finding.

3. Which woman is at greatest risk for early postpartum hemorrhage (PPH)? A. A primiparous woman (G 2 P 1 0 0 1) being prepared for an emergency cesarean birth for fetal distress B. A woman with severe preeclampsia on magnesium sulfate whose labor is being induced C. A multiparous woman (G 3 P 2 0 0 2) with an 8-hour labor D. A primigravida in spontaneous labor with preterm twins

B. A woman with severe preeclampsia on magnesium sulfate whose labor is being induced Although many causes and risk factors are associated with PPH, a primiparous woman (G 2 P 1 0 0 1) being prepared for an emergency cesarean birth for fetal distress does not pose risk factors or causes of early PPH. Magnesium sulfate administration during labor poses a risk for PPH. Magnesium acts as a smooth muscle relaxant, thereby contributing to uterine relaxation and atony. Although many causes and risk factors are associated with PPH, a multiparous woman (G 3 P 2 0 0 2) with an 8-hour labor does not pose risk factors or causes of early PPH. Although many causes and risk factors are associated with PPH, a primigravida in spontaneous labor with preterm twins does not pose risk factors or causes of early PPH.

3. A newborn was admitted to the neonatal intensive care unit after being delivered at 29 weeks of gestation to a 28-year-old multiparous, married, Caucasian female whose pregnancy was uncomplicated until premature rupture of membranes and preterm birth. The newborn's parents arrive for their first visit after the birth. The parents walk toward the bedside but remain approximately 5 feet away from the bed. The nurse's most appropriate action would be to: A. Wait quietly at the newborn's bedside until the parents come closer. B. Go to the parents, introduce himself or herself, and gently encourage them to come meet their infant; explain the equipment first, and then focus on the newborn. C. Leave the parents at the bedside while they are visiting so they can have some privacy. D. Tell the parents only about the newborn's physical condition and caution them to avoid touching their baby.

B. Go to the parents, introduce himself or herself, and gently encourage them to come meet their infant; explain the equipment first, and then focus on the newborn. Parents often need encouragement and recognition from the nurse to acknowledge the reality of the infant's condition. The nurse is instrumental in the initial interactions with the infant. The nurse can help the parents see the infant rather than focus on the equipment. The importance and purpose of the apparatus that surrounds their infant also should be explained to them. Parents need to see and touch their infant as soon as possible to acknowledge the reality of the birth and the infant's appearance and condition. Encouragement from the nurse is instrumental in this process. Telling the parents not to touch the infant is inappropriate.

5. While providing care in an obstetric setting, the nurse should understand that postpartum care of the woman with cardiac disease: A. Is the same as that for any pregnant woman. B. Includes rest, stool softeners, and monitoring of the effect of activity. C. Includes ambulating frequently, alternating with active range of motion. D. Includes limiting visits with the infant to once per day.

B. Includes rest, stool softeners, and monitoring of the effect of activity. Care of the woman with cardiac disease in the postpartum period is tailored to the woman's functional capacity. Bed rest may be ordered, with or without bathroom privileges. Bowel movements without stress or strain for the woman are promoted with stool softeners, diet, and fluid. The woman will be on bed rest to conserve energy and reduce the strain on the heart. Although the woman may need help caring for the infant, breastfeeding and infant visits are not contraindicated.

2. Which choice gives indicators for performing a contraction stress test? A. Increased fetal movement and small for gestational age B. Maternal diabetes mellitus and postmaturity C. Adolescent pregnancy and poor prenatal care D. History of preterm labor and intrauterine growth restriction

B. Maternal diabetes mellitus and postmaturity Decreased fetal movement is an indicator for performing a contraction stress test; the size (small for gestational age) is not an indicator. Maternal diabetes mellitus and postmaturity are two indications for performing a contraction stress test. Although adolescent pregnancy and poor prenatal care are risk factors of poor fetal outcomes, they are not indicators for performing a contraction stress test. Intrauterine growth restriction is an indicator; but history of a previous stillbirth, not preterm labor, would be the other indicator.

9. With regard to the classification of neonatal bacterial infection, nurses should be aware that: A. Congenital infection progresses slower than nosocomial infection. B. Nosocomial infection can be prevented by effective handwashing; early-onset infections cannot. C. Infections occur with about the same frequency in boy and girl infants, although female mortality is higher. D. The clinical sign of a rapid, high fever makes infection easier to diagnose.

B. Nosocomial infection can be prevented by effective handwashing; early-onset infections cannot. Congenital (early-onset) infections progress more rapidly than nosocomial (late-onset) infections. Handwashing is an effective preventive measure for late-onset (nosocomial) infections because these infections come from the environment around the infant. Early-onset, or congenital, infections are caused by the normal flora at the maternal vaginal tract. Congenital (early-onset) infections progress more rapidly than nosocomial (late-onset) infections. Infection occurs about twice as often in boys and results in higher mortality. Congenital (early-onset) infections progress more rapidly than nosocomial (late-onset) infections. Clinical signs of neonatal infection are nonspecific and similar to noninfectious problems, making diagnosis difficult. Congenital (early-onset) infections progress more rapidly than nosocomial (late-onset) infections.

4. A new mother with which of these thyroid disorders would be strongly discouraged from breastfeeding? A. Hyperthyroidism B. Phenylketonuria (PKU) C. Hypothyroidism D. Thyroid storm

B. Phenylketonuria (PKU) A woman with hyperthyroidism would have no particular reason not to breastfeed. PKU is a cause of mental retardation in infants; mothers with PKU pass on phenylalanine through breast milk. A woman with hypothyroidism would have no particular reason not to breastfeed. A thyroid storm is a complication of hyperthyroidism.

12. The nurse is caring for an infant born at 28 weeks of gestation. Which complication could the nurse expect to observe during the course of the neonate's hospitalization? Select all that apply. A. Polycythemia B. Respiratory distress syndrome C. Meconium aspiration syndrome D. Periventricular hemorrhage E. Persistent pulmonary hypertension F. Patent ductus arteriosus

B. Respiratory distress syndrome D. Periventricular hemorrhage F. Patent ductus arteriosus Polycythemia, meconium aspiration syndrome, and persistent pulmonary hypertension are complications of postmaturity. Respiratory distress syndrome, periventricular hemorrhage, and a patent ductus arteriosus are common complications with preterm infants

3. Diabetes in pregnancy puts the fetus at risk in several ways. Nurses should be aware that: A. With good control of maternal glucose levels, sudden and unexplained stillbirth is no longer a major concern. B. The most important cause of perinatal loss in diabetic pregnancy is congenital malformations. C. Infants of mothers with diabetes have the same risks for respiratory distress syndrome because of the careful monitoring. D. At birth the neonate of a diabetic mother is no longer in any risk.

B. The most important cause of perinatal loss in diabetic pregnancy is congenital malformations. Even with good control, sudden and unexplained stillbirth remains a major concern. Congenital malformations account for 30% to 50% of perinatal deaths. Infants of diabetic mothers are at increased risk for respiratory distress syndrome. The transition to extrauterine life often is marked by hypoglycemia and other metabolic abnormalities.

anaphylactoid syndrome of pregnancy (ASP)

Rare complication of pregnancy characterized by the sudden, acute onset of hypoxia, hypotension, or cardiac arrest and coagulopathy that can occur either during labor or during birth or immediately after birth; also known as amniotic fluid embolism

2. With regard to dysfunctional labor, nurses should be aware that: A. Women who are underweight are more at risk. B. Women experiencing precipitous labor are about the only "dysfunctionals" not to be exhausted. C. Hypertonic uterine dysfunction is more common than hypotonic dysfunction. D. Abnormal labor patterns are most common in older women.

B. Women experiencing precipitous labor are about the only "dysfunctionals" not to be exhausted. Short women more than 30 pounds overweight are more at risk for dysfunctional labor. Precipitous labor lasts less than 3 hours. Hypotonic uterine dysfunction, in which the contractions become weaker, is more common. Abnormal labor patterns are more common in women under 20 years of age.

-Fetal alarm signal

BIOPHYSICAL ASSESSMENT ▪ Daily Fetal Movement Count Assessment of fetal activity by the mother is a simple yet valuable method for monitoring the condition of the fetus. The daily fetal movement count (DFMC) (also called kick count) can be assessed at home and is noninvasive, inexpensive, simple to understand, and usually does not interfere with a daily routine. The DFMC is frequently used to monitor the fetus in pregnancies complicated by conditions that may affect fetal oxygenation (see Box 19-2). The presence of movements is generally a reassuring sign of fetal health. Several different protocols are used for counting. One recommendation is to count once a day for 60 minutes. Another common recommendation is that mothers count fetal activity two or three times daily for 60 minutes each time. Except for establishing a very low number of daily fetal movements or a trend toward decreased motion, the clinical value of the absolute number of fetal movements has not been established, other than in the situation in which fetal movements cease entirely for 12 hours (the so-called fetal alarm signal). A count of fewer than three fetal movements within 1 hour warrants further evaluation by a nonstress test (NST) or contraction stress test (CST) (oxytocin challenge test [OCT]), biophysical profile (BPP), or a combination of these (see later discussion). Women should be taught the significance of the presence or absence of fetal movements (or both), the procedure for counting that is to be used, how to record findings on a daily fetal movement record, and when to notify the health care provider. NURSING ALERT In assessing fetal movements, it is important to remember that they are usually not present during the fetal sleep cycle and that they may be temporarily reduced if the woman is taking depressant medications, drinking alcohol, or smoking a cigarette. They do not decrease as the woman nears term. Obesity decreases the ability of the mother to perceive fetal movement.

-placenta previa

Because of advances in ultrasonography, especially transvaginal ultrasound, and an increased understanding of the changing relationship between the placenta and the internal cervical os as pregnancy progresses, definitions and classifications of placental previa have recently changed. In placenta previa, the placenta is implanted in the lower uterine segment such that it completely or partially covers the cervix or is close enough to the cervix to cause bleeding when the cervix dilates or the lower uterine segment effaces (Fig. 21-11) (Hull & Resnik, 2009). When transvaginal ultrasound is used, the placenta is classified as a complete placenta previa if it totally covers the internal cervical os. In a marginal placenta previa the edge of the placenta is seen on transvaginal ultrasound to be 2.5 cm or closer to the internal cervical os. When the exact relationship of the placenta to the internal cervical os has not been determined or in the case of apparent placenta previa in the second trimester, the term low-lying placenta is used (Hull & Resnik). Incidence and etiology Placenta previa affects approximately 1 in 200 pregnancies at term. Some evidence suggests that the incidence of placenta previa is increasing, perhaps as a result of the increasing cesarean birth rate. In addition to a history of previous cesarean birth, other risk factors for placenta previa include advanced maternal age (>35-40 years of age), multiparity, history of prior suction curettage, and smoking (Hull & Resnik, 2009). Cigarette smoking leads to a decrease in uteroplacental oxygenation and thus a need for increased placental surface area. Placenta previa is more likely to occur in women with multiple gestations because of the larger placental area associated with these pregnancies. Women who had placenta previa in a previous pregnancy are more likely than others to develop the problem in a subsequent pregnancy, perhaps as a result of a genetic predisposition. Previous cesarean birth and curettage in the past for miscarriage or induced abortion are risk factors for placenta previa because both result in endometrial damage and uterine scarring (Francois & Foley, 2007; Hull & Resnik). Clinical manifestations Placenta previa is typically characterized by painless, bright-red vaginal bleeding during the second or third trimester. In the past, placenta previa was usually diagnosed after an episode of bleeding. Currently, however, most cases are diagnosed by ultrasound before significant vaginal bleeding occurs (Francois & Foley, 2007). This bleeding is associated with the disruption of placental blood vessels that occurs with stretching and thinning of the lower uterine segment (Francois & Foley). The initial bleeding is usually a small amount and stops as clots form; however, it can recur at any time (Gilbert, 2007). Vital signs may be normal, even with heavy blood loss, because a pregnant woman can lose up to 40% of her blood volume without showing signs of shock. Clinical presentation and decreasing urinary output may be better indicators of acute blood loss than vital signs alone. The FHR is reassuring unless a major detachment of the placenta occurs. Abdominal examination usually reveals a soft, relaxed, nontender uterus with normal tone. The presenting part of the fetus usually remains high because the placenta occupies the lower uterine segment. Thus the fundal height is often greater than expected for gestational age. Because of the abnormally located placenta, fetal malpresentation (breech and transverse or oblique lie) is common. Maternal and fetal outcomes Complications associated with placenta previa include PROM, preterm labor and birth, surgery-related trauma to structures adjacent to the uterus, anesthesia complications, blood transfusion reactions, overinfusion of fluids, abnormal placental attachments, postpartum hemorrhage, anemia, thrombophlebitis, and infection (Cunningham et al., 2005). The greatest risk of fetal death is caused by preterm birth. Other fetal risks include malpresentation and fetal anemia (Gilbert, 2007). Infants who are small for gestational age or have IUGR have been associated with placenta previa. This association may be related to poor placental exchange or hypovolemia resulting from maternal blood loss and maternal anemia (Gilbert). Diagnosis All women with painless vaginal bleeding after 20 weeks of gestation should be assumed to have a placenta previa until proven otherwise. A transabdominal ultrasound examination should be performed initially followed by a transvaginal scan, unless the transabdominal ultrasound clearly shows that the placenta is not located in the lower uterine segment. A transvaginal ultrasound is better than a transabdominal scan for accurately determining placental location (Hull & Resnik, 2009). If ultrasonographic scanning reveals a normally implanted placenta, a speculum examination may be performed to rule out local causes of bleeding (e.g., cervicitis, polyps, carcinoma of the cervix), and a coagulation profile is obtained to rule out other causes of bleeding. Management Once placenta previa has been diagnosed, a management plan is developed. The woman will be managed either expectantly or actively, depending on the gestational age, amount of bleeding, and fetal condition (see Nursing Process box: Placenta Previa.) Expectant management Expectant management (observation and bed rest) is implemented if the fetus is at less than 36 weeks of gestation and has a reassuring FHR tracing, the bleeding is mild (<250 ml) and stops, and the patient is not in labor. The purpose of expectant management is to allow the fetus time to mature (Gilbert, 2007). The woman will initially be hospitalized in a labor and birth unit for continuous FHR and contraction monitoring. Large-bore intravenous access should be initiated immediately. Initial laboratory tests include hemoglobin, hematocrit, platelet count, and coagulation studies. A "type and screen" blood sample should be maintained at all times in the blood bank to allow for immediate crossmatch of blood component therapy if necessary. If the woman is at less than 34 weeks of gestation, antenatal corticosteroids should be administered (Francois & Foley, 2007; Gilbert). If the bleeding stops, the woman will most likely be placed on bedrest with bathroom privileges and limited activity (able to use the bathroom, shower, and move around her hospital room for 15 to 30 minutes at a time, four times a day). No vaginal or rectal examinations are performed, and the woman is placed on "pelvic rest" (nothing in the vagina). Ultrasonographic examinations may be performed every 2 to 3 weeks. Fetal surveillance may include an NST or BPP once or twice weekly. Bleeding is assessed by checking the amount of blood on perineal pads, bed pads, and linens. Serial laboratory values are evaluated for decreasing hemoglobin and hematocrit levels and changes in coagulation values. The woman should also be monitored for signs of preterm labor. Magnesium sulfate can be given for tocolysis if uterine contractions are identified (Francois & Foley, 2007; Gilbert, 2007). The woman with placenta previa should always be considered a potential emergency because massive blood loss with resulting hypovolemic shock can occur quickly if bleeding resumes. The possibility always exists that she will require an emergency cesarean for birth. Placenta previa in a preterm gestation may be an indication for transfer to a tertiary-care perinatal center, given that a neonatal intensive care unit may be necessary for care of the preterm neonate. Also, because many community hospitals are not prepared to perform emergency surgery 24 hours per day, 7 days per week, transfer to a tertiary-care center may be necessary to ensure constant access to cesarean birth. Home care Occasionally, women with placenta previa are discharged from the hospital before giving birth to be managed at home. The woman's condition should be stable, and she should have experienced no vaginal bleeding for at least 48 hours before discharge (Hull & Resnik, 2009). A candidate for home care must meet other strict criteria as well. She should be able and willing to comply with activity restrictions (bedrest with bathroom privileges and pelvic rest), have access to a telephone, close supervision by family or friends in the home, and constant access to transportation. If bleeding resumes, she will need to return to the hospital immediately. She must also be able to keep all appointments for fetal testing, laboratory assessments, and prenatal care. Visits by a perinatal home care nurse may be arranged. If hospitalization or home care with activity restriction is prolonged, the woman may have concerns about her work- or family-related responsibilities or may become bored with inactivity. She should be encouraged to participate in her own care and decisions about care as much as possible. Provision of diversionary activities or encouragement to participate in activities she enjoys and can perform during bed rest is needed. Participation in a support group made up of other women on bed rest while hospitalized or online if at home may be a helpful coping mechanism. (See Patient Instructions for Self-Management box: Coping with Activity Restrictions, p. 634). Active management If the woman is at or beyond 36 weeks of gestation or bleeding is excessive or persistent, immediate cesarean birth is indicated (Hull & Resnik, 2009). Expectant management will be terminated as soon as the fetus is mature, if excessive bleeding develops, active labor begins, or any other obstetric reason to terminate the pregnancy (e.g., chorioamnionitis) develops (Gilbert, 2007). Cesarean birth is indicated in all women with ultrasound evidence of placenta previa (Francois & Foley, 2007; Hull & Resnik). In women with placental "migration" or movement of the placenta in relationship to the internal os, a vaginal birth may be attempted (Cunningham et al., 2005). If cesarean birth is planned, the nurse continuously assesses maternal and fetal status while preparing the woman for surgery. Maternal vital signs are assessed frequently for decreasing BP, increasing pulse rate, changes in level of consciousness, and oliguria. Fetal assessment is maintained by continuous EFM to assess for signs of hypoxia. Blood loss may not cease with the birth of the infant. The large vascular channels in the lower uterine segment may continue to bleed because of that segment's diminished muscle content. The natural mechanism to control bleeding so characteristic of the upper part of the uterus—the interlacing muscle bundles, the "living ligature" contracting around open vessels—is absent in the lower part of the uterus. Postpartum hemorrhage may therefore occur even if the fundus is contracted firmly. Emotional support for the woman and her family is extremely important. The actively bleeding woman is concerned not only for her own well-being, but also for the well-being of her fetus. All procedures should be explained, and a support person should be present. The woman should be encouraged to express her concerns and feelings. If the woman and her support person or family desire pastoral support, the nurse can notify the hospital chaplain service or provide information about other supportive resources.

vacuum-assisted birth

Birth involving attachment of a vacuum cap to the fetal head (occiput) and application of negative pressure to assist in birth of the fetus

preterm birth

Birth occurring before the completion of 37 weeks of gestation

cesarean birth

Birth of a fetus by an incision through the abdominal wall and uterus

late preterm birth

Birth that occurs between 34 and 36 weeks of gestation

6. During the initial acute distress phase of grieving, parents still must make unexpected and unwanted decisions about funeral arrangements and even naming the baby. The nurse's role should be to: A. Take over as much as possible to relieve the pressure. B. Encourage grandparents to take over. C. Make sure the parents themselves approve the final decisions. D. Let them alone to work things out.

C. Make sure the parents themselves approve the final decisions. The nurse is always the patient's advocate. Nurses can offer support and guidance and leave room for the same from grandparents. However, in the end nurses should strive to let the parents make the final decisions.

2. The nurse is preparing to discharge a 30-year-old woman who has experienced a miscarriage at 10 weeks of gestation. Which statement by the woman would indicate a correct understanding of the discharge instructions? A. "I will not experience mood swings since I was only at 10 weeks of gestation." B. "I will avoid sexual intercourse for 6 weeks and pregnancy for 6 months." C. "I should eat foods that are high in iron and protein to help my body heal." D. "I should expect the bleeding to be heavy and bright red for at least 1 week."

C. "I should eat foods that are high in iron and protein to help my body heal." After a miscarriage a woman may experience mood swings and depression as a result of the reduction of hormones and the natural grieving process. Sexual intercourse should be avoided for 2 weeks or until the bleeding has stopped and pregnancy should be avoided for 2 months. A woman who has experienced a miscarriage should be advised to eat foods that are high in iron and protein to help replenish her body after the loss. The woman should not experience bright red, heavy, profuse bleeding; if such occurs, it should be reported to the health care provider.

10. The most common neurologic disorder accompanying pregnancy is: A. Eclampsia. B. Bell's palsy. C. Epilepsy. D. Multiple sclerosis.

C. Epilepsy. Eclampsia sometimes may be confused with epilepsy, which is the most common neurologic disorder accompanying pregnancy. Bell's palsy is a form of facial paralysis. Epilepsy is the most common neurologic disorder accompanying pregnancy. Epilepsy is the most common neurologic disorder accompanying pregnancy. The effects of pregnancy on epilepsy are unpredictable. Multiple sclerosis is a patchy demyelinization of the spinal cord that does not affect the normal course of pregnancy or birth. However, epilepsy is the most common neurologic disorder accompanying pregnancy.

7. In the past factors to determine whether a woman was likely to develop a high risk pregnancy were evaluated primarily from a medical point of view. A broader, more comprehensive approach to high risk pregnancy has been adopted today. There are now four categories based on threats to the health of the woman and the outcome of pregnancy. These categories include all of the following except: A. Biophysical. B. Psychosocial. C. Geographic. D. Environmental.

C. Geographic. Biophysical is one of the broad categories used for determining risk. These include genetic considerations, nutritional status, and medical and obstetric disorders. Psychosocial risks include smoking, caffeine, drugs, alcohol, and psychologic status. All of these adverse lifestyles can have a negative effect on the health of the mother or fetus. This category is correctly referred to as sociodemographic risk, not geographic risk. These factors stem from the mother and her family. Ethnicity may be one of the risks to pregnancy; however, it is not the only factor in this category. Low income, lack of prenatal care, age, parity, and marital status also are included. Environmental risks are those that can affect both fertility and fetal development. These include infections, chemicals, radiation, pesticides, illicit drugs, and industrial pollutants.

1. An infant was born 2 hours ago at 37 weeks of gestation, weighing 4.1 kg. The infant appears chubby with a flushed complexion and is very tremulous. The tremors are most likely the result of: A. Birth injury. B. Hypocalcemia. C. Hypoglycemia. D. Seizures.

C. Hypoglycemia. Hypoglycemia is common in the macrosomic infant. Signs of hypoglycemia include jitteriness, apnea, tachypnea, and cyanosis. This infant is macrosomic and at risk for hypoglycemia. The description is indicative of a macrocosmic infant. The tremors are jitteriness that is associated with hypoglycemia.

6. Nurses should be aware that HELLP syndrome: A. Is a mild form of preeclampsia. B. Can be diagnosed by a nurse alert to its symptoms. C. Is characterized by hemolysis, elevated liver enzymes, and low platelets. D. Is associated with preterm labor but not perinatal mortality.

C. Is characterized by hemolysis, elevated liver enzymes, and low platelets. HELLP syndrome is a variant of severe preeclampsia. HELLP syndrome is difficult to identify because the symptoms often are not obvious. It must be diagnosed in the laboratory. The acronym HELLP stands for hemolysis (H), elevated liver enzymes (EL), and low platelets (LP). Preterm labor is greatly increased in women with HELLP syndrome and so is perinatal mortality.

6. The nurse providing care for the antepartum woman should understand that the contraction stress test (CST): A. Sometimes uses vibroacoustic stimulation. B. Is an invasive test; however, contractions are stimulated. C. Is considered negative if no late decelerations are observed with the contractions. D. Is more effective than the nonstress test (NST) if the membranes have already been ruptured.

C. Is considered negative if no late decelerations are observed with the contractions. Vibroacoustic stimulation is sometimes used with the NST. The CST is invasive if stimulation is by intravenous oxytocin but not if by nipple stimulation. No late decelerations is good news. The CST is contraindicated if the membranes have ruptured.

6. The nurse providing care to a woman in labor should be aware that cesarean birth: A. Is declining in frequency in the twenty-first century in the United States. B. Is more likely to be done for the poor in public hospitals who do not get the nurse counseling that wealthier patients do. C. Is performed primarily for the benefit of the fetus. D. Can be either elected or refused by women as their absolute legal right.

C. Is performed primarily for the benefit of the fetus. Cesarean births are increasing in the United States in this century. Wealthier women who have health insurance and who give birth in a private hospital are more likely to experience cesarean birth. The most common indications for cesarean birth are danger to the fetus related to labor and birth complications. A woman's right to elect cesarean surgery is in dispute, as is her right to refuse it if in doing so she endangers the fetus. Legal issues are not absolutely clear.

5. With regard to the use of tocolytic therapy to suppress uterine activity, nurses should be aware that: A. The drugs can be given efficaciously up to the designated beginning of term at 37 weeks. B. There are no important maternal (as opposed to fetal) contraindications. C. Its most important function is to afford the opportunity to administer antenatal glucocorticoids. D. If the patient develops pulmonary edema while on tocolytics, intravenous (IV) fluids should be given.

C. Its most important function is to afford the opportunity to administer antenatal glucocorticoids. Once the pregnancy has reached 34 weeks, the risks of tocolytic therapy outweigh the benefits. There are important maternal contraindications to tocolytic therapy. Buying time for antenatal glucocorticoids to accelerate fetal lung development might be the best reason to use tocolytics. Tocolytic-induced edema can be caused by IV fluids.

cardiac decompensation

Condition of heart failure in which the heart is unable to maintain a sufficient cardiac output

6. What infection is contracted mostly by first-time mothers who are breastfeeding? A. Endometritis B. Wound infections C. Mastitis D. Urinary tract infections

C. Mastitis Endometritis is an infection of the uterus and is not confined to first-time mothers. Wound infections are most likely to occur after a cesarean birth and are not seen more often in first-time mothers. Mastitis is infection in a breast, usually confined to a milk duct. Most women who suffer this are first-timers who are breastfeeding. UTI can occur both during pregnancy and postpartum. Breastfeeding or being a first-time mother are not contributing factors.

11. As a result of large body surface in relation to weight, the preterm infant is at high risk for heat loss and cold stress. By understanding the four mechanisms of heat transfer (convection, conduction, radiation, and evaporation), the nurse can create an environment for the infant that prevents temperature instability. While evaluating the plan that has been implemented, the nurse knows that the infant is experiencing cold stress when he or she exhibits: A. Decreased respiratory rate. B. Bradycardia followed by an increased heart rate. C. Mottled skin with acrocyanosis. D. Increased physical activity.

C. Mottled skin with acrocyanosis. The respiratory rate increases followed by periods of apnea. The infant initially tries to conserve heat and burns more calories, after which the metabolic system goes into overdrive. In the preterm infant experiencing heat loss, the heart rate initially increases followed by periods of bradycardia. The infant has minimal-to-no fat stores. During times of cold stress the skin will become mottled, and acrocyanosis will develop, progressing to cyanosis. Even if the infant is being cared for on a radiant warmer or in an isolette, the nurse's role is to observe the infant frequently to prevent heat loss and respond quickly if signs and symptoms occur. In the term infant, the natural response to heat loss is increased physical activity. However, in a term infant experiencing respiratory distress or in a preterm infant, physical activity is decreased.

4. The first and most important nursing intervention when a nurse observes profuse postpartum bleeding is to: A. Call the woman's primary health care provider. B. Administer the standing order for an oxytocic. C. Palpate the uterus and massage it if it is boggy. D. Assess maternal blood pressure and pulse for signs of hypovolemic shock.

C. Palpate the uterus and massage it if it is boggy. Calling the woman's primary care provider is appropriate. However, the primary intervention should be to assess the uterus. Uterine atony is the leading cause of postpartum hemorrhage (PPH). Administering a standing order for an oxytocic is appropriate. However, the primary intervention should be to assess the uterus. Uterine atony is the leading cause of PPH. The initial management of excessive postpartum bleeding is firm massage of the uterine fundus. Assessing maternal blood pressure and pulse is appropriate. However, the primary intervention should be to assess the uterus. Uterine atony is the leading cause of PPH.

7. For clinical purposes, preterm and postterm infants are defined as: A. Preterm before 34 weeks if appropriate for gestational age (AGA); before 37 weeks if small for gestational age (SGA). B. Postterm after 40 weeks if large for gestational age (LGA); beyond 42 weeks if AGA. C. Preterm before 37 weeks, postterm beyond 42 weeks, no matter the size for gestational age at birth. D. Preterm, SGA before 38 to 40 weeks; postterm, LGA beyond 40 to 42 weeks.

C. Preterm before 37 weeks, postterm beyond 42 weeks, no matter the size for gestational age at birth. Preterm and postterm are strictly measures of time—before 37 weeks and beyond 42 weeks respectively—regardless of size for gestational age.

8. Bleeding disorders in late pregnancy include all of the following except: A. Placenta previa. B. Abruptio placentae. C. Spontaneous abortion. D. Cord insertion.

C. Spontaneous abortion. Placenta previa is a cause of bleeding disorders in later pregnancy. Abruptio placentae is a cause of bleeding disorders in later pregnancy. Spontaneous abortion is another name for miscarriage; by definition it occurs early in pregnancy. Cord insertion is a cause of bleeding disorders in later pregnancy.

8. A plan of care for an infant experiencing symptoms of drug withdrawal should include: A. Administering chloral hydrate for sedation. B. Feeding every 4 to 6 hours to allow extra rest. C. Swaddling the infant snugly and holding the baby tightly. D. Playing soft music during feeding.

C. Swaddling the infant snugly and holding the baby tightly. Phenobarbital or diazepam may be administered to decrease central nervous system (CNS) irritability. The infant should be fed in small, frequent amounts and burped well to diminish aspiration and maintain hydration. The infant should be wrapped snugly to reduce self-stimulation behaviors and protect the skin from abrasions. The infant should not be stimulated (such as with music) because this will increase activity and potentially increase CNS irritability.

developmental care

Care that takes into consideration the gestational age and condition of the infant and promotes the development of the infant

premature dilation of the cervix

Cervix that is unable to remain closed until a pregnancy reaches term because of a mechanical defect in the cervix; also called incompetent cervix

hemorrhagic (hypovolemic) shock

Clinical condition in which the peripheral blood flow is inadequate to return sufficient blood to the heart for normal function, particularly oxygen transport to the organs or tissue

shoulder dystocia

Condition in which the head is born but the anterior shoulder cannot pass under the pubic arch

cephalopelvic disproportion (CPD)

Condition in which the infant's head is of such a shape, size, or position that it cannot pass through the mother's pelvis, or the maternal pelvis is too small, abnormally shaped, or deformed to allow the passage of a fetus of average size

inversion of the uterus

Condition in which the uterus is turned inside out such that the fundus intrudes into the cervix or vagina

1. A woman who is at 36 weeks of gestation is having a nonstress test. Which statement by the woman would indicate a correct understanding of the test? A. "I will need to have a full bladder for the test to be done accurately." B. "I should have my husband drive me home after the test because I may be nauseous." C. "This test will help to determine whether the baby has Down syndrome or a neural tube defect." D. "This test will observe for fetal activity and an acceleration of the fetal heart rate to determine the well-being of the baby."

D. "This test will observe for fetal activity and an acceleration of the fetal heart rate to determine the well-being of the baby." An ultrasound is the test that requires a full bladder. A pregnant woman should be driven home after having an amniocentesis test. A nonstress test should not cause nausea. A maternal alpha-fetoprotein test is used in conjunction with unconjugated estriol levels, and human chorionic gonadotropin helps to determine the presence of Down syndrome. The nonstress test is one of the most widely used techniques to determine fetal well-being and is accomplished by monitoring fetal heart rate in conjunction with fetal activity and movements.

6. A woman with gestational diabetes has had little or no experience reading and interpreting glucose levels. She shows the nurse her readings for the past few days. Which one should the nurse tell her indicates a need for adjustment (insulin or sugar)? A. 75 mg/dl before lunch; this is low, better eat now B. 115 mg/dl 1 hour after lunch; this is a little high, maybe eat a little less next time C. 115 mg/dl 2 hours after lunch; this is too high, time for insulin D. 60 mg/dl just after waking up from a nap; this is too low, maybe eat a snack before going to sleep

D. 60 mg/dl just after waking up from a nap; this is too low, maybe eat a snack before going to sleep The premeal acceptable range is 65 to 95 mg/dl. The readings 1 hour after a meal should be less than 130 mg/dl. Two hours after eating, the readings should be under 120 mg/dl. A reading of 60 mg/dl is too low. During hours of sleep glucose levels should not be under 60 mg/dl. Snacks before sleeping can be helpful.

8. During a physical assessment of an at-risk patient, the nurse notes generalized edema, crackles at the base of the lungs, and some pulse irregularity. These are most likely signs of: A. Euglycemia. B. Rheumatic fever. C. Pneumonia. D. Cardiac decompensation.

D. Cardiac decompensation. Euglycemia is a condition of normal glucose levels. These symptoms indicate cardiac decompensation. Rheumatic fever can cause heart problems, but it does not present with these symptoms, which indicate cardiac decompensation. Pneumonia is an inflammation of the lungs and would not likely generate these symptoms, which indicate cardiac decompensation. These symptoms of cardiac decompensation may appear abruptly or gradually.

2. In planning for the care of a 30-year-old woman with pregestational diabetes, the nurse recognizes that the most important factor affecting pregnancy outcome is the: A. Mother's age. B. Number of years since diabetes was diagnosed. C. Amount of insulin required prenatally. D. Degree of glycemic control during pregnancy.

D. Degree of glycemic control during pregnancy. Mother's age is not relevant to the plan of care for a patient with pregestational diabetes. The number of years a woman has been a diabetic is not as important as determining the degree of glycemic control during the current pregnancy. The amount of insulin required prenatally is only relevant in relation to the current level of glycemic control. Women with excellent glucose control and no blood vessel disease should have good pregnancy outcomes.

1. Two hours after giving birth, a primiparous woman becomes anxious and complains of intense perineal pain with a strong urge to have a bowel movement. Her fundus is firm, at the umbilicus, and midline. Her lochia is moderate rubra with no clots. The nurse would suspect: A. Bladder distention. B. Uterine atony. C. Constipation. D. Hematoma formation.

D. Hematoma formation. Bladder distension would result in an elevation of the fundus above the umbilicus and deviation to the right or left of midline. Uterine atony would result in a boggy fundus. Constipation is unlikely at this time. Increasing perineal pressure along with a firm fundus and moderate lochial flow are characteristic of hematoma formation.

5. Nurses should be aware of the strengths and limitations of various biochemical assessments during pregnancy, including that: A. Chorionic villus sampling (CVS) is becoming more popular because it provides early diagnosis. B. Screening for maternal serum alpha-fetoprotein (MSAFP) levels is recommended only for women at risk for neural tube defects. C. Percutaneous umbilical blood sampling (PUBS) is one of the triple-marker tests for Down syndrome. D. MSAFP is a screening tool only; it identifies candidates for more definitive procedures.

D. MSAFP is a screening tool only; it identifies candidates for more definitive procedures. CVS does provide a rapid result, but it is declining in popularity because of advances in noninvasive screening techniques. MSAFP screening is recommended for all pregnant women. MSAFP, not PUBS, is part of the triple-marker tests for Down syndrome. MSAFP is a screening tool, not a diagnostic tool.

8. To provide adequate postpartum care, the nurse should be aware that postpartum depression (PPD) with psychotic features: A. Is more likely to occur in women with more than two children. B. Is rarely delusional and then usually about someone trying to harm her (the mother). C. Although serious, is not likely to need psychiatric hospitalization. D. May include bipolar disorder (formerly called "manic depression").

D. May include bipolar disorder (formerly called "manic depression"). PPD is more likely to occur in first-time mothers. Delusions may be present in 50% of women with PPD, usually involving something being wrong with the infant. PPD with psychosis is a psychiatric emergency that requires hospitalization. Manic mood swings are possible.

Bishop score

Rating system to evaluate inducibility (ripeness) of the cervix; a higher score increases the likelihood of a successful induction of labor

-Biophysical profile- components of- what is normal (8-10)

Real-time ultrasound permits detailed assessment of the physical and physiologic characteristics of the developing fetus and cataloging of normal and abnormal biophysical responses to stimuli. The biophysical profile (BPP) is a noninvasive dynamic assessment of a fetus that is based on acute and chronic markers of fetal disease. The BPP includes AFV, FBMs, fetal movements, and fetal tone determined by ultrasound and FHR reactivity determined by means of NST. The BPP may therefore be considered a physical examination of the fetus, including determination of vital signs. FHR reactivity, FBMs, fetal movement, and fetal tone reflect current central nervous system (CNS) status, whereas the AFV demonstrates the adequacy of placental function over a longer period (Tucker et al., 2009). BPP scoring and management are detailed in Tables 19-2 and 19-3. The BPP is used very frequently for antepartum fetal testing because it is a reliable predictor of fetal well-being. A BPP of 8 to 10 with a normal AFV is considered normal. Advantages of the test include excellent sensitivity and a low false-negative rate (Tucker et al., 2009). One limitation of the test is that, if the fetus is in a quiet sleep state, the BPP can require a long period of observation. Also, unless the ultrasound examination is videotaped, it cannot be reviewed (Druzin et al., 2007). Nursing role Although a growing number of nurses perform ultrasound scans and BPPs in certain centers, the main role of nurses is in counseling and educating women about the procedure. Ultrasound is widely used and, in fact, is considered a standard part of current prenatal care. Unlike many diagnostic tests, most women look forward to and enjoy their prenatal ultrasound. In the 30 years that diagnostic ultrasonography has been used, no evidence of any harmful effects on humans has emerged (Richards, 2007). TABLE 19-2 Biophysical Profile Scoring BIOPHYSICAL VARIABLE: Fetal breathing movements NORMAL (SCORE = 2): At least one episode of >30 seconds' duration in 30 minutes' observation ABNORMAL (SCORE = 0): Absent or no episode of ≥30 seconds' duration in 30 minutes BIOPHYSICAL VARIABLE: Gross body movement NORMAL (SCORE = 2): At least 3 discrete body/limb movements in 30 minutes (episodes of active continuous movement considered a single movement) ABNORMAL (SCORE = 0): Up to two episodes of body/limb movements in 30 minutes BIOPHYSICAL VARIABLE: Fetal tone NORMAL (SCORE = 2): At least one episode of active extension with return to flexion of fetal limb(s) or trunk, opening and closing of hand considered normal tone ABNORMAL (SCORE = 0): Either slow extension with return to partial flexion or movement of limb in full extension or absent fetal movement BIOPHYSICAL VARIABLE: Reactive fetal heart rate NORMAL (SCORE = 2): At least two episodes of acceleration of ≥15 bpm and 15 seconds/duration associated with fetal movement in 30 minutes ABNORMAL (SCORE = 0): Fewer than two accelerations or acceleration <15 bpm in 30 minutes BIOPHYSICAL VARIABLE: Qualitative amniotic fluid volume NORMAL (SCORE = 2): At least one pocket of amniotic fluid measuring 2 cm in two perpendicular planes ABNORMAL (SCORE = 0): Either no amniotic fluid pockets or a pocket <2 cm in two perpendicular planes TABLE 19-3 Biophysical Profile Management SCORE: 10 INTERPRETATION: Normal infant; low risk of chronic asphyxia MANAGEMENT: Repeat testing at weekly intervals; repeat twice weekly in diabetic patients and patients at 41 weeks of gestation SCORE: 8 INTERPRETATION: Normal infant; low risk of chronic asphyxia MANAGEMENT: Repeat testing at weekly intervals; repeat testing twice weekly in diabetic patients and patients at 41 weeks of gestation; oligohydramnios is an indication for delivery SCORE: 6 INTERPRETATION: Suspect chronic asphyxia MANAGEMENT: If 36 weeks of gestation and conditions are favorable, deliver; if at >36 weeks and L/S <2.0, repeat test in 4-6 hours; deliver if oligohydramnios is present SCORE: 4 INTERPRETATION: Suspect chronic asphyxia MANAGEMENT: If 36 weeks of gestation, deliver; if <32 weeks of gestation, repeat score SCORE: 0-2 INTERPRETATION: Strongly suspect chronic asphyxia MANAGEMENT: Extend testing time to 120 minutes; if persistent score ≤4, deliver, regardless of gestational age

7. For a woman at 42 weeks of gestation, which finding would require more assessment by the nurse? A. Fetal heart rate of 116 beats/min B. Cervix dilated 2 cm and 50% effaced C. Score of 8 on the biophysical profile D. One fetal movement noted in 1 hour of assessment by the mother

D. One fetal movement noted in 1 hour of assessment by the mother Fetal heart rate of 116 beats/min is a normal finding in a 42-week gestation. A cervix dilated to 2 cm and 50% effaced is a normal finding in a 42-week gestation. A score of 8 on the BPP is a normal finding in a 42-week gestation. Self-care in a postterm pregnancy should include performing daily fetal kick counts three times per day. The mother should feel four fetal movements per hour. If fewer than four movements have been felt by the mother, she should count for 1 more hour. Fewer than four movements in that hour warrants evaluation.

2. Postpartum women experience an increased risk for urinary tract infection. A prevention measure the nurse could teach the postpartum woman would be to: A. Acidify the urine by drinking 3 glasses of orange juice each day. B. Maintain a fluid intake of 1 to 2 L/day. C. Empty bladder every 4 hours throughout the day. D. Perform perineal care on a regular basis.

D. Perform perineal care on a regular basis. Urine is acidified with cranberry juice. The woman should drink at least 3 L of fluid each day. The woman should empty her bladder every 2 hours to prevent stasis of urine. Keeping the perineum clean will help prevent a urinary tract infection.

7. A woman at 39 weeks of gestation with a history of preeclampsia is admitted to the labor and birth unit. She suddenly experiences increased contraction frequency of every 1 to 2 minutes; dark red vaginal bleeding; and a tense, painful abdomen. The nurse suspects the onset of: A. Eclamptic seizure. B. Rupture of the uterus. C. Placenta previa. D. Placental abruption.

D. Placental abruption. Eclamptic seizures are evidenced by the presence of generalized tonic-clonic convulsions. Uterine rupture presents as hypotonic uterine activity, signs of hypovolemia, and in many cases the absence of pain. Placenta previa presents with bright red, painless vaginal bleeding. Uterine tenderness in the presence of increasing tone may be the earliest finding of premature separation of the placenta (abruptio placentae or placental abruption). Women with hypertension are at increased risk for an abruption.

3. A woman at 26 weeks of gestation is being assessed to determine whether she is experiencing preterm labor. What finding indicates that preterm labor is occurring? A. Estriol is not found in maternal saliva. B. Irregular, mild uterine contractions are occurring every 12 to 15 minutes. C. Fetal fibronectin is present in vaginal secretions. D. The cervix is effacing and dilated to 2 cm.

D. The cervix is effacing and dilated to 2 cm. Estriol is a form of estrogen produced by the fetus that is present in plasma at 9 weeks of gestation. Levels of salivary estriol have been shown to increase before preterm birth. Irregular, mild contractions that do not cause cervical change are not considered a threat. The presence of fetal fibronectin in vaginal secretions between 24 and 36 weeks of gestation could predict preterm labor, but it has only a 20% to 40% positive predictive value. Of more importance are other physiologic clues of preterm labor such as cervical changes. Cervical changes such as shortened endocervical length, effacement, and dilation are predictors of imminent preterm labor. Changes in the cervix accompanied by regular contractions indicate labor at any gestational age.

4. A 40-year-old woman is 10 weeks pregnant. Which diagnostic tool would be appropriate to suggest to her at this time? A. Biophysical profile B. Amniocentesis C. Maternal serum alpha-fetoprotein (MSAFP) D. Transvaginal ultrasound

D. Transvaginal ultrasound A biophysical profile would be a method of biophysical assessment of fetal well-being in the third trimester. An amniocentesis is performed after the fourteenth week of pregnancy. An MSAFP test is performed from week 15 to week 22 of the gestation (weeks 16 to 18 are ideal). An ultrasound is the method of biophysical assessment of the infant that would be performed at this gestational age.

-DTR's

DTRs ≥3+ indicates preeclampsia Deep tendon reflexes (DTRs) are evaluated as a baseline and to detect any changes. The biceps and patellar reflexes and ankle clonus are assessed and the findings recorded (Fig. 21-4; Table 21-4). To elicit the biceps reflex the examiner strikes a downward blow with a percussion hammer over the thumb, which is situated over the biceps tendon (Fig. 21-4, A). Normal response is flexion of the arm at the elbow, described as a 2+ response. The patellar reflex is elicited with the woman's legs hanging freely over the end of the examining table or with the woman lying on her side with the knee slightly flexed (Fig. 21-4, B). A blow with a percussion hammer is dealt directly to the patellar tendon, inferior to the patella. Normal response is the extension or kicking out of the leg. To assess for hyperactive reflexes (clonus) at the ankle joint the examiner supports the leg with the knee flexed (Fig. 21-4, C). With one hand the examiner sharply dorsiflexes the foot, maintains the position for a moment, and then releases the foot. Normal (negative clonus) response is elicited when no rhythmic oscillations (jerks) are felt while the foot is held in dorsiflexion. When the foot is released, no oscillations are seen as the foot drops to the plantar-flexed position. Abnormal (positive clonus) response is recognized by rhythmic oscillations of one or more "beats" felt when the foot is in dorsiflexion and seen as the foot drops to the plantar-flexed position. TABLE 21-4 Assessing Deep Tendon Reflexes GRADE: DEEP TENDON REFLEX RESPONSE 0: No response 1+: Sluggish or diminished 2+: Active or expected response 3+: More brisk than expected, slightly hyperactive 4+: Brisk, hyperactive, with intermittent or transient clonus

uteroplacental insufficiency (UPI)

Decline in placental function (exchange of gases, nutrients, and wastes) leading to fetal hypoxia and acidosis; evidenced by late decelerations of the fetal heart rate in response to uterine contractions

hypothyroidism

Deficiency of thyroid gland activity with underproduction of thyroxine

8. __________ is defined as long, difficult, or abnormal labor. It is caused by various conditions associated with the five factors affecting labor.

Dystocia Abnormal uterine activity can be further defined as being hypertonic or hypotonic. Contractions may be frequent and painfully strong with hypertonic uterine activity but are ineffective in promoting cervical effacement and dilation. With hypotonic uterine activity, the rise in uterine pressure generated during contractions is insufficient to promote cervical effacement and dilation.

10. __________ is responsible for 10% to 15% of all maternal mortality and is the leading cause of infertility.

Ectopic pregnancy

cervical funneling

Effacement of the internal cervical os

neutral thermal environment (NTE)

Environment that enables the neonate to maintain a normal body temperature with minimum use of oxygen and energy

amniotic fluid index (AFI)

Estimation of amount of amniotic fluid by means of ultrasound to determine excess or decrease

nonstress test (NST)

Evaluation of fetal response (fetal heart rate) to natural contractile uterine activity or to an increase in fetal activity

hyperglycemia

Excess glucose in the blood, usually caused by inadequate secretion of insulin by the islet cells of the pancreas or inadequate control of diabetes mellitus

postpartum hemorrhage (PPH)

Excessive bleeding after childbirth; traditionally defined as a loss of 500 ml or more after a vaginal birth and 1000 ml after a cesarean birth

hyperthyroidism

Excessive functional activity of the thyroid gland

subinvolution

Failure of a part (e.g., the uterus) to reduce to its normal size and condition after enlargement from functional activity (e.g., pregnancy)

alpha-fetoprotein (AFP)

Fetal antigen; elevated levels in amniotic fluid and maternal blood are associated with neural tube defects

meconium aspiration syndrome (MAS)

Function of fetal hypoxia; with hypoxia the anal sphincter relaxes and meconium is released; reflex gasping movements draw meconium and other particulate matter in the amniotic fluid into the infant's bronchial tree, obstructing the airflow after birth

hydatidiform mole (molar pregnancy)

Gestational trophoblastic neoplasm usually resulting from fertilization of an egg that has no nucleus or an inactivated nucleus

vaginal birth after cesarean (VBAC)

Giving birth vaginally after having had a previous cesarean birth

gestational diabetes mellitus (GDM)

Glucose intolerance first recognized during pregnancy

glycosylated hemoglobin A1c

Glycohemoglobin, a minor hemoglobin with glucose attached; the glycosylated hemoglobin concentration represents the average blood glucose level over the previous several weeks and is a measurement of glycemic control in diabetic therapy

autoimmune disorders

Group of diseases that disrupt the function of the immune system, causing the body to produce antibodies against itself, resulting in tissue damage

inborn error of metabolism

Group of recessive disorders caused by a metabolic defect that results from the absence of or change in a protein, usually an enzyme, and mediated by the action of a certain gene

-hypertension- most common complication of pregnancy

HYPERTENSION IN PREGNANCY ▪ Significance and Incidence Hypertensive disorders are the most common medical complication of pregnancy, occurring in 5% to 10% of all pregnancies. The incidence varies among hospitals, regions, and countries. Hypertensive disorders are a major cause of maternal and perinatal morbidity and mortality worldwide (Sibai, 2007). The four most common types of hypertensive disorders occurring in pregnancy are (1) gestational hypertension, (2) preeclampsia, (3) chronic hypertension, and (4) preeclampsia superimposed on chronic hypertension (Gilbert, 2007). Classification The classification of hypertensive disorders in pregnancy is confusing because standard definitions are not consistently used by all health care providers. The classification system most commonly used in the United States today is based on reports from the American College of Obstetricians and Gynecologists (ACOG) (2002) and the National High Blood Pressure Education Program Working Group on High Blood Pressure in Pregnancy (Working Group) (2000). This classification system is summarized in Table 21-1. TABLE 21-1 Classification of Hypertensive States of Pregnancy GESTATIONAL HYPERTENSIVE DISORDERS Gestational hypertension: Development of mild hypertension after week 20 of pregnancy in previously normotensive woman without proteinuria Preeclampsia: Development of hypertension and proteinuria in previously normotensive woman after 20 weeks of gestation or in early postpartum period; in presence of trophoblastic disease, preeclampsia can develop before 20 weeks of gestation Eclampsia: Development of convulsions or coma not attributable to other causes in preeclamptic woman CHRONIC HYPERTENSIVE DISORDERS Chronic hypertension: Hypertension or proteinuria (or both) in pregnant woman present before pregnancy or diagnosed before 20 weeks of gestation and persistent after 6 weeks postpartum Superimposed preeclampsia or eclampsia: In women with hypertension before 20 weeks of gestation: new-onset proteinuria (≥0.5 g protein in a 24-hr collection). In women with both hypertension and proteinuria before 20 weeks of gestation: significant increase in hypertension, plus one of the following: new onset of symptoms, thrombocytopenia, or elevated liver enzymes Gestational hypertension Gestational hypertension is the onset of hypertension without proteinuria after week 20 of pregnancy (ACOG, 2002; Working Group, 2000). Hypertension is defined as a systolic blood pressure (BP) greater than 140 mmHg or a diastolic BP greater than 90 mmHg. The hypertension should be recorded on at least two separate occasions at least 4 to 6 hours apart but within a maximum of a 1-week period (ACOG, 2002; Sibai, 2007; Working Group, 2000). Both the Working Group and the American Heart Association (AHA) have published extensive recommendations for accurately measuring BP (Pickering et al., 2005; Working Group, 2000). Box 21-1 provides detailed instructions for measuring BP. Gestational hypertension is the most frequent cause of hypertension during pregnancy, with an incidence of 6% to 17% in primigravidas and 2% to 4% in multiparous women. It occurs much more frequently in women with multifetal pregnancies than in other women (Sibai, 2007). Gestational hypertension usually develops at or after 37 weeks of gestation. Women with gestational hypertension have no evidence of preexisting hypertension, and their BPs return to normal levels within 6 weeks after giving birth. Women with mild gestational hypertension usually have good pregnancy outcomes. Some women who are initially thought to have gestational hypertension will eventually be diagnosed with chronic hypertension instead. Others will go on to develop proteinuria, thereby changing their diagnosis to preeclampsia. Women who are diagnosed with gestational hypertension before 35 weeks of gestation are more likely to progress to preeclampsia than women whose onset of hypertension occurs closer to term (Sibai).

erythroblastosis fetalis

Hemolytic disease of the newborn usually caused by isoimmunization resulting from Rh incompatibility or ABO incompatibility

ABO incompatibility

Hemolytic disease that occurs when the mother's blood type is O and the newborn's is A, B, or AB

-hemmorrhagic shock- signs and symptoms

Hemorrhage may result in hemorrhagic (hypovolemic) shock. Shock is an emergency situation in which the perfusion of body organs may become severely compromised and death may occur. Physiologic compensatory mechanisms are activated in response to hemorrhage. The adrenal glands release catecholamines, causing arterioles and venules in the skin, lungs, gastrointestinal tract, liver, and kidneys to constrict. The available blood flow is diverted to the brain and heart and away from other organs, including the uterus. If shock is prolonged, the continued reduction in cellular oxygenation results in an accumulation of lactic acid and acidosis (from anaerobic glucose metabolism). Acidosis (reduced serum pH) causes arteriolar vasodilation; venule vasoconstriction persists. A circular pattern is established; that is, decreased perfusion, increased tissue anoxia and acidosis, edema formation, and pooling of blood further decrease the perfusion. Cellular death occurs. (See the Emergency box for assessments and interventions for hemorrhagic shock.) EMERGENCY: Hemorrhagic Shock ASSESSMENTS: CHARACTERISTICS Respirations: Rapid and shallow Pulse: Rapid, weak, irregular Blood pressure: Decreasing (late sign) Skin: Cool, pale, clammy Urinary output: Decreasing Level of consciousness: Lethargy → coma Mental status: Anxiety → coma Central venous pressure: Decreased INTERVENTIONS • Summon assistance and equipment. • Start intravenous infusion per standing orders. • Ensure patent airway; administer oxygen. • Continue to monitor status. CARE MANAGEMENT ▪ Medical Management Vigorous treatment is necessary to prevent adverse sequelae. Medical management of hypovolemic shock involves restoring circulating blood volume and treating the cause of the hemorrhage (e.g., lacerations, uterine atony, or inversion). To restore circulating blood volume a rapid IV infusion of crystalloid solution is given at a rate of 3 ml infused for every 1 ml of estimated blood loss (e.g., 3000 ml infused for 1000 ml of blood loss). Packed RBCs are usually infused if the woman is still actively bleeding and no improvement in her condition is noted after the initial crystalloid infusion. Infusion of fresh-frozen plasma may be needed if clotting factors and platelet counts are below normal values (Cunningham et al., 2005; Francois & Foley, 2007). Nursing Interventions Hemorrhagic shock can occur rapidly, but the classic signs of shock may not appear until the postpartum woman has lost 30% to 40% of blood volume. The nurse must continue to reassess the woman's condition, as evidenced by the degree of measurable and anticipated blood loss, and mobilize appropriate resources. Most interventions are instituted to improve or monitor tissue perfusion. The nurse continues to monitor the woman's pulse and blood pressure. If invasive hemodynamic monitoring is ordered, the nurse may assist with the placement of the central venous pressure (CVP) or pulmonary artery (Swan-Ganz) catheter and monitor CVP, pulmonary artery pressure, or pulmonary artery wedge pressure as ordered (Gilbert, 2007). Additional assessments to be made include evaluating skin temperature, color, and turgor, as well as assessing the woman's mucous membranes. Breath sounds should be auscultated before fluid volume replacement, if possible, to provide a baseline for future assessment. Inspection for oozing at the sites of incisions or injections and assessment of the presence of petechiae or ecchymosis in areas not associated with surgery or trauma are critical in the evaluation for DIC. Oxygen is administered, preferably by nonrebreathing facemask, at 10 to 12 L/min to maintain oxygen saturation. Oxygen saturation should be monitored with a pulse oximeter, although measurements may not always be accurate in a woman with hypovolemia or decreased perfusion. Level of consciousness is assessed frequently and provides an additional indication of blood volume and oxygen saturation (Gilbert, 2007). In early stages of decreased blood flow the woman may report "seeing stars" or feeling dizzy or nauseated. She may become restless and orthopneic. As cerebral hypoxia increases, she may become confused and react slowly or not at all to stimuli. Some women complain of headaches (Curran, 2003). An improved sensorium is an indicator of improved perfusion. Continuous electrocardiographic monitoring may be indicated for the woman who is hypotensive or tachycardic, continues to bleed profusely, or is in shock. A Foley catheter with a urometer is inserted to allow hourly assessment of urinary output. The most objective and least invasive assessment of adequate organ perfusion and oxygenation is urinary output of at least 30 ml/hr (Cunningham et al., 2005). Blood may be drawn and sent to the laboratory for studies that include hemoglobin and hematocrit levels, platelet count, and coagulation profile. Fluid or Blood Replacement Therapy Critical to successful management of the woman with a hemorrhagic complication is the establishment of venous access, preferably with a large-bore IV catheter. The establishment of two IV lines facilitates fluid resuscitation. Vigorous fluid resuscitation includes the administration of crystalloids (lactated Ringer's, normal saline solutions), colloids (albumin), blood, and blood components (Francois & Foley, 2007). Fluid resuscitation must be closely monitored because fluid overload may occur. Intravascular fluid overload occurs more frequently with colloid therapy compared with other fluids. Transfusion reactions may follow the administration of blood or blood components, including cryoprecipitates. Even in an emergency, each unit must be checked per hospital protocol. Complications of fluid or blood replacement therapy include hemolytic reactions, febrile reactions, allergic reactions, circulatory overload, and air embolism. LEGAL TIP: Standard of Care for Bleeding Emergencies The standard of care for obstetric emergency situations such as PPH or hypovolemic shock is that provision should be made for the nurse to implement actions independently. Policies, procedures, standing orders or protocols, and clinical guidelines should be established by each health care facility in which births occur and should be agreed on by health care providers involved in the care of obstetric patients.

-diabetic ketoacidosis- medical emergency and can lead to maternal and fetal death

Hyperinsulinemia accounts for many of the problems the fetus or infant develops. In addition to fluctuating glucose levels, maternal vascular involvement or superimposed maternal infection adversely affects the fetus. Normally, maternal blood has a more alkaline pH than carbon dioxide-rich fetal blood. This phenomenon encourages the exchange of oxygen and carbon dioxide across the placental membrane. When the maternal blood is more acidotic than the fetal blood, such as during ketoacidosis, little carbon dioxide or oxygen exchange occurs at the level of the placenta. The mortality for the unborn infant resulting from an episode of maternal ketoacidosis may be as high as 50% or more (Kalhan & Parimi, 2006).

ectopic pregnancy

Implantation of the fertilized ovum outside of the uterine cavity; locations include the uterine tubes, ovaries, and abdomen

peripartum cardiomyopathy

Inability of the heart to maintain an adequate cardiac output; congestive heart failure occurring during the peripartum

Coombs test

Indirect: determination of Rh-positive antibodies in maternal blood; direct: determination of maternal Rh-positive antibodies in fetal cord blood; positive test result indicates the presence of antibodies or titer

mastitis

Infection in a breast, usually confined to a milk duct, characterized by influenza-like symptoms and redness and tenderness in the affected breast

puerperal infection

Infection of the pelvic organs during the postbirth period; also called postpartum infection

TORCH

Infections caused by organisms that damage the embryo or fetus; acronym for toxoplasmosis, other (e.g., syphilis), rubella, cytomegalovirus, and herpes simplex

TORCH infections

Infections caused by organisms that damage the embryo or fetus; acronym for toxoplasmosis, other (e.g., syphilis), rubella, cytomegalovirus, and herpes simplex virus

chorioamnionitis

Inflammatory reaction in fetal membranes to bacteria or viruses in the amniotic fluid, which then become infiltrated with polymorphonuclear leukocytes

Couvelaire uterus

Interstitial myometrial hemorrhage after premature separation (abruption) of placenta; purplish-blue discoloration of the uterus is noted

-magnesium sulfate- indications, uses, side-effects, signs of toxicity, normal blood levels- in your book)

Label: Important "Magnesium sulfate is the most commonly used tocolytic agent because clinicians are familiar with its use as treatment of preeclampsia and its presumed safety as compared with beta-adrenergic agonists. Evidence for its effectiveness as a tocolytic is weak, however. Magnesium sulfate apparently promotes smooth-muscle relaxation by competing with calcium in cells (Iams & Romero, 2007; Rideout, 2005). Magnesium sulfate is administered intravenously. It may be a good choice for use in patients in whom other tocolytic agents are contraindicated (see Medication Guide: Tocolytic Therapy for Preterm Labor) (Iams & Romero, 2007; Iams et al., 2009). Medication Guide: Tocolytic Therapy for Preterm Labor Medication: MAGNESIUM SULFATE Action: CNS depressant; relaxes smooth muscles including uterus Dosage and Route: • Intravenous fluid should contain 40 g in 1000 ml, piggyback to primary infusion, and administer using controller pump • Loading dose: 4-6 g over 20- 30 min • Maintenance dose: 1-4 g/hr • Use for stabilization only • Discontinue within 24-48 hours at the maintenance dose or if intolerable adverse reactions occur Adverse Effects: Maternal adverse reactions: • Hot flushes, sweating, burning at the IV insertion site, nausea and vomiting, dry mouth, drowsiness, blurred vision, diplopia, headache, ileus, generalized muscle weakness, lethargy, dizziness • Hypocalcemia • SOB • Transient hypotension • Some reactions may subside when loading dose is completed Intolerable adverse reactions: • Respiratory rate fewer than 12 breaths/min • Pulmonary edema • Absent DTRs • Chest pain • Severe hypotension • Altered level of consciousness • Extreme muscle weakness • Urine output less than 25-30 ml/hr or less than 100 ml/4 hours • Serum magnesium level of 10 mEq/L (9 mg/dl) or greater Fetal (uncommon): • Decreased breathing movement, reduced FHR variability, nonreactive NST Nursing Considerations: • Assess woman and fetus to obtain baseline before beginning therapy and then before and after each increment; follow frequency of agency protocol • Monitor serum magnesium levels with higher doses; therapeutic range is between 4 and 7.5 mEq/L or 5-8 mg/dl • Discontinue infusion and notify physician if intolerable adverse reactions occur • Ensure that calcium gluconate 1 g (10 ml of 10% solution) is available for emergency administration to reverse magnesium sulfate toxicity • Should not be given to women with myasthenia gravis • Total IV intake should be limited to 125 ml/hr

macrosomia

Large body size as seen in neonates of mothers with pregestational or gestational diabetes

pelvic relaxation

Lengthening and weakening of the fascial supports of pelvic structures

hypoglycemia

Less than a normal amount of glucose in the blood; usually caused by the administration of too much insulin, excessive secretion of insulin by the islet cells of the pancreas, or dietary deficiency

miscarriage

Loss of pregnancy that occurs naturally without interference or known cause; also called spontaneous abortion

-low birth weight, extremely low birth weight- definitions

Low Birth Weight and Preterm Birth The risks of morbidity and mortality increase for newborns weighing less than 2500 g (5 lb, 8 oz)—LBW infants. Multiple births contribute to the incidence of LBW. In 2006 the incidence of LBW births was 8.3%, and the incidence of very low-birth-weight (VLBW; less than 1500 g [3.3 lb]) births was 1.4% (Martin et al., 2008). Racial disparity exists in the incidence of LBW. Non-Hispanic black babies are twice as likely as non-Hispanic white babies to be LBW and to die within the first year of life. By race the incidence of LBW for non-Hispanic black births was 14%; for non-Hispanic white births, 7.3%; and for Hispanic births, 6.9%. Cigarette smoking is associated with LBW, prematurity, and intrauterine growth restriction. In 2005, 10.7% of pregnant women smoked, a proportion that has declined slightly from 2004 (Martin et al.). Preterm Birth versus Low Birth Weight Although they have distinctly different meanings, the terms preterm birth or prematurity and low birth weight were often used interchangeably in the past. Preterm birth describes the length of gestation (i.e., less than 37 weeks regardless of the weight of the infant), whereas low birth weight describes only weight at the time of birth (i.e., ≤2500 g). Because birth weight was far easier to determine than gestational age, in many settings and publications, low birth weight was used as a substitute term for preterm birth. Preterm birth, however, is a more dangerous health condition for an infant than low birth weight because a decreased length of time in the uterus correlates with immaturity of body systems. Low birth weight babies can be, but are not necessarily, preterm; low birth weight can be caused by conditions other than preterm birth, such as intrauterine growth restriction (IUGR), a condition of inadequate fetal growth not necessarily correlated with initiation of labor. On the other hand, infants born at a preterm gestation can weigh more than 2500 g at birth. Today, thanks to advances in pregnancy dating, outcomes related to gestational age can be increasingly distinguished from outcomes related to birth weight (Iams et al., 2009). BOX 24-1 Classification of High Risk Infants CLASSIFICATION ACCORDING TO SIZE Low-birth-weight (LBW) infant: an infant whose birth weight is less than 2500 g, regardless of gestational age Very-low-birth-weight (VLBW) infant: an infant whose birth weight is less than 1500 g Extremely-low-birth-weight (ELBW) infant: an infant whose birth weight is less than 1000 g Late preterm (near term) infant: an infant born between and weeks of gestation, regardless of birth weight* Appropriate-for-gestational-age (AGA) infant: an infant whose birth weight falls between the 10th and 90th percentiles on intrauterine growth curves Small-for-date (SFD) or small-for-gestational-age (SGA) infant: an infant whose rate of intrauterine growth was restricted and whose birth weight falls below the 10th percentile on intrauterine growth curves Large-for-gestational-age (LGA) infant: an infant whose birth weight falls above the 90th percentile on intrauterine growth charts Intrauterine growth restriction (IUGR): found in infants whose intrauterine growth is restricted (sometimes used as a more descriptive term for the SGA infant) Symmetric IUGR: growth restriction in which the weight, length, and head circumference are all affected Asymmetric IUGR: growth restriction in which the head circumference remains within normal parameters while the birth weight falls below the 10th percentile

daily fetal movement count (DFMC)

Maternal assessment of fetal activity; the number of fetal movements within a specified time are counted; also called "kick count"

continuous positive airway pressure (CPAP)

Means of infusing oxygen or air under a preset pressure via nasal prongs, a facemask, or an endotracheal tube

antenatal glucocorticoids

Medications administered to the mother for the purpose of accelerating fetal lung maturity when an increased risk exists for preterm birth between 24 and 34 weeks of gestation

tocolytics

Medications used to suppress uterine activity and relax the uterus in cases of hyperstimulation or preterm labor

-diabetes during pregnancy- risk for diabetes later on in life, insulin needs at the beginning of preg (low) and later in preg (higher)

Metabolic changes associated with pregnancy Normal pregnancy is characterized by complex alterations in maternal glucose metabolism, insulin production, and metabolic homeostasis. During normal pregnancy, adjustments in maternal metabolism allow for adequate nutrition for both the mother and the developing fetus. Glucose, the primary fuel used by the fetus, is transported across the placenta through the process of carrier-mediated facilitated diffusion, meaning that the glucose levels in the fetus are directly proportional to maternal levels. Although glucose crosses the placenta, insulin does not. Around the tenth week of gestation the fetus begins to secrete its own insulin at levels adequate to use the glucose obtained from the mother. Therefore, as maternal glucose levels rise, fetal glucose levels are increased, resulting in increased fetal insulin secretion. TABLE 20-1 White's Classification of Diabetes in Pregnancy (Modified) GESTATIONAL DIABETES Class A1: Patient has two or more abnormal values on the OGTT with a normal fasting blood sugar. Blood glucose levels are diet controlled. Class A2: Patient was not known to have diabetes before pregnancy but requires medication for blood glucose control. PREGESTATIONAL DIABETES Class B: Onset of disease occurs after age 20 and duration of illness <10 years. Class C: Onset of disease occurs between 10 and 19 years of age or duration of illness for 10-19 years or both. Class D: Onset of disease occurs <10 years of age or duration of illness >20 years or both. Class F: Patient has developed diabetic nephropathy. Class R: Patient has developed retinitis proliferans. Class T: Patient has had a renal transplant. During the first trimester of pregnancy the pregnant woman's metabolic status is significantly influenced by the rising levels of estrogen and progesterone. These hormones stimulate the beta cells in the pancreas to increase insulin production, which promotes increased peripheral use of glucose and decreased blood glucose, with fasting levels being reduced by approximately 10% (Fig. 20-1, A). At the same time, an increase in tissue glycogen stores and a decrease in hepatic glucose production occur, which further encourage lower fasting glucose levels. As a result of these normal metabolic changes of pregnancy, women with insulin-dependent diabetes are prone to hypoglycemia during the first trimester. During the second and third trimesters, pregnancy exerts a "diabetogenic" effect on the maternal metabolic status. Because of the major hormonal changes, decreased tolerance to glucose, increased insulin resistance, decreased hepatic glycogen stores, and increased hepatic production of glucose occur. Rising levels of human chorionic somatomammotropin, estrogen, progesterone, prolactin, cortisol, and insulinase increase insulin resistance through their actions as insulin antagonists. Insulin resistance is a glucose-sparing mechanism that ensures an abundant supply of glucose for the fetus. Maternal insulin requirements gradually increase from approximately 18 to 24 weeks of gestation to approximately 36 weeks of gestation. Maternal insulin requirements may double or quadruple by the end of the pregnancy (Fig. 20-1, B and C). At birth, expulsion of the placenta prompts an abrupt drop in levels of circulating placental hormones, cortisol, and insulinase (Fig. 20-1, D). Maternal tissues quickly regain their prepregnancy sensitivity to insulin. For the nonbreastfeeding mother the prepregnancy insulin-carbohydrate balance usually returns in approximately 7 to 10 days (Fig. 20-1, E). Lactation uses maternal glucose; therefore the breastfeeding mother's insulin requirements will remain low during lactation. On completion of weaning the mother's prepregnancy insulin requirement is reestablished (Fig. 20-1, F). Gestational Diabetes Mellitus Gestational diabetes mellitus (GDM), complicates approximately 3% to 9% of all pregnancies (Moore & Catalano, 2009) and accounts for more than 90% of all cases of diabetic pregnancy (Landon et al., 2007). According to White's classification system, these women fall into classes A1 and A2 (see Table 20-1). GDM is more likely to occur among Latina, Native American, Asian, and African-American women than in Caucasians (Moore & Catalano). GDM is likely to recur in future pregnancies, and the risk for development of overt diabetes in later life is also increased (Moore & Catalano). This tendency is especially true of women whose GDM is diagnosed early in pregnancy or who are obese (Landon et al.). Classic risk factors for GDM include maternal age over 25 years, previous macrosomic infant, previous unexplained IUFD, previous pregnancy with GDM, strong immediate family history of type 2 diabetes or GDM, obesity (weight >90 kg [198 pounds]), and fasting blood glucose above 140 mg/dl or random blood glucose above 200 mg/dl. Women at high risk for developing GDM should have glucola screening at the first prenatal visit and again at 24 to 28 weeks of gestation if the initial screen is negative (Landon et al.). GDM is usually diagnosed during the second half of pregnancy. As fetal nutrient demands rise during the late second and the third trimesters, maternal nutrient ingestion induces greater and more sustained levels of blood glucose. At the same time, maternal insulin resistance is also increasing because of the insulin-antagonistic effects of the placental hormones, cortisol, and insulinase. Consequently, maternal insulin demands rise as much as threefold. Most pregnant women are capable of increasing insulin production to compensate for insulin resistance and to maintain euglycemia. When the pancreas is unable to produce sufficient insulin or the insulin is not used effectively, however, gestational diabetes can result. Fetal risks No increase in the incidence of birth defects has been found among infants of women who develop gestational diabetes after the first trimester because the critical period of organ formation has already passed by that time (Moore & Catalano, 2009). However, Anderson, Waller, Canfield, Shaw, Watkins, and Werler (2005) found that women who were obese before conception (BMI >30 kg/m2) and developed gestational diabetes were at greater risk to give birth to infants with CNS defects. Screening for gestational diabetes mellitus All pregnant women should be screened for GDM by history, clinical risk factors, or laboratory screening of blood glucose levels. Based on history and clinical risk factors, some women are at low risk for the development of GDM. Therefore glucose testing for this low risk population is not cost effective (ADA, 2008). This group includes normal-weight women younger than 25 years of age who have no family history of diabetes, are not members of an ethnic or a racial group known to have a high prevalence of the disease, and have no previous history of abnormal glucose tolerance or adverse obstetric outcomes usually associated with GDM (ADA). The screening test (glucola screening) most often used consists of a 50-g oral glucose load followed by a plasma glucose measurement 1 hour later. The woman need not be fasting. A glucose value of 130 to 140 mg/dl is considered a positive screen and should be followed by a 3-hour (100-g) oral glucose tolerance test (OGTT). The OGTT is administered after an overnight fast and at least 3 days of unrestricted diet (at least 150 g of carbohydrate) and physical activity. The woman is instructed to avoid caffeine because it will increase glucose levels and to abstain from smoking for 12 hours before the test. The 3-hour OGTT requires a fasting blood glucose level, which is drawn before giving a 100-gram glucose load. Blood glucose levels are then drawn 1, 2, and 3 hours later. The woman is diagnosed with gestational diabetes if two or more values are met or exceeded (Moore & Catalano, 2009) (Fig. 20-4). Nursing diagnoses and expected outcomes of care for women with GDM are basically the same as those for women with pregestational diabetes except that the time frame for planning may be shortened with GDM because the diagnosis is usually made later in pregnancy (see Nursing Process box: Preexisting Diabetes). CARE MANAGEMENT ▪ Antepartum When the diagnosis of gestational diabetes is made, treatment begins immediately, allowing little or no time for the woman and her family to adjust to the diagnosis before they are expected to participate in the treatment plan. With each step of the treatment plan the nurse and other health care providers should educate the woman and her family, providing detailed and comprehensive explanations to ensure understanding, participation, and adherence to the necessary interventions. Potential complications should be discussed, and the need for maintenance of euglycemia throughout the remainder of the pregnancy reinforced. Knowing that gestational diabetes typically disappears when the pregnancy is over may be reassuring for the woman and her family. As with pregestational diabetes, the aim of therapy in women with GDM is strict blood glucose control. Fasting blood glucose levels should range from 65 to 95 mg/dl, and 1-hour postprandial blood glucose levels should be less than 130 to 140 mg/dl (Moore & Catalano, 2009). Diet Dietary modification is the mainstay of treatment for GDM. The woman with GDM is placed on a standard diabetic diet. The usual prescription is 30 kcal/kg/day based on a normal preconceptional weight. For obese women the usual prescription is up to 25 kcal/kg/day, which translates into 1500 to 2000 kcal/day for most women (Moore & Catalano, 2009). Carbohydrate intake is restricted to approximately 50% of caloric intake (Moore & Catalano). Dietary counseling by a nutritionist is recommended. Exercise Exercise in women with GDM helps lower blood glucose levels and may be instrumental in decreasing the need for insulin (Gilbert, 2007). Women with GDM who already have an active lifestyle should be encouraged to continue an exercise program. Monitoring blood glucose levels Blood glucose monitoring is necessary to determine whether euglycemia can be maintained by diet and exercise. Women are instructed to monitor their blood sugar daily. The frequency and timing of blood glucose monitoring should be individualized for each woman. However, a typical schedule for monitoring blood glucose is on rising in the morning, after breakfast, before and after lunch, after dinner, and at bedtime (Moore & Catalano, 2009). Women with GDM may perform self-monitoring at home, or monitoring may be performed at the clinic or office visit. Medications for controlling blood glucose levels Up to 20% of women with GDM will require insulin during the pregnancy to maintain adequate blood glucose levels, despite compliance with the prescribed diet. In contrast to women with insulin-dependent diabetes, women with gestational diabetes are initially managed with diet and exercise alone. If fasting plasma glucose levels are greater than 95 mg/dl or 2-hour postprandial levels are greater than 120 mg/dl, then insulin therapy is begun (Gilbert, 2007) (see Table 20-3). Glyburide, an oral hypoglycemic agent, is currently being used more frequently with women with GDM instead of insulin. The fact that only minimal amounts of glyburide cross the placenta to the fetus makes it a good drug for use during pregnancy. It has also been used in women with type 2 diabetes who required large amounts of insulin to achieve glucose control with smaller insulin doses. Recent studies have shown that glyburide should be taken at least 30 minutes (preferably 1 hour) before a meal so that its peak effect covers the 2-hour postprandial blood glucose level. Because episodes of hypoglycemia can occur between meals, women taking glyburide should always carry with them sources of fast sugar (Moore & Catalano, 2009). Women with diabetes who are unable or unwilling to take insulin by injection or are cognitively impaired may be candidates for glyburide use. Fetal surveillance No standard recommendation has been formulated for fetal surveillance in pregnancies complicated by GDM. Women whose blood glucose levels are well controlled by diet are at low risk for fetal complications. Limited antepartum fetal testing is performed in women with gestational diabetes as long as their fasting and 2-hour postprandial blood glucose levels remain within normal limits and they have no other risk factors. Women with hypertension, a history of a prior IUFD, or suspected macrosomia or those who require insulin for blood glucose control may have twice-weekly nonstress testing beginning at 32 weeks of gestation (Landon et al., 2007). In general, women with GDM can continue pregnancy until 40 weeks of gestation and the spontaneous onset of labor. However, fetal growth should be monitored carefully because the risk for macrosomia as the pregnancy approaches 40 weeks of gestation is apparently increased (Landon, et al.). Intrapartum During the labor and birth process, blood glucose levels are monitored hourly to maintain levels at 80 to 120 mg/dl (Moore & Catalano, 2009). Blood glucose levels within this range will decrease the incidence of neonatal hypoglycemia. Infusing regular insulin intravenously may be necessary during labor to maintain blood glucose levels within this range. IV fluids containing glucose are not commonly given during labor. Although gestational diabetes is not an indication for cesarean birth, this procedure may be necessary in the presence of preeclampsia or macrosomia. Postpartum Most women with GDM will return to normal glucose levels after childbirth. However, GDM is likely to recur in future pregnancies, and women with GDM are at significant risk for developing type 2 diabetes later in life. Assessment for carbohydrate intolerance with a 75 g oral glucose tolerance test should be performed at 6 to 12 weeks postpartum or after breastfeeding has stopped. Obesity is a major risk factor for the later development of diabetes. Women with a history of GDM, particularly those who are overweight, should be encouraged to make lifestyle changes that include weight loss and exercise to reduce this risk (Gilbert, 2007). Children born to women with GDM are also at risk for becoming obese in childhood or adolescence (Lindsay, 2006).

uterine atony

Relaxation of uterus; leads to postpartum hemorrhage

chorionic villus sampling (CVS)

Removal of fetal tissue from the placenta for genetic diagnostic studies

exchange transfusion

Replacement of 75% to 85% of circulating blood by withdrawal of the recipient's blood and injection of a donor's blood in equal amounts, the purposes of which are to prevent an accumulation of bilirubin in the blood above a dangerous level, to prevent the accumulation of other by-products of hemolysis in hemolytic disease, and to correct anemia and acidosis

mechanical ventilation

Respiratory support technique used to provide predetermined amount of oxygen; requires intubation

hydrops fetalis

Most severe expression of fetal hemolytic disorder with high mortality, a possible sequela to maternal Rh isoimmunization; infants exhibit gross edema (anasarca), cardiac decompensation, and profound pallor from anemia

superimposed preeclampsia

New-onset proteinuria in a woman with hypertension before 20 weeks of gestation, sudden increase in proteinuria if already present in early gestation, sudden increase in hypertension, or the development of HELLP syndrome

biophysical profile (BPP)

Noninvasive assessment of the fetus and its environment using ultrasonography and fetal monitoring; includes fetal breathing movements, gross body movements, fetal tone, reactive fetal heart rate, and qualitative amniotic fluid volume

magnetic resonance imaging (MRI)

Noninvasive nuclear procedure for imaging tissues with high fat and water content; in obstetrics, uses include evaluation of fetal structures, placenta, and amniotic fluid volume

-Non stress test- reactive is good. Non-reactive is NOT good

Nonstress Test The NST is the most widely applied technique for antepartum evaluation of the fetus. It is an ideal screening test and is the primary method of antepartum fetal assessment at most sites. The basis for the NST is that the normal fetus will produce characteristic heart rate patterns in response to fetal movement. In the term fetus, accelerations are associated with movement more than 85% of the time (Druzin et al., 2007). The most common reason for the absence of FHR accelerations is the quiet fetal sleep state. However, medications such as narcotics, barbiturates, and beta-blockers, maternal smoking, and the presence of fetal malformations can also adversely affect the test (Druzin; Gilbert, E.S., 2007). The NST can be performed easily and quickly in an outpatient setting because it is noninvasive, is relatively inexpensive, and has no known contraindications. Disadvantages include the requirement for twice-weekly testing and a high false-positive rate. The test also is slightly less sensitive in detecting fetal compromise than the CST or BPP (Tucker et al., 2009). Procedure The woman is seated in a reclining chair (or in semi-Fowler position) with a slight left tilt to optimize uterine perfusion and prevent supine hypotension. The FHR is recorded with a Doppler transducer, and a tocodynamometer is applied to detect uterine contractions or fetal movements. The tracing is observed for signs of fetal activity and a concurrent acceleration of FHR. If evidence of fetal movement is not apparent on the tracing, the woman may be asked to depress a button on a hand-held event marker connected to the monitor when she feels fetal movement. The movement is then noted on the tracing. Because almost all accelerations are accompanied by fetal movement, the movements need not be recorded for the test to be considered reactive. The test is usually completed within 20 to 30 minutes, but more time may be required if the fetus must be awakened from a sleep state. BOX 19-6 Indications for Electronic Fetal Monitoring Assessment Using the Nonstress Test and the Contraction Stress Test • Maternal diabetes mellitus • Chronic hypertension • Hypertensive disorders in pregnancy • Intrauterine growth restriction • Sickle cell disease • Maternal cyanotic heart disease • Postmaturity • History of previous stillbirth • Decreased fetal movement • Isoimmunization • Hyperthyroidism • Collagen disease • Chronic renal disease Caregivers sometimes suggest that the woman drink orange juice or be given glucose to increase her blood sugar level and thereby stimulate fetal movements. This practice is common; however, research has not proven it to be effective (Druzin et al., 2007). Vibroacoustic stimulation is often used to stimulate fetal activity if the initial NST result is nonreactive and thus hopefully shortens the time required to complete the test (Druzin et al.). Interpretation NST results are either reactive (Fig. 19-10) or nonreactive (Fig. 19-11). Box 19-7 lists criteria for both results. A nonreactive test requires further evaluation. The testing period is often extended, usually for an additional 20 minutes, with the expectation that the fetal sleep state will change and the test will become reactive. During this time, vibroacoustic stimulation (see later discussion) may be used to stimulate fetal activity. If the test does not meet the criteria after 40 minutes, a CST or BPP will usually be performed. Once NST testing is initiated, it is usually repeated once or twice weekly for the remainder of the pregnancy (Druzin et al., 2007; Tucker et al., 2009). BOX 19-7 Interpretation of the Nonstress Test Reactive test: Two accelerations in a 20-minute period, each lasting at least 15 seconds and peaking at least 15 beats per minute above the baseline (Before 32 weeks of gestation, an acceleration is defined as an increase of at least 10 beats per minute and lasting at least 10 seconds.) Nonreactive test: A test that does not produce two or more qualifying accelerations in a 20-minute period. Vibroacoustic stimulation Vibroacoustic stimulation (also called the acoustic stimulation test) is another method of testing antepartum FHR response. This test is generally performed in conjunction with the NST and uses a combination of sound and vibration to stimulate the fetus. Whether the acoustic or the vibratory component alters the fetal state is unclear. The test takes approximately 15 minutes to complete, with the fetus monitored for 5 to 10 minutes before stimulation to obtain a baseline FHR. If the fetal baseline pattern is nonreactive, the sound source (usually a laryngeal stimulator) is then activated for 3 seconds on the maternal abdomen over the fetal head. Monitoring continues for another 5 minutes, after which the monitor tracing is assessed. The desired result is a reactive NST. The accelerations produced may have a significant increase in duration (Fig. 19-12). The test may be repeated at 1-minute intervals up to three times when no response is noted. Further evaluation is needed with BPP or CST if the pattern is still nonreactive (Druzin et al., 2007).

extracorporeal membrane oxygenation (ECMO)

Oxygenation of blood external to body using cardiopulmonary bypass and a membrane oxygenator, used primarily for newborns with refractory respiratory failure or meconium aspiration syndrome

-puerperal infection- signs and symptoms

POSTPARTUM INFECTIONS Postpartum infection, or puerperal infection, is any clinical infection of the genital canal that occurs within 28 days after miscarriage, induced abortion, or childbirth. The definition used in the United States continues to be the presence of a fever of 38° C or more on 2 successive days of the first 10 postpartum days (not counting the first 24 hours after birth) (Cunningham et al., 2005). Puerperal infection is probably the major cause of maternal morbidity and mortality throughout the world; endometritis is the most common cause. In the United States, it occurs after approximately 2% of vaginal births and 10% to 15% of cesarean births (Katz, 2007b). Other common postpartum infections include wound infections, mastitis, urinary tract infections (UTIs), and respiratory tract infections. The most common infecting organisms are the numerous streptococcal and anaerobic organisms. Staphylococcus aureus, gonococci, coliform bacteria, and clostridia are less common but serious pathogenic organisms that also cause puerperal infection. Postpartum infections are common in women who have concurrent medical or immunosuppressive conditions or who had a cesarean or operative vaginal birth. Intrapartal factors such as prolonged rupture of membranes, prolonged labor, and internal maternal or fetal monitoring also increase the risk of infection (Duff, 2007). Factors that predispose the woman to postpartum infection are listed in Box 23-4. BOX 23-4 Predisposing Factors for Postpartum Infection PRECONCEPTION OR ANTEPARTAL FACTORS • History of previous venous thrombosis, urinary tract infection, mastitis, pneumonia • Diabetes mellitus • Alcoholism • Drug abuse • Immunosuppression • Anemia • Malnutrition INTRAPARTAL FACTORS • Cesarean birth • Operative vaginal birth • Prolonged rupture of membranes • Chorioamnionitis • Prolonged labor • Bladder catheterization • Internal fetal or uterine pressure monitoring • Multiple vaginal examinations after rupture of membranes • Epidural anesthesia • Retained placental fragments • Postpartum hemorrhage • Episiotomy or lacerations • Hematomas

-postpartum depression (PPD)

POSTPARTUM PSYCHOLOGIC COMPLICATIONS ▪ Mental health disorders have implications for the mother, the newborn, and the entire family. Such conditions can interfere with attachment to the newborn and family integration, and some may threaten the safety and well-being of the mother, newborn, and other children. Mood Disorders Mood disorders are the predominant mental health disorder in the postpartum period, typically occurring within 4 weeks of childbirth (American Psychiatric Association [APA], 2000). Many women experience a mild depression, or "baby blues," after the birth of a child. Others can have more serious depressions that can eventually incapacitate them to the point of being unable to care for themselves or their babies. Nurses are strategically positioned to offer anticipatory guidance, to assess the mental health of new mothers, to offer therapeutic interventions, and to refer when necessary. Failure to do so may result in tragic consequences. The Diagnostic and Statistical Manual of Mental Disorders contains the official guidelines for the assessment and diagnosis of psychiatric illness (APA, 2000). However, specific criteria for postpartum depression (PPD) are not listed. Instead, postpartum onset can be specified for any mood disorder either without psychotic features (i.e., PPD) or with psychotic features (i.e., postpartum psychosis) if the onset occurs within 4 weeks of childbirth (APA). Etiology and risk factors The cause of PPD may be biologic, psychologic, situational, or multifactorial. It affects 13% of women worldwide. In a comprehensive review of the literature, Bina (2008) found that cultural practices could positively or negatively affect the development of PPD. A personal history or a family history of mood disorder, mood and anxiety symptoms in the antepartal period, as well as postpartum blues also increases the risk for PPD (APA, 2000; Milgrom et al., 2008). Beck (2008a, 2008b) published an integrative review of 141 studies of what nurse researchers internationally have contributed to the state of the science on postpartum depression. One aspect was in identifying risk factors. Beck described at least five instruments that have been developed since 1990 to assess risk factors or symptoms of PPD. The most common risk factors that have been identified are discussed in a later section. Postpartum depression without psychotic features PPD is an intense and pervasive sadness with severe and labile mood swings and is more serious and persistent than postpartum blues. Intense fears, anger, anxiety, and despondency that persist past the baby's first few weeks are not a normal part of postpartum blues. Occurring in approximately 10% to 15% of new mothers, these symptoms rarely disappear without outside help. Approximately 50% of these mothers do not seek help from any source (Dennis & Chung-Lee, 2006). The occurrence of PPD among teenage mothers is approximately 50% more than that for older mothers (Driscoll, 2006). Young mothers (younger than 20 years) and those with a high school education or less are less likely to seek help and have higher rates of PPD than other women (Mayberry, Horowitz, & Declercq, 2007). The symptoms of postpartum major depression do not differ from the symptoms of nonpostpartum mood disorders except that the mother's ruminations of guilt and inadequacy feed her worries about being an incompetent and inadequate parent. In PPD the woman may have odd food cravings (often sweet desserts) and binges with abnormal appetite and weight gain. New mothers report an increased yearning for sleep, sleeping heavily but awakening instantly with any infant noise, and an inability to go back to sleep after infant feedings. A distinguishing feature of PPD is irritability. These episodes of irritability may flare up with little provocation, and they may sometimes escalate to violent outbursts or dissolve into uncontrollable sobbing. Many of these outbursts are directed against significant others ("He never helps me.") or the baby ("She cries all the time and I feel like hitting her."). Women with postpartum major depressive episodes often have severe anxiety, panic attacks, and spontaneous crying long after the usual duration of baby blues. Many women feel especially guilty about having depressive feelings at a time when they believe they should be happy. They may be reluctant to discuss their symptoms or their negative feelings toward the infant. A prominent feature of PPD is rejection of the infant, often caused by abnormal jealousy (APA, 2000). The mother may be obsessed by the notion that the offspring may take her place in her partner's affections. Attitudes toward the infant may include disinterest, annoyance with care demands, and blaming because of her lack of maternal feeling. When observed, she may appear awkward in her responses to the baby. Obsessive thoughts about harming the child are very frightening to her. In many instances, she does not share these thoughts because of embarrassment, but when she does, other family members become very frightened. Medical management The natural course is one of gradual improvement over the 6 months after birth. Supportive treatment alone is not efficacious for major PPD. Pharmacologic intervention is needed in most instances. Treatment options include antidepressants, anxiolytic agents, and electroconvulsive therapy (ECT). Alternative therapies such as herbs, dietary supplements, massage, aromatherapy, and acupuncture may be helpful (Weier & Beal, 2004). Psychotherapy focuses on the mother's fears and concerns regarding her new responsibilities and roles, as well as monitoring for suicidal or homicidal thoughts. For some women, hospitalization is necessary. Postpartum depression with psychotic features Postpartum psychosis is a syndrome most often characterized by depression (as described previously), delusions, and thoughts by the mother of harming either the infant or herself (Kaplan & Sadock, 2005). A postpartum mood disorder with psychotic features occurs in 1 to 2 per 1000 births and may occur more often in primiparas (Kaplan & Sadock). Once a woman has had one postpartum episode with psychotic features, a 30% to 50% likelihood of recurrence exists with each subsequent birth (APA, 2000). Symptoms often begin within days after the birth, although the mean time to onset is 2 to 3 weeks and almost always within 8 weeks of birth (Kaplan & Sadock, 2005). Characteristically, the woman begins to complain of fatigue, insomnia, and restlessness and may have episodes of tearfulness and emotional lability. Complaints regarding the inability to move, stand, or work are also common. Later, suspiciousness, confusion, incoherence, irrational statements, and obsessive concerns about the baby's health and welfare may be present (Kaplan & Sadock). Delusions may be present in 50% of all women with this disorder and hallucinations in approximately 25%. Auditory hallucinations that command the mother to kill the infant can also occur in severe cases. When delusions are present, they are often related to the infant. The mother may think the infant is possessed by the devil, has special powers, or is destined for a terrible fate (APA, 2000). Grossly disorganized behavior may be exhibited as a disinterest in the infant or an inability to provide care. Some women will insist that something is wrong with the baby or accuse nurses or family members of hurting or poisoning him or her. NURSING ALERT Nurses are advised to be alert for mothers who are agitated, overactive, confused, complaining, or suspicious. A specific illness included in depression with psychotic features is bipolar disorder. This mood disorder is preceded or accompanied by manic episodes, characterized by elevated, expansive, or irritable moods. Clinical manifestations of a manic episode include at least three of the following symptoms that have been significantly present for at least 1 week: grandiosity, decreased need for sleep, pressured speech, flight of ideas, distractibility, psychomotor agitation, and excessive involvement in pleasurable activities without regard for negative consequences (APA, 2000). Because these women are hyperactive, they may not take the time to eat or sleep, which leads to inadequate nutrition, dehydration, and sleep deprivation. While in a manic state, mothers will need constant supervision when caring for their infants. Mostly, they will be too preoccupied to provide child care. Medical management A favorable outcome is associated with a good premorbid adjustment (before the onset of the disorder) and a supportive family network (Kaplan & Sadock, 2005). Because mood disorders are usually episodic, women may experience another episode of symptoms within a year or two of the birth. Postpartum psychosis is a psychiatric emergency, and the mother will probably need psychiatric hospitalization. Antipsychotics and mood stabilizers such as lithium are the treatments of choice. If the mother is breastfeeding, some sources advise caution while prescribing some agents (ACOG, 2008; Newport & Stowe, 2006.). Antipsychotics and lithium should be avoided in breastfeeding mothers, but other mood stabilizers may be compatible with breastfeeding (see later discussion). Having contact with her baby is usually advantageous for the mother if she so desires, but visits must be closely supervised. Psychotherapy is indicated after the period of acute psychosis is past. CARE MANAGEMENT ▪ Even though the prevalence of PPD is fairly well established, it often remains undetected because women are hesitant to report symptoms of depression even to their own health care providers (McQueen, Montgomery, Lappan-Gracon, Evans, & Hunter, 2008). The following discussion identifies ways to assess for symptoms of PPD and describes the treatment options (see the Nursing Process box: Postpartum Depression). NURSING PROCESS: Postpartum Depression ASSESSMENT • Listen actively and demonstrate a caring attitude because women may not volunteer unsolicited information about their depression. • Observe for signs of depression. • Ask appropriate questions to determine moods, appetite, sleep, energy and fatigue levels, and ability to concentrate. An example of how to initiate a conversation is, "Now that you have had your baby, how are things going for you?" • Use a screening tool to assess whether the depressive symptoms have progressed from postpartum blues to postpartum depression (PPD) (see text). • If depression is identified, ask if the mother has thought about hurting herself or the baby. Postpartum Depression Screening Tools Examples of screening tools are the Edinburgh Postnatal Depression Scale (EPDS), the Postpartum Depression Predictors Inventory (PDPI), and the Postpartum Depression Screening Scale (PDSS). The EPDS is a self-report assessment designed specifically to identify women experiencing PPD. It has been used and validated in studies in numerous cultures and is viewed as a valid screening tool for PPD (Lintner & Gray, 2006). The assessment tool asks the woman to respond to 10 statements about the common symptoms of depression. The woman is asked to choose the response that is closest to describing how she has felt for the past week (Cox, Holden, & Sagovsky, 1987). Through focused research over at least a decade, Beck has developed and continues to refine the PDPI (PDPI-R [Revised]) (Beck, 2001, 2002) and the PDSS (Beck & Gable, 2002). The PDPI-R consists of 13 risk factors related to PPD. The PDSS is a 35-item Likert response scale that assesses for seven dimensions of depression: sleeping or eating disturbances, anxiety or insecurity, emotional lability, mental confusion, loss of self, guilt or shame, and suicidal thoughts (Beck, 2008a). Both published tools are designed to be used by nurses and other health care providers to elicit information from the woman during an interview to assess risk. Areas assessed include the predictors of depression as listed in Box 23-6. In addition, a simple two-item tool has been developed that nurses can use that has been shown to be effective in identifying women at risk for PPD. Ask, "Are you sad and depressed?" and "Have you had a loss of pleasurable activities?" An affirmative answer to both questions suggests that depression is likely (Jesse & Graham, 2005). If any initial assessment reveals some question that the woman might be depressed, a formal screening is helpful in determining the urgency of the referral and the type of provider. Nursing Interventions Nursing care on the postpartum unit The postpartum nurse must observe the new mother closely for any signs of tearfulness and conduct further assessments as necessary. PPD must be discussed by nurses to prepare new parents for potential problems in the postpartum period (see Patient Instructions for Self-Management box: Coping with Postpartum Blues, p. 427). The family must be able to recognize the symptoms and know where to go for help. Written materials that explain what the woman can do to prevent depression could be used as part of discharge planning. PATIENT INSTRUCTIONS FOR SELF-MANAGEMENT Activities to Prevent Postpartum Depression • Share knowledge about postpartum emotional problems with close family and friends. • Take care of yourself: Eat a balanced diet, exercise on a regular basis, and get enough sleep. Ask someone to take care of the baby so that you can get a full night's sleep. • Share your feelings with someone close to you; do not isolate yourself at home. • Do not overcommit yourself or feel as though you need to be a superwoman. • Do not place unrealistic expectations on yourself. • Do not be ashamed of having emotional problems after your baby is born—it happens to approximately 15% of women. Mothers are often discharged from the hospital before the blues or depression occurs. If the postpartum nurse is concerned about the mother, a mental health consultation should be requested before the mother leaves the hospital. Routine instructions regarding PPD should be given to the person who comes to take the woman home; for example, "If you notice that your wife (or daughter) is upset or crying a lot, please call the postpartum care provider immediately—don't wait for the routine postpartum appointment." NURSING ALERT Because the newborn may be scheduled for a checkup before the mother's 6-week checkup, nurses in well-baby clinics or pediatrician offices should be alert for signs of PPD in new mothers and be knowledgeable about community referral resources. BOX 23-6 Risk Factors for Postpartum Depression • Low self-esteem • Stress of child care • Prenatal anxiety • Life stress • Lack of social support • Marital relationship problems • History of depression • "Difficult" infant temperament • Postpartum blues • Single status • Low socioeconomic status • Unplanned or unwanted pregnancy Nursing care in the home and community Postpartum home visits can reduce the incidence of or complications from depression. A brief home visit or telephone call at least once a week until the new mother returns for her postpartum visit may save the life of a mother and her infant; however, these contacts may not be feasible or available. Supervision of the mother with emotional complications may become a prime concern. Because depression can greatly interfere with her mothering functions, family and friends may need to participate in the infant's care. The extended family and friends can use this time to determine what they can do to help, and the nurse can work with them to ensure adequate supervision of the woman and their understanding of the woman's mental illness. When the woman has PPD, a partner often reacts with confusion, shock, denial, and anger and feels neglected and blamed. Both the woman and her partner need an opportunity to express their needs, fears, thoughts, and feelings in a nonjudgmental environment. The nurse can talk with the woman about how her condition is hard for her partner too and that he is probably very worried about her. Men often withdraw or criticize when they are deeply worried about their significant others. The nurse can provide opportunities for the partner to verbalize feelings and concerns, help the partner identify positive coping strategies, and be a source of encouragement for the partner to continue supporting the woman. Even if the mother is severely depressed, hospitalization can be avoided if adequate resources can be mobilized to ensure safety for both mother and infant. The nurse in home health care will need to make frequent telephone calls or home visits to do assessment and counseling. Community resources that may be helpful are temporary child care or foster care, homemaker service, Meals on Wheels, parenting guidance centers, mother's-day-out programs, and telephone support groups or other support programs. Referral Women with moderate to severe PPD should be referred to a mental health therapist, such as an advanced practice psychiatric nurse, for evaluation and therapy to prevent the effects that PPD can have on the woman and on her relationships with her partner, baby, and other children (Lintner & Gray, 2006). Inpatient psychiatric hospitalization may be necessary. This decision is made when the safety needs of the mother or children are threatened. Providing safety When depression is suspected, the nurse asks, "Have you thought about hurting yourself?" If delusional thinking about the baby is suspected, the nurse asks, "Have you thought about hurting your baby?" Four criteria can be used to assess the seriousness of a suicidal plan: (1) method, (2) availability, (3) specificity, and (4) lethality. Has the woman specified a method? Is the method of choice available? How specific is the plan? If the method is concrete and detailed, with access to it right at hand, the suicide risk increases. How lethal is the method? The most lethal method is shooting, with hanging a close second. The least lethal is slashing one's wrists. Medication overdose can also be used to cause death. NURSING ALERT Suicidal thoughts or attempts are one of the most serious symptoms of PPD and require immediate assessment and intervention (Lintner & Gray, 2006). Psychiatric hospitalization Women with postpartum psychosis are a psychiatric emergency and must be referred immediately to a psychiatrist who is experienced in working with women with PPD, who can prescribe medication and other forms of therapy, and who can assess the need for hospitalization. LEGAL TIP Legal Commitment If a woman with PPD is experiencing active suicidal ideation or harmful delusions about the baby and is unwilling to seek treatment, legal intervention may be necessary to commit the woman to a psychiatric inpatient setting for treatment. If the infant is allowed in the hospital psychiatric setting, the reintroduction of the baby to the mother can occur at the mother's own pace. A schedule is set for increasing the number of hours the mother cares for the baby over several days, culminating in the infant staying overnight in the mother's room. This method allows the mother to experience meeting the infant's needs and giving up sleep for the baby, a difficult situation for new mothers even under ideal conditions. The mother's readiness for discharge and caring for the baby is assessed. Her interactions with her baby are also carefully supervised and guided. A postpartum nurse is often asked to assist the psychiatric nursing staff in assessment of the mother-infant interactions. Psychotropic medications PPD is usually treated with antidepressant medications. If the woman with PPD is not breastfeeding, antidepressants can be prescribed without special precautions. The commonly used antidepressant drugs are often divided into four groups: selective serotonin reuptake inhibitors (SSRIs), heterocyclics (including the tricyclic antidepressants [TCAs]), monoamine oxidase inhibitors (MAOIs), and other antidepressant agents not in the above classifications (ACOG, 2008) (Table 23-1). The SSRIs are prescribed more frequently today than other groups of antidepressant medications. They are relatively safe and carry fewer side effects than TCAs. The most frequent side effects with the SSRIs are gastrointestinal disturbances (nausea, diarrhea), headache, and insomnia. In approximately one third of patients the SSRIs reduce libido, arousal, or orgasmic function (Keltner, 2007a). TCAs cause many central nervous system (CNS) and peripheral nervous system (PNS) side effects. A common CNS effect is sedation, which can easily interfere with mothers caring for their babies. A mother could fall asleep while holding the baby and drop him or her, or she could have trouble getting fully awake during the night to care for the baby. Other side effects include weight gain, tremors, grand mal seizures, nightmares, agitation or mania, and extrapyramidal side effects. Anticholinergic side effects include dry mouth, blurred vision (usually temporary), difficulty voiding, constipation, sweating, and orgasm difficulty (Keltner, 2007a). Hypertensive crisis is the main reason that MAOIs are not prescribed more frequently than other psychotropic medications. The woman should be taught to watch for signs of hypertensive crisis—throbbing, occipital headache, stiff neck, chills, nausea, flushing, retroorbital pain, apprehension, pallor, sweating, chest pain, and palpitations (Keltner, 2007a). This crisis is brought o

abruptio placentae

Partial or complete premature separation of a normally implanted placenta

disseminated intravascular coagulation (DIC)

Pathologic form of coagulation in which clotting factors are consumed to such an extent that generalized bleeding can occur; associated with abruptio placentae, eclampsia, intrauterine fetal demise, amniotic fluid embolism, and hemorrhage

trial of labor (TOL)

Period of observation to determine whether a laboring woman is likely to be successful in progressing to a vaginal birth

euglycemia

Pertaining to a normal blood glucose level; also called normoglycemia

barotrauma

Physical injury resulting from changing air pressure, often associated with ventilatory assistance in preterm infants

grief

Physical, emotional, social, and cognitive response to a loss such as the death of a loved one

placenta previa

Placenta that is abnormally implanted in the thin, lower uterine segment. The condition is further classified as complete placenta previa, marginal placenta previa, or low-lying placenta according to gestational age and placental location in relation to the internal cervical os.

external cephalic version (ECV)

Turning of the fetus to a vertex presentation by external exertion of pressure on the fetus through the maternal abdomen

-pp hemorrhage- causes and treatments- drugs used such as methergine, hemabate, pitocon and cytotec

Postpartum hemorrhage (PPH) continues to be a leading cause of maternal morbidity and mortality in the United States and worldwide (American College of Obstetricians and Gynecologists [ACOG], 2006; Johnson, Gregory, & Niebyl, 2007). It is a life-threatening event that can occur with little warning and is often unrecognized until the mother has profound symptoms. PPH has been traditionally defined as the loss of more than 500 ml of blood after vaginal birth and 1000 ml after cesarean birth. A 10% change in hematocrit between admission for labor and postpartum or the need for erythrocyte transfusion also has been used to define PPH (Francois & Foley, 2007). However, defining PPH is not a clear-cut issue. The diagnosis is often based on subjective observations, with blood loss often being underestimated by as much as 50% (Cunningham, Leveno, Bloom, Hauth, Gilstrap, & Wenstrom, 2005). Traditionally, PPH has been classified as early or late with respect to the birth. Early, acute, or primary PPH occurs within 24 hours of the birth. Late or secondary PPH occurs after 24 hours and up to 6 to 12 weeks postpartum (ACOG, 2006; Francois & Foley, 2007). Today's health care environment encourages shortened stays after birth, thereby increasing the potential for acute episodes of PPH to occur outside the traditional hospital or birth center setting. Etiology and Risk Factors Considering the problem of excessive bleeding with reference to the stages of labor is helpful. From birth of the infant until separation of the placenta the character and quantity of blood passed may suggest excessive bleeding. For example, dark blood is probably of venous origin, perhaps from varices or superficial lacerations of the birth canal. Bright blood is arterial and may indicate deep lacerations of the cervix. Spurts of blood with clots may indicate partial placental separation. Failure of blood to clot or remain clotted indicates a pathologic condition or coagulopathy such as disseminated intravascular coagulation (DIC) (Francois & Foley, 2007). Excessive bleeding may occur during the period from the separation of the placenta to its expulsion or removal. Commonly, such excessive bleeding is the result of incomplete placental separation, undue manipulation of the fundus, or excessive traction on the cord. After the placenta has been expelled or removed, persistent or excessive blood loss is usually the result of atony of the uterus or inversion of the uterus into the vagina. Late PPH may be the result of subinvolution of the uterus, endometritis, or retained placental fragments (Francois & Foley, 2007). BOX 23-1 Risk Factors for Postpartum Hemorrhage • Uterine atony • Overdistended uterus • Large fetus • Multiple fetuses • Hydramnios • Distention with clots • Anesthesia and analgesia • Conduction anesthesia • Previous history of uterine atony • High parity • Prolonged labor, oxytocin-induced labor • Trauma during labor and birth • Forceps-assisted birth • Vacuum-assisted birth • Cesarean birth • Unrepaired lacerations of the birth canal • Retained placental fragments • Ruptured uterus • Inversion of the uterus • Placenta accreta, increta, percreta • Coagulation disorders • Placental abruption • Placenta previa • Manual removal of a retained placenta • Magnesium sulfate administration during labor or postpartum period • Chorioamnionitis • Uterine subinvolution Drugs Used to Manage Postpartum Hemorrhage DRUG: Oxytocin (Pitocin) ACTION: Contraction of uterus; decreases bleeding SIDE EFFECTS: Infrequent: water intoxication, nausea and vomiting CONTRAINDICATIONS: None for PPH DOSAGE AND ROUTE: 10 to 40 units/L diluted in lactated Ringer's solution or normal saline at 125 to 200 milliunits/min IV; or 10 to 20 units IM NURSING CONSIDERATIONS: Continue to monitor vaginal bleeding and uterine tone DRUG: Methylergonovine (Methergine)* ACTION: Contraction of uterus SIDE EFFECTS: Hypertension, nausea, vomiting, headache CONTRAINDICATIONS: Hypertension, cardiac disease DOSAGE AND ROUTE: 0.2 mg IM every 2-4 hr up to five doses; may also be given intrauterine or orally NURSING CONSIDERATIONS: Check blood pressure before giving, and do not give if >140/90 mm Hg; continue monitoring vaginal bleeding and uterine tone DRUG: 15-Methylprostaglandin F2α (Prostin/15m; Carboprost, Hemabate) ACTION: Contraction of uterus SIDE EFFECTS: Headache, nausea and vomiting, fever, tachycardia, hypertension, diarrhea CONTRAINDICATIONS: Avoid with asthma or hypertension DOSAGE AND ROUTE: 0.25 mg IM or intrauterine every 15-90 min up to eight doses NURSING CONSIDERATIONS: Continue to monitor vaginal bleeding and uterine tone DRUG: Dinoprostone (Prostin E2) ACTION: Contraction of uterus SIDE EFFECTS: Headache, nausea and vomiting, fever, chills, diarrhea CONTRAINDICATIONS: Avoid with hypotension DOSAGE AND ROUTE: 20 mg vaginal or rectal suppository every 2 hr NURSING CONSIDERATIONS: Continue to monitor vaginal bleeding and uterine tone DRUG: Misoprostol (Cytotec) ACTION: Contraction of uterus SIDE EFFECTS: Headache, nausea and vomiting, diarrhea CONTRAINDICATIONS: History of allergy to prostaglandins DOSAGE AND ROUTE: 800 to 1000 mcg rectally once NURSING CONSIDERATIONS: Continue to monitor vaginal bleeding and uterine tone

endometritis

Postpartum uterine infection, often beginning at the site of the placental implantation

-pre-eclampsia, eclampsia

Preeclampsia Preeclampsia is a pregnancy-specific condition in which hypertension and proteinuria develop after 20 weeks of gestation in a previously normotensive woman. A significant contributor to maternal and perinatal morbidity and mortality, preeclampsia complicates approximately 3% to 7% of all pregnancies (American Academy of Pediatrics [AAP] & ACOG, 2007). Preeclampsia is a vasospastic, systemic disorder and is usually categorized as mild or severe for purposes of management (ACOG, 2002; Working Group, 2000). Table 21-2 lists criteria for mild and severe preeclampsia, and Table 21-3 gives common laboratory changes that occur in mild and severe preeclampsia. Proteinuria is defined as a concentration at or above 30 mg/dl (≥1+ on dipstick measurement) or more in at least two random urine specimens collected at least 6 hours apart with no evidence of urinary tract infection. In a 24-hour specimen, proteinuria is defined as a concentration at or above 300 mg/24 hours. Because of the discrepancy between random protein determinations the diagnosis of proteinuria should be based on a 24-hour urine collection if possible or a timed collection corrected for creatinine excretion if a 24-hour specimen is not feasible (ACOG, 2002; Longo, Dola, & Pridjian, 2003). To ensure accurate results, proteinuria should be determined using only a urine specimen that has been collected either by catheterization or by a thorough clean-catch technique. Eclampsia Eclampsia is the onset of seizure activity or coma in a woman with preeclampsia but with no history of a preexisting abnormality that can result in seizure activity (ACOG, 2002). The initial presentation of eclampsia varies, with one third of the women developing eclampsia during the pregnancy, one third during labor, and one third within 72 hours postpartum (Emery, 2005). Chronic hypertension Chronic hypertension is defined as hypertension that occurs before the pregnancy or is diagnosed before the twentieth week of gestation. Hypertension initially diagnosed during pregnancy that persists longer than 6 weeks postpartum is also classified as chronic hypertension (Sibai, 2007). Other authorities believe that a diagnosis of chronic hypertension can be made only if the BP has not returned to normal levels by 12 weeks after birth (Roberts & Funai, 2009). Chronic hypertension with superimposed preeclampsia Women with chronic hypertension may develop superimposed preeclampsia, which increases the morbidity for both the mother and the fetus. A diagnosis of superimposed preeclampsia is made with the following findings (Sibai, 2007): • In women with hypertension before 20 weeks of gestation --- New-onset proteinuria (≥0.5 g protein in a 24-hour collection) • In women with both hypertension and proteinuria before 20 weeks of gestation --- Significant increase in hypertension, plus one of the following: ~~~~ New onset of symptoms ~~~~ Thrombocytopenia ~~~~ Elevated liver enzymes Preeclampsia Etiology Preeclampsia is a condition unique to human pregnancy; signs and symptoms develop only during pregnancy and disappear quickly after birth of the fetus and placenta. The cause of preeclampsia is not known. Although preeclampsia is generally a disease of primigravidas, its cause may not be the same for all women. For example, the pathogenesis for a healthy nulliparous woman who develops mild preeclampsia near term or in labor may be very different than that of the woman who has preexisting vascular disease or diabetes, a multifetal pregnancy, or who develops severe preeclampsia earlier in the pregnancy (Sibai, 2007). Box 21-2 lists risk factors for the development of preeclampsia. Many theories have been suggested to explain the cause of preeclampsia. Current theories that are still being considered include abnormal trophoblast invasion, coagulation abnormalities, vascular endothelial damage, cardiovascular maladaptation, and dietary deficiencies or excesses. Immunologic factors and genetic predisposition may also play an important role (Sibai, 2007). Pathophysiology Preeclampsia can progress along a continuum from mild to severe preeclampsia to eclampsia. Current thought is that the pathologic changes that occur in the woman with preeclampsia are caused by disruptions in placental perfusion and endothelial cell dysfunction (Gilbert, 2007; Peters, 2008). These pathologic changes are present long before the clinical diagnosis of preeclampsia is made (Roberts & Funai, 2009). Normally in pregnancy the spiral arteries in the uterus widen from thick-walled muscular vessels to thinner, saclike vessels with much larger diameters. This change increases the capacity of the vessels, allowing them to handle the increased blood volume of pregnancy. Because this vascular remodeling does not occur or only partially develops in women with preeclampsia, decreased placental perfusion and hypoxia result (Peters). Placental ischemia is thought to cause endothelial cell dysfunction by stimulating the release of a substance that is toxic to endothelial cells. This anomaly causes generalized vasospasm, which results in poor tissue perfusion in all organ systems, increased peripheral resistance and BP, and increased endothelial cell permeability, leading to intravascular protein and fluid loss and ultimately to less plasma volume. The main pathogenic factor is not an increase in BP but poor perfusion as a result of vasospasm and reduced plasma volume (Fig. 21-1) (Gilbert; Peters; Roberts & Funai). Fig. 21-2 demonstrates how endothelial cell dysfunction causes many of the common signs and symptoms of preeclampsia. BOX 21-2 Risk Factors Associated with the Development of Preeclampsia • Nulliparity • Family history of preeclampsia • Obesity • Multifetal gestation • Preeclampsia in previous pregnancy • Poor outcome in previous pregnancy: • Intrauterine growth restriction • Placental abruption • Fetal death • Preexisting medical—genetic conditions: • Chronic hypertension • Renal disease • Type 1 (insulin-dependent) diabetes mellitus • Thrombophilias: ----- Antiphospholipid antibody syndrome ----- Protein C, protein S, antithrombin deficiency Reduced kidney perfusion decreases the glomerular filtration rate and leads to degenerative glomerular changes and possibly oliguria. Pathologic changes in the endothelial cells of the glomeruli (glomerular endotheliosis) are uniquely characteristic of preeclampsia. Protein, primarily albumin, is lost in the urine. Uric acid clearance is decreased. Serum uric acid levels, however, increase. Sodium and water are retained. Acute tubular necrosis and renal failure may occur (Gilbert, 2007; Peters, 2008; Roberts & Funai, 2009). Plasma colloid osmotic pressure decreases as serum albumin levels decrease. Intravascular volume is reduced as fluid moves out of the intravascular compartment, resulting in hemoconcentration, increased blood viscosity, and tissue edema. The hematocrit value increases as fluid leaves the intravascular space. In severe preeclampsia, blood volume may decrease to or below nonpregnancy levels; severe edema develops, and rapid weight gain is seen. Arteriolar vasospasm can cause endothelial damage and contribute to an increased capillary permeability, which increases edema and further decreases intravascular volume, predisposing the woman with preeclampsia to pulmonary edema (see Fig. 21-2) (Gilbert, 2007; Roberts & Funai, 2009). Decreased liver perfusion results in impaired liver function. Liver enzyme levels increase in the wake of liver damage. If hepatic edema and subcapsular hemorrhage develop, the woman may complain of epigastric or right upper quadrant pain. Rupture of a subcapsular hematoma is a life-threatening complication and a surgical emergency (Gilbert, 2007) (see Fig. 21-2). Arteriolar vasospasms and decreased blood flow to the retina lead to visual symptoms such as scotomata (blind spots) and blurring. Neurologic complications associated with preeclampsia include cerebral edema and hemorrhages and increased central nervous system (CNS) irritability, the last of which causes headache, hyperreflexia, positive ankle clonus, and seizures (Gilbert, 2007; Longo et al., 2003; Roberts & Funai, 2009) (see Fig. 21-2). Preeclampsia contributes significantly to restriction of fetal growth and the incidence of placental abruption (Hull & Resnik, 2009; Peters, 2008). Impaired placental perfusion leads to early degenerative aging of the placenta. The rate of fetal complications is directly related to the severity of the disease (Longo et al., 2003).

postterm pregnancy

Pregnancy that extends past 42 weeks of gestation

preterm premature rupture of membranes (preterm PROM)

Premature rupture of membranes that occurs before 37 weeks of gestation

-Premature separation of placenta (abruptio placentae)

Premature separation of the placenta, or abruptio placentae, is the detachment of part or all of a normally implanted placenta from the uterus (Fig. 21-12). Separation occurs in the area of the decidua basalis after 20 weeks of gestation and before the birth of the infant. Incidence and etiology Premature separation of the placenta is a serious complication that accounts for significant maternal and fetal morbidity and mortality. Approximately 1 in 75 to 1 in 226 of pregnancies is complicated by abruptio placentae. The range in incidence likely reflects both variable criteria for diagnosis and an increased recognition of milder forms of abruption. Approximately one third of all antepartum bleeding is caused by placental abruption (Francois & Foley, 2007). Maternal hypertension, whether chronic or pregnancy related, is the most consistently identified risk factor for abruption. Cocaine use is also a risk factor because it causes vascular disruption in the placental bed. Blunt external abdominal trauma, most often the result of motor-vehicle accidents (MVAs) or maternal battering, is another frequent cause of placental abruption (Francois & Foley, 2007). Other risk factors include cigarette smoking, a history of abruption in a previous pregnancy and PROM (Cunningham et al., 2005; Hull & Resnik, 2009). Abruption is more likely to occur in twin gestations than in singletons (Francois & Foley). Women who have had two previous abruptions have a recurrence risk of 25% in the next pregnancy (Hull & Resnik). Classification The most common classification of placental abruption is according to type and severity. This classification system is summarized in Table 21-7. Clinical manifestations The separation may be partial or complete, or only the margin of the placenta may be involved. Bleeding from the placental site may dissect (separate) the membranes from the decidua basalis and flow out through the vagina (70%-80%), it may remain concealed (retroplacental hemorrhage) (10%-20%), or both (Fig. 21-13) (Francois & Foley, 2007; Gilbert, 2007). Clinical symptoms vary with degree of separation (see Table 21-7). If cesarean birth is performed, blood clots may be noted on entry into the uterus. Blood clot will often be attached to the posterior surface of the placenta (referred to as a retroplacental clot) (see Fig. 21-12). Classic symptoms of abruptio placentae include vaginal bleeding, abdominal pain, and uterine tenderness and contractions (Cunningham et al., 2005; Hull & Resnik, 2009). Bleeding may result in maternal hypovolemia (i.e., shock, oliguria, anuria) and coagulopathy. Mild to severe uterine hypertonicity is present. Pain is mild to severe and localized over one region of the uterus or diffuse over the uterus with a boardlike abdomen. Extensive myometrial bleeding damages the uterine muscle. If blood accumulates between the separated placenta and the uterine wall, it may produce a Couvelaire uterus. The uterus appears purple or blue, rather than its usual "bubble gum pink" color and contractility is lost. Shock may occur and is out of proportion to blood loss. Laboratory findings include a positive Apt test result (blood in the amniotic fluid), a decrease in hemoglobin and hematocrit levels, which may appear later, and a decrease in coagulation factor levels. Clotting defects (e.g., DIC) are present in approximately 40% of women who develop a large abruption (Francois & Foley, 2007). A Kleihauer-Betke (KB) test may be ordered to determine the presence of fetal-to-maternal bleeding (transplacental hemorrhage), although this test appears to have no value in the general workup of patients with abruption. The KB test may be useful to guide Rho(D) immune globulin therapy in Rh-negative women who have had an abruption (Hull & Resnik, 2009). Maternal and fetal outcomes The mother's prognosis depends on the extent of placental detachment, overall blood loss, degree of coagulopathy present and time between placental detachment and birth. Maternal complications are associated with the abruption or its treatment. Hemorrhage, hypovolemic shock, hypofibrinogenemia, and thrombocytopenia are associated with severe abruption. Renal failure and pituitary necrosis may result from ischemia. In rare cases, women who are Rh negative can become sensitized if fetal-to-maternal hemorrhage occurs and the fetal blood type is Rh positive. Placental abruption is associated with a perinatal mortality rate of 20% to 30%. If more than 50% of the placenta is involved, fetal death is likely to occur. Other fetal and neonatal risks include IUGR and preterm birth (Francois & Foley, 2007; Hull & Resnik, 2009). Risks for neurologic defects and death from sudden infant death syndrome are also increased in newborns following placental abruption (Cunningham et al., 2005; Francois & Foley). Diagnosis Placental abruption is primarily a clinical diagnosis. Although ultrasound can be used to rule out placenta previa, it cannot detect all cases of abruption. A retroplacental mass may be detected with ultrasonographic examination, but negative findings do not rule out a life-threatening abruption. In fact, at least 50% of abruptions cannot be identified on ultrasound (Hull & Resnik, 2009). Hypofibrinogenemia and evidence of DIC support the diagnosis, but many women with placental abruption do not develop coagulopathy. The diagnosis of abruption is confirmed after birth by visual inspection of the placenta. Adherent clot on the maternal surface of the placenta and depression of the underlying placental surface are usually present (see Fig. 21-12) (Francois & Foley, 2007; Gilbert, 2007). Abruptio placentae should be highly suspected in the woman with a sudden onset of intense, usually localized, uterine pain, with or without vaginal bleeding. Initial assessment is much the same as for placenta previa. Physical examination usually reveals abdominal pain, uterine tenderness, and contractions. The fundal height may be measured over time because an increasing fundal height indicates concealed bleeding. Approximately 60% of live fetuses exhibit nonreassuring FHR patterns, and elevated uterine resting tone may also be noted on the monitor tracing (Francois & Foley, 2007). Coagulopathy, as evidenced by abnormal clotting studies (fibrinogen, platelet count, PTT, fibrin split products), may be present if a large or complete abruption has occurred. Management Expectant management Management depends on the severity of blood loss and fetal maturity and status. If the abruption is mild and the fetus is less than 36 weeks of gestation and not in distress, expectant management may be implemented. The woman is hospitalized and observed closely for signs of bleeding and labor. The fetal status is also monitored with intermittent FHR monitoring and NSTs or BPPs until fetal maturity is determined or until the woman's condition deteriorates and immediate birth is indicated. Corticosteroids should be given to accelerate fetal lung maturity (Cunningham et al., 2005). Women who are Rh negative may be given Rho(D) immune globulin if fetal-to-maternal hemorrhage occurs. Active management Immediate birth is the management of choice if the fetus is at term gestation or if the bleeding is moderate to severe and the mother or fetus is in jeopardy. At least one large-bore (16- to 18-gauge) IV line should be started. Maternal vital signs are monitored frequently to observe for signs of declining hemodynamic status, such as increasing pulse rate and decreasing BP. Serial laboratory studies include hematocrit or hemoglobin determinations and clotting studies. Continuous EFM is mandatory. An indwelling Foley catheter is inserted for continuous assessment of urine output, an excellent indirect measure of maternal organ perfusion. Blood and fluid volume replacement may be necessary, along with administering blood products to correct any coagulation defects. Vaginal birth is usually feasible and is desirable, especially in cases of fetal death. Labor induction or augmentation may initiated so long as the mother and fetus are closely monitored for any evidence of compromise. Cesarean birth should be reserved for cases of fetal distress or other obstetric complications. Cesarean birth should not be attempted when the women has severe and uncorrected coagulopathy because it may result in uncontrollable bleeding (Francois & Foley, 2007). Nursing care of patients experiencing moderate to severe abruption is demanding because it requires constant close monitoring of the maternal and fetal condition. Information about abruptio placentae, including the cause, treatment, and expected outcome, is given to the woman and her family. Emotional support is also extremely important because the woman and her family may be experiencing fetal loss in addition to the woman's critical illness.

-pre-term labor- treatment options

Preterm labor is defined as cervical changes and uterine contractions occurring between 20 and 37 weeks of pregnancy. Preterm birth is any birth that occurs before the completion of 37 weeks of gestation (Iams & Romero, 2007). It occurs in approximately 12.8% of all live births, and the rate has been increasing for the last several years. Preterm birth is the major unsolved problem in perinatal medicine today (Iams, Romero, & Creasy, 2009). Approximately 75% of all preterm births are termed late preterm births because they occur between 34 and 36 weeks of gestation. Although these babies experience significant complications, the great majority of infant deaths and the most serious morbidity occur among the 16% of all preterm infants who are born before 32 weeks of gestation (Iams et al., 2009). See Chapters 18 and 24 for more discussion of problems related to late preterm birth. Predicting Spontaneous Preterm Labor and Birth A history of previous preterm birth, multiple gestation, bleeding after the first trimester of pregnancy, and a low maternal body mass index have been shown to be major risk factors for spontaneous preterm birth (Iams & Romero, 2007). Other risk factors include non-Caucasian race (especially African-American), low socioeconomic and educational status, living with chronic stress, smoking, substance abuse, physically demanding working conditions, and periodontal disease (Iams et al., 2009). A recent study found that perceived levels of stress measured at 28 weeks of gestation in African-American women experiencing preterm labor were higher in those who gave birth prematurely than in those whose pregnancies reached term (Gennaro, Shults, & Garry, 2008). In addition, the risk for preterm birth appears to be genetically related. Relatives of women who were born prematurely or gave birth prematurely also have an increased risk for spontaneous preterm birth (Iams et al., 2009). Many risk scoring systems have been developed in an attempt to determine which women might go into labor prematurely. None of these systems has been very successful, however, because at least 50% of all women who ultimately give birth prematurely have no identifiable risk factors (Iams & Romero, 2007; Iams et al., 2009). Therefore all women should be educated about prematurity not only in early pregnancy, but also in the preconceptional period. Biochemical markers Fetal fibronectin has been studied extensively and is currently marketed in the United States as a diagnostic test for preterm labor. Fetal fibronectin is a glycoprotein found in plasma produced during fetal life. The test is performed by collecting fluid from the woman's cervix and vagina using a swab during a vaginal examination. Fetal fibronectin is normally present in cervical and vaginal fluid early in pregnancy and then again in late pregnancy. The presence of fetal fibronectin during the late second and early third trimesters of pregnancy may be related to placental inflammation, which is thought to be one cause of spontaneous preterm labor. The presence of fetal fibronectin is not very sensitive as a predictor of preterm birth, however. Before 35 weeks of gestation a positive fetal fibronectin test predicts preterm birth only approximately 25% of the time. The test's sensitivity may be better earlier in pregnancy. In one study the fetal fibronectin test predicted 65% of preterm births occurring before 28 weeks of gestation when it was performed between 22 and 24 weeks. The test is often used to predict who will not go into preterm labor because preterm birth is very unlikely to occur in women with a negative result. Use of the fetal fibronectin test in women who are at low risk for preterm birth as a screening tool is not recommended (Iams et al., 2009). Cervical length Another possible predictor of preterm birth is endocervical length. Changes in cervical length occur before uterine activity; therefore cervical measurement can identify women in whom the labor process has begun. However, because preterm cervical shortening occurs over a period of weeks, neither digital nor ultrasound cervical examination is very sensitive at predicting imminent preterm birth (Iams et al., 2009). Women whose cervical length is more that 30 mm are unlikely to give birth prematurely even if they have symptoms of preterm labor (Iams & Romero, 2007; Iams et al., 2009). Causes of spontaneous preterm labor and birth Infection is currently the only factor that has been definitely shown to cause preterm labor. Another proposed cause of preterm labor and birth is bleeding at the site of placental implantation in the uterus in the first or second trimester of pregnancy. The resulting uteroplacental ischemia or hemorrhage at the decidual layer of the placenta may somehow activate the preterm labor process. Intrauterine inflammation is associated with infection, uterine vascular compromise, and decidual hemorrhage, and may contribute to preterm labor. Maternal and fetal stress, uterine overdistention, allergic reaction, and a decrease in progesterone are other factors that may play a part in initiating preterm labor. That preterm labor is caused by multiple pathologic processes that eventually result in uterine contractions, cervical changes, or membrane rupture is becoming increasingly clear (Iams et al., 2009; Romero & Lockwood, 2009). Two recent research studies suggest that recurrent preterm birth can be prevented in some women by administering prophylactic progesterone supplementation. In one study, women were given vaginal suppositories daily. In the other, women received weekly intramuscular injections of 17-alpha hydroxyprogesterone caproate. In both studies the risk of recurrent preterm birth was reduced by approximately one third. Exactly how progesterone works to prevent recurrent preterm birth is unclear; thus more study is necessary. Another important point to note is that prophylactic supplemental progesterone administration is recommended only for women who have previously given birth prematurely (Meis & Society for Maternal-Fetal Medicine, 2005; Romero & Lockwood, 2009). CARE MANAGEMENT ▪ Because all pregnant women must be considered at risk for preterm labor, assessment regarding knowledge of this condition begins early in pregnancy and continues throughout the prenatal period. Nursing diagnoses, interventions, and expected outcomes of care will be established for each woman based on her assessment findings (see the Nursing Process box: Preterm Labor and the Nursing Care Plan box: Preterm Labor). Prevention Primary prevention strategies that address risk factors associated with preterm labor and birth are less costly in human and financial terms than the high-tech and often lifelong care required by preterm infants and their families. Programs aimed at health promotion and disease prevention that encourage healthy lifestyles for the population in general and women of childbearing age in particular should be developed. Preconceptional counseling for women with a history of preterm birth may identify correctable risk factors. Smoking cessation, for example, has been shown to prevent preterm labor and birth (Freda, 2006; Iams et al., 2009). Many interventions intended to prevent spontaneous preterm birth have been recommended in the past and are still often prescribed. However, some of these interventions have not been shown to reduce the rate of preterm birth. Ongoing research is needed, especially given that our understanding of the pathophysiologic mechanisms of preterm birth is increasing (Iams et al., 2009). Early Recognition and Diagnosis Although preterm birth is often not preventable, early recognition of preterm labor is still essential in the effort to implement interventions that have been demonstrated to reduce neonatal morbidity and mortality These interventions include transfer of the mother before birth to a hospital equipped to care for her preterm infant, giving antibiotics in labor to prevent neonatal group B Streptococcus infection, and giving antenatal corticosteroids to the woman in preterm labor to prevent or reduce neonatal morbidity or mortality from conditions including respiratory distress syndrome, intraventricular hemorrhage, and necrotizing enterocolitis (Iams & Romero, 2007). Because more than half of preterm births occur in women without obvious risk factors, all pregnant women should be taught the symptoms of preterm labor (Box 22-3). Pregnant women must also be taught what to do if the symptoms of preterm labor occur. Interventions must be initiated promptly to allow time for corticosteroid administration and transfer to a hospital capable of providing care for the infant. See Patient Instructions for Self-Management box: What to Do If Symptoms of Preterm Labor Occur for recommended actions. In particular, patient education regarding any symptoms of uterine contractions or cramping between 20 and 37 weeks of gestation should be directed toward telling the woman that these symptoms are not just normal discomforts of pregnancy, but rather indications of possible preterm labor (Fig. 22-1). BOX 22-3 Signs and Symptoms of Preterm Labor UTERINE ACTIVITY • Uterine contractions that occur more frequently than every 10 minutes persisting for 1 hour or more • Uterine contractions that may be painful or painless DISCOMFORT • Lower abdominal cramping similar to gas pains; may be accompanied by diarrhea • Dull, intermittent low back pain (below the waist) • Painful, menstrual-like cramps • Suprapubic pain or pressure • Pelvic pressure or heaviness • Urinary frequency VAGINAL DISCHARGE • Change in character and amount of usual discharge: thicker (mucoid) or thinner (watery); bloody, brown, or colorless; increased amount; odor • Rupture of amniotic membranes PATIENT INSTRUCTIONS FOR SELF-MANAGEMENT What to Do if Symptoms of Preterm Labor Occur • Empty your bladder. • Drink two to three glasses of water or juice. • Lie down on your side for 1 hour. • Palpate for contractions. • If symptoms continue, call your health care provider, or go to the hospital. • If symptoms go away, resume light activity but not what you were doing when the symptoms began. • If symptoms return, call your health care provider, or go to the hospital. • If any of the following symptoms occur, call your health care provider immediately: • Uterine contractions every 10 minutes or less for 1 hour or more • Vaginal bleeding • Odorous vaginal discharge • Fluid leaking from the vagina The diagnosis of preterm labor is based on three major diagnostic criteria: 1. Gestational age between 20 and 37 weeks 2. Uterine activity (e.g., contractions) 3. Progressive cervical change (e.g., effacement of 80%, or cervical dilation of 2 cm or greater) If the presence of fetal fibronectin is used as another diagnostic criterion, a sample of cervical mucus for testing should be obtained before an examination for cervical changes because the lubricant used to examine the cervix can reduce the accuracy of the test for fetal fibronectin. Lifestyle Modifications Activity restriction Activity restriction, including bed rest and limited work, is a commonly prescribed intervention for the prevention of preterm birth. Bed rest, however, is not a benign intervention, and no evidence has been published in the literature to support the effectiveness of this intervention in reducing preterm birth rates (Iams et al., 2009). In fact, the American College of Obstetricians and Gynecologists (ACOG) (2003) states in its practice bulletin on management of preterm labor that bed rest should not be routinely recommended. Research indicates that bed rest causes adverse physical effects, including risk of thrombus formation, muscle atrophy, osteoporosis, and cardiovascular deconditioning (Iams & Romero, 2007). In many instances, these symptoms are not resolved by 6 weeks postpartum (Maloni & Park, 2005). Additionally, bed rest also affects women and their families psychologically, emotionally, socially, and financially. Box 22-4 lists adverse effects of bed rest. Restriction of sexual activity Restriction of sexual activity is also frequently recommended for women at risk for preterm birth. This intervention has not been shown to be effective at preventing preterm birth. However, sexual abstinence has not been studied in women with specific risk factors for preterm birth, such as a short cervix. Therefore more research is indicated (Iams et al., 2009). If, however, symptoms of preterm labor occur after sexual activity, then that activity may need to be curtailed until 37 weeks of gestation. Home care Women who are at high risk for preterm birth are commonly told that "taking it easy" at home for weeks or months would be best for them. Many health care providers now recommend only modified bed rest. The home care of the woman at risk for preterm birth is a challenge, however, for the nurse, who must assist the woman and her family in dealing with the many difficulties faced by families in which one member is incapacitated. The woman's environment can be modified for convenience by using tables and storage units around her bed or couch to keep essential items within reach (e.g., telephone, television, radio, tape or compact disc player, computer with Internet access, snacks, books, magazines, newspapers, items for hobbies) (Fig. 22-2). Ensuring that the bed or couch is near a window and the bathroom is also helpful. Covering the bed with an egg crate mattress can relieve discomfort. Women often find that following a daily schedule of meals, activities, and hygiene and grooming (e.g., shower, dressing in street clothes, applying make-up) reduces boredom and helps them maintain control and normalcy. See the Patient Instructions for Self-Management box: Coping with Activity Restriction (in Chapter 21) for more information. With modified bed rest, women are usually allowed bathroom privileges for toileting and showering and can be up to the table for meals. Suppression of Uterine Activity Tocolytics are medications given to arrest labor after uterine contractions and cervical change have occurred. Usually, tocolytic therapy will not prolong the pregnancy long enough for further fetal growth or maturation to occur. Rather, the goal of tocolytic therapy is to delay birth long enough to institute interventions that have been demonstrated to reduce neonatal morbidity and mortality (Iams et al., 2009). Maternal and fetal contraindications to tocolytic therapy are listed in Box 22-5. Box 22-6 describes nursing care for women receiving tocolytic therapy. Selecting the appropriate tocolytic medication requires consideration of each drug's effectiveness, risks, and side effects. No medication currently used for tocolysis in the United States has been approved by the U.S. Food and Drug Administration (FDA) for the purpose of arresting preterm labor. Instead, drugs marketed for other purposes, such as treatment of asthma or hypertension or as antiinflammatory or analgesic agents, are used on an "off-label" basis (i.e., drugs known to be effective for a specific purpose, although not specifically developed and tested for this purpose) (Iams et al., 2009). Important contraindications exist to the use of all tocolytics (see Box 22-5).

percutaneous umbilical blood sampling (PUBS) (also called cordocentesis)

Procedure during which a fetal umbilical vessel is accessed for blood sampling or for transfusions

amniocentesis

Procedure in which a needle is inserted through the abdominal and uterine walls to obtain amniotic fluid; used for assessment of fetal health and maturity

dystocia

Prolonged, painful, or otherwise difficult labor caused by various conditions associated with the five factors affecting labor (powers, passage, passenger, maternal position, and maternal emotions)

prolapse of the umbilical cord

Protrusion of the umbilical cord in advance of the presenting part

precipitous labor

Rapid or sudden labor lasting less than 3 hours from the onset of uterine contractions to complete birth of the fetus

premature rupture of membranes (PROM)

Rupture of the amniotic sac and leakage of amniotic fluid before the onset of labor at any gestational age

eclampsia

Severe complication of pregnancy of unknown cause and occurring more often in the primigravida than in multiparous women; characterized by new-onset grand mal seizures in a woman with preeclampsia occurring during pregnancy or shortly after birth

-sickle cell crisis- need hydration, oxygen and pain management

Sickle cell hemoglobinopathy is a disease caused by the presence of abnormal hemoglobin in the blood. Sickle cell trait (SA hemoglobin pattern) is sickling of the RBCs but with a normal RBC life span. Most people with sickle cell trait are asymptomatic. Approximately 1 in 12 African-American adults in the United States have sickle cell trait (Samuels, 2007). Women with sickle cell trait require genetic counseling and partner testing to determine their risk of producing children with sickle cell trait or disease. Women with sickle cell trait usually do well in pregnancy. However, they are at increased risk for preeclampsia, IUFD, preterm birth and low-birthweight infants, and postpartum endometritis. They are also at increased risk for UTIs and may be deficient in iron (Kilpatrick, 2009; Samuels, 2007). Sickle cell anemia (sickle cell disease) is a recessive, hereditary, familial hemolytic anemia that affects persons of African or Mediterranean ancestry. These individuals usually have abnormal hemoglobin types (SS or SC). The average life span of RBCs in a person with sickle cell anemia is only 5 to 10 days, in comparison to the 120 day life span of a normal RBC. Sickle cell anemia (SS disease) occurs in 1 in 708 African-Americans in the United States (Samuels, 2007). Persons with sickle cell anemia have recurrent attacks (crises) of fever and pain, most often in the abdomen, joints, or extremities, although virtually all organ systems can be affected. These attacks are attributed to vascular occlusion when RBCs assume a characteristic sickled shape. Crises are usually triggered by dehydration, hypoxia, or acidosis (Samuels). Women with sickle cell anemia require genetic counseling before pregnancy. All children born to a woman with sickle cell anemia will be affected in some way by the disease. The woman's partner must be tested to determine the couple's risk of producing children with sickle cell disease rather than sickle cell trait. Women with sickle cell anemia are at risk for poor pregnancy outcomes, including miscarriage, IUGR, and stillbirth. Although maternal mortality is rare, maternal morbidity is significant and includes an increased risk for preeclampsia and infection, particularly in the urinary tract and in the lungs. The frequency of painful crises also appears to be increased during pregnancy (Samuels, 2007). The woman will be monitored carefully during pregnancy for the development of urinary tract infection or preeclampsia. In addition, she will have serial ultrasound examinations to monitor fetal growth and will likely have antepartum fetal testing performed regularly during the third trimester. Infections are treated aggressively with antibiotics. If crises occur, they are managed with analgesia, oxygen, and hydration. Some authorities recommend prophylactic transfusions as a way to improve oxygen-carrying capacity and suppress the synthesis of sickle hemoglobin. Others, however, believe that prophylactic transfusions do not improve fetal or neonatal outcome (Samuels, 2007). NURSING ALERT Women with sickle cell anemia are not iron deficient. Therefore routine iron supplementation, even that found in prenatal vitamins, should be avoided because these women can develop iron overload (Samuels, 2007). If no complications occur, pregnancy can continue until term. Intrapartum, women with sickle cell disease should be encouraged to labor in a side-lying position. They may require supplemental oxygen. Adequate hydration should be maintained while preventing fluid overload. Conduction anesthesia (e.g., epidural or combined spinal epidural anesthesia) is recommended because it provides excellent pain relief. Vaginal birth is preferred. Cesarean birth should be performed only for obstetric indications (Samuels, 2007).

neonatal abstinence syndrome

Signs and symptoms associated with drug withdrawal in the neonate

kangaroo care

Skin-to-skin infant care, especially for preterm infants, that provides warmth to infant; infant is placed naked or diapered against the mother's or the father's bare chest and is covered with the parent's shirt or a warm blanket

clonus

Spasmodic alternation of muscular contraction and relaxation; counted in beats

augmentation of labor

Stimulation of ineffective uterine contractions after labor has started spontaneously but is not progressing satisfactorily

postpartum psychosis

Syndrome characterized by depression, delusions, and thoughts by the mother of harming herself or her infant

chronic hypertension

Systolic pressure of 140 mmHg or higher or diastolic pressure of 90 mmHg or higher that is present preconceptionally or occurs before 20 weeks of gestation

-rubella vaccine- need to wait 1 or 3 months before getting pregnant

TABLE 21-10 Maternal Infection: TORCH INFECTION: Rubella (3-day or German measles) (virus) MATERNAL EFFECTS: Rash, fever, mild symptoms such as headache, malaise, myalgias, and arthralgias; postauricular lymph nodes may be swollen; mild conjunctivitis FETAL EFFECTS: Approximately 50%-80% of fetuses exposed to the virus within 12 weeks after conception will show signs of congenital infection. Very few fetuses are affected if infection occurs after 18 weeks of gestation. The most common fetal anomalies associated with congenital rubella syndrome are deafness, eye defects (e.g., cataracts or retinopathy), central nervous system defects, and cardiac defects. COUNSELING: PREVENTION, IDENTIFICATION, AND MANAGEMENT: Vaccination of pregnant women contraindicated; pregnancy should be prevented for 1 month after vaccination. Women may breastfeed after vaccination and the vaccine can be administered along with immunoglobulin preparations such as Rh immune globulin.

-normal and abnormal glucose levels- fasting, 1 hour GTT, 3 hour GTT

TABLE 7-1 Recommended Laboratory Tests in Prenatal Period 1-hr glucose tolerance: Screens for gestational diabetes; performed at initial visit for women with risk factors; performed at 24-28 weeks for all pregnant women who are not already known to be diabetic 3-hr glucose tolerance: Screens for diabetes in women with elevated glucose level after 1-hr test; must have two elevated readings for diagnosis of gestational diabetes

-thrombophlebitis

THROMBOEMBOLIC DISEASE Thrombosis results from the formation of a blood clot or clots inside a blood vessel and is caused by inflammation (thrombophlebitis) or partial obstruction of the vessel. Three thromboembolic conditions are of concern in the postpartum period: 1. Superficial venous thrombosis—involvement of the superficial saphenous venous system 2. Deep venous thrombosis—involvement varies but can extend from the foot to the iliofemoral region 3. Pulmonary embolism—complication of deep venous thrombosis occurring when part of a blood clot dislodges and is carried to the pulmonary artery, where it occludes the vessel and obstructs blood flow to the lungs Incidence and Etiology The incidence of thromboembolic disease in the postpartum period varies from approximately 1 in 1000 to 1 in 2000 women (Pettker & Lockwood, 2007). The incidence has declined in the last 20 years because early ambulation after childbirth has become the standard practice. The major causes of thromboembolic disease are venous stasis and hypercoagulation, both of which are present in pregnancy and continue into the postpartum period. Other risk factors include operative vaginal birth, cesarean birth, history of venous thrombosis or varicosities, obesity, maternal age older than 35 years, multiparity, infection, immobility, and smoking. Women with associated genetic risk factors are also at risk (Pettker & Lockwood). Clinical Manifestations Superficial venous thrombosis is the most frequent form of postpartum thrombophlebitis. It is characterized by pain and tenderness in the lower extremity. Physical examination may reveal warmth, redness, and an enlarged, hardened vein over the site of the thrombosis. Deep vein thrombosis is more common during pregnancy than after the birth and is characterized by unilateral leg pain, calf tenderness, and swelling (Fig. 23-2). Physical examination may reveal redness and warmth, but many women may have few if any symptoms (Pettker & Lockwood, 2007). A positive Homans sign may be present, but further evaluation is needed because the calf pain may be attributed to other causes, such as a strained muscle resulting from the birthing position (Pettker & Lockwood). Acute pulmonary embolism is characterized by dyspnea and tachypnea (>20 breaths/min). Other signs and symptoms frequently seen include tachycardia (>100 beats/min), apprehension, cough, hemoptysis, elevated temperature, syncope, and pleuritic chest pain (Cunningham et al., 2005; Pettker & Lockwood). Physical examination is not a sensitive diagnostic indicator for thrombosis. Venography is the most accurate method for diagnosing deep venous thrombosis; however, it is an invasive procedure that is associated with serious complications. Noninvasive diagnostic methods are commonly used; these methods include real-time and color Doppler ultrasound. Cardiac auscultation may reveal murmurs with pulmonary embolism. Electrocardiograms are usually normal. Arterial oxygen pressure may be lower than normal (Katz, 2007a; Pettker & Lockwood, 2007). CARE MANAGEMENT Medical Management Superficial venous thrombosis is treated with analgesia (nonsteroidal antiinflammatory agents), rest with elevation of the affected leg, and elastic stockings (Cunningham et al., 2005; Katz, 2007a). Local application of heat also may be used. Deep venous thrombosis is initially treated with anticoagulant (usually continuous IV heparin) therapy, bed rest with the affected leg elevated, and analgesia. After the symptoms have decreased, the woman may be fitted with elastic stockings to use when she is allowed to ambulate. IV heparin therapy continues for 3 to 5 days or until symptoms resolve. Oral anticoagulant therapy (warfarin) is started during this time and will be continued for approximately 3 months. Continuous IV heparin therapy is used for pulmonary embolism until symptoms have resolved and is followed by subcutaneous heparin or oral anticoagulant therapy for up to 6 months (Pettker & Lockwood, 2007). Nursing Interventions In the hospital setting, nursing care of the woman with a thrombosis consists of continued assessments: inspection and palpation of the affected area; palpation of peripheral pulses; checking Homans sign; measurement and comparison of leg circumferences; inspection for signs of bleeding; monitoring for signs of pulmonary embolism, including chest pain, coughing, dyspnea, and tachypnea; and respiratory status for presence of crackles. Laboratory reports are monitored for prothrombin or partial thromboplastin times. The woman and her family are assessed for their level of understanding about the diagnosis and their ability to cope during the unexpected extended period of recovery. Interventions include explanations and education about the diagnosis and the treatment. The woman will need assistance with personal care as long as she is on bed rest; the family should be encouraged to participate in the care if that is what she and they wish. While the woman is on bed rest, she should be encouraged to change positions frequently but to avoid placing the knees in a sharply flexed position, which could cause pooling of blood in the lower extremities. She should also be cautioned to avoid rubbing the affected area, given that this action can cause the clot to dislodge. Once the woman is allowed to ambulate, she is taught how to prevent venous congestion by putting on the elastic stockings before getting out of bed. Heparin and warfarin are administered as ordered, and the physician is notified if clotting times are outside the therapeutic level. If the woman is breastfeeding, she is assured that neither heparin nor warfarin is excreted in significant quantities in breast milk. If the infant has been discharged, the family is encouraged to bring the infant for feedings as permitted by hospital policy; the mother can also express milk to be sent home. Pain can be managed with a variety of measures. Position changes, elevating the leg, and application of moist warm heat may decrease discomfort. Administration of analgesics and antiinflammatory medications may be needed. NURSING ALERT Medications containing aspirin are not given to women receiving anticoagulant therapy because aspirin inhibits synthesis of clotting factors and can lead to prolonged clotting time and increased risk of bleeding. The woman is usually discharged home with oral anticoagulants and will need explanations about the treatment schedule and possible side effects. If subcutaneous injections are to be given, the woman and family are taught how to administer the medication and about site rotation. The woman and her family also should be given information about safe care practices to prevent bleeding and injury while she is receiving anticoagulant therapy, such as using a soft toothbrush and using an electric razor. She also will need information about follow-up with her health care provider to monitor clotting times and to make sure the correct dose of anticoagulant therapy is maintained. The woman also should use a reliable method of contraception if taking warfarin, because this medication is considered teratogenic. Oral contraceptives are contraindicated because of the increased risk for thrombosis (Gilbert, 2007).

corrected age

Taking into account the gestational age and the postnatal age of a preterm infant when determining expectations for development

-terbutaline- indications, side effects- pulmonary edema- dyspnea

Terbutaline (Brethine), the best-known beta-adrenergic agonist medication used for tocolysis, works by relaxing uterine smooth muscle as a result of stimulation of beta2-receptors on uterine smooth muscle. A single dose of terbutaline given subcutaneously may help diagnose preterm labor. In one study, women whose contractions persisted or recurred after a single injection of terbutaline were more likely to actually be in preterm labor than those whose contractions ceased. Terbutaline is often given subcutaneously to facilitate maternal transfer to a tertiary-care center or to initiate tocolytic therapy while another agent with a slower onset of action is being administered concurrently. Long-term oral or subcutaneous administration of terbutaline has not been proven to be effective at reducing prematurity or neonatal morbidity (Iams & Romero, 2007; Iams et al., 2009). Medication Guide: Tocolytic Therapy for Preterm Labor Medication: BETA-ADRENERGIC AGONIST (BETA-MIMETIC): Terbutaline (Brethine) Action: Relaxes smooth muscles, inhibiting uterine activity, and causing bronchodilation Dosage and Route: • Subcutaneous injection of 0.25 mg every 4 hrs • Treatment should last no longer than 24 hrs • Discontinue use if intolerable side effects occur Adverse Effects: Maternal: (most are mild and of limited duration) • Tachycardia, chest discomfort, palpitations, arrhythmias • Tremors, dizziness, nervousness • Headache • Nasal congestion, • Nausea and vomiting • Hypokalemia • Hyperglycemia • Hypotension Intolerable adverse reactions: • Tachycardia greater than 130 beats/min • BP less than 90/60 • Chest pain • Cardiac dysrhythmias • Myocardial infarction • Pulmonary edema • Should not be used in women with a history of cardiac disease, pregestational or gestational diabetes, severe preeclampsia or eclampsia, or hyperthyroidism, or with significant hemorrhage • Use cautiously if woman has controlled migraines • Myocardial infarction leading to death has been reported after use • Validate that woman is in PTL and is over 20 weeks and less than 35 weeks of gestation Fetal: • Tachycardia • Hyperinsulinemia • Hyperglycemia Nursing Considerations: • Should not be used in women with a history of cardiac disease, pregestational or gestational diabetes, severe preeclampsia or eclampsia, or hyperthyroidism, or with significant hemorrhage • Use cautiously if woman has controlled migraines • Myocardial infarction leading to death has been reported after use • Validate that woman is in PTL and is over 20 weeks and less than 35 weeks of gestation • Assess woman and fetus according to agency protocol, being alert for adverse reactions • Assess maternal glucose and potassium levels before treatment is initiated and on occasion during treatment. Significant hyperglycemia (greater than 180 mg/dl) and hypokalemia (less than 2.5 mEq/L) may occur • Notify physician if the woman exhibits the following: • Maternal heart rate greater than 130 beats/min; arrhythmias, chest pain • BP less than 90/60 mm Hg • Signs of pulmonary edema (e.g., dyspnea, crackles, decreased Sao2) • Fetal heart rate greater than 180 beats/min • Hyperglycemia occurs more frequently in women who are being treated simultaneously with corticosteroids • Ensure that propranolol (Inderal) is available to reverse adverse effects related to cardiovascular function

contraction stress test (CST) (also called oxytocin challenge test [OCT])

Test to stimulate uterine contractions for the purpose of assessing fetal response; a healthy fetus does not react to contractions, whereas a compromised fetus demonstrates late decelerations in the fetal heart rate that are indicative of uteroplacental insufficiency

bereavement

The feelings of loss, pain, desolation, and sadness that occur after the death of a loved one

gestational hypertension

The new onset of hypertension without proteinuria after week 20 of pregnancy

11. Less than 10% of women who are substance abusers receive treatment for their addiction during pregnancy. True or False?

True

urinary incontinence (UI)

Uncontrollable leakage of urine

hypertonic uterine dysfunction

Uncoordinated, painful, frequent uterine contractions that do not cause cervical dilation and effacement; primary dysfunctional labor

cerclage

Use of a nonabsorbable suture to keep a premature dilating cervix closed; usually removed when pregnancy is at term

nonnutritive sucking

Use of a pacifier by infants; may include thumb or fingers

Doppler blood flow analysis

Use of ultrasound for noninvasive measurement of blood flow in the fetus and placenta

-uterine atony

Uterine atony is marked hypotonia of the uterus. Normally, placental separation and expulsion are facilitated by contraction of the uterus, which also prevents hemorrhage from the placental site. The corpus is in essence a basket weave of strong, interlacing smooth-muscle bundles through which many large maternal blood vessels pass (see Fig. 2-3). If the uterus is flaccid after detachment of all or part of the placenta, brisk venous bleeding occurs, and normal coagulation of the open vasculature is impaired and continues until the uterine muscle is contracted. Uterine atony is the leading cause of PPH, complicating approximately 1 in 20 births (Francois & Foley, 2007). It is associated with high parity, hydramnios, a macrosomic fetus, and multifetal gestation. In such conditions the uterus is "overstretched" and contracts poorly after the birth. Other causes of atony include traumatic birth, use of halogenated anesthesia (e.g., halothane) or magnesium sulfate, rapid or prolonged labor, chorioamnionitis, and use of oxytocin for labor induction or augmentation (Francois & Foley). PPH in a previous pregnancy is a predominant risk factor for recurrent PPH (Kominiarek & Kilpatrick, 2007).

preterm labor

Uterine contractions causing cervical change that occur between 20 weeks and 37 weeks of pregnancy

forceps-assisted birth

Vaginal birth in which forceps (i.e., curved-bladed instruments) are used to assist in the birth of the fetal head

trophic feedings

Very small feedings given to stimulate maturation of the gut

hypotonic uterine dysfunction

Weak, ineffective uterine contractions usually occurring in the active phase of labor; often related to cephalopelvic disproportion or malposition of the fetus; secondary uterine inertia

thrombophlebitis

inflammation of a vein with secondary clot formation

8. MSAFP levels have been used as a screening tool for __________ in pregnancy.

neural tube defects

11. The condition in which the placenta is implanted in the lower uterine segment near or over the internal cervical os is __________.

placenta previa


Related study sets

Project Management Ch. 1 Quizzes

View Set

Issues in International Business :)

View Set

Section 6.1: Definition of Percent and Converting Percents

View Set